final exam goodies

¡Supera tus tareas y exámenes ahora con Quizwiz!

After teaching a group of students about intestinal obstruction, the instructor determines that the teaching was effective when the students identify which of the following as a cause of a functional obstruction?

Abdominal surgery

In addition to teaching a client with constipation to increase dietary fiber intake to 25 g/day, which of the following would the nurse include as important?

Adding fiber-rich foods to the diet gradually

When preparing a client for a hemorrhoidectomy, the nurse should take which action?

Administer an enema as ordered.

A client admitted with severe epigastric abdominal pain radiating to the back is vomiting and reports difficulty breathing. Upon assessment, the nurse determines that the client is experiencing tachycardia and hypotension. Which actions are priority interventions for this client? Select all that apply.

Administer pain-relieving medication Administer electrolytes Administer plasma Assist the client to a semi-Fowler position Hinkle, J.L., & Cheever, K.H., Brunner & Suddarth's Textbook of Medical-Surgical Nursing, 14th ed., Philadelphia, Wolters Kluwer, 2018, Chapter 50: Assessment and Management of Patients With Biliary Disorders, p. 1434.

The nurse is to insert a postpyloric feeding tube. How can the nurse aid in placement of the tube past the pylorus?

Administer prescribed metoclopramide.

The nurse is caring for a patient who has dumping syndrome from high carbohydrate foods being administered over a period of less than 20 minutes. What is a nursing measure to prevent or minimize the dumping syndrome?

Administer the feeding with the patient in semi-Fowler's position to decrease transit time influenced by gravity.

A nurse caring for a client with small-bowel obstruction should plan to implement which nursing intervention first?

Administering I.V. fluids

A client with a history of alcohol abuse comes to the emergency department and complains of abdominal pain. Laboratory studies help confirm a diagnosis of acute pancreatitis. The client's vital signs are stable, but the client's pain is worsening and radiating to his back. Which intervention takes priority for this client?

Administering morphine I.V. as ordered Hinkle, J.L., & Cheever, K.H., Brunner & Suddarth's Textbook of Medical-Surgical Nursing, 14th ed., Philadelphia, Wolters Kluwer, 2018, Chapter 50: Assessment and Management of Patients With Biliary Disorders, Relieving Pain and Discomfort, p. 1443.

A client is recovering from percutaneous endoscopic gastrostomy (PEG) tube placement. The nurse

Administers an initial bolus of 50 mL water Chapter 44: Digestive and Gastrointestinal Treatment Modalities - Page 1254 The first fluid nourishment may consist of water, saline, or 10% dextrose. This may be administered as a bolus of 30 to 60 mL. By the second day, formula feeding may begin. A gauze dressing is applied between the tube insertion site and the gastrostomy tube. The dressing is changed daily or as needed. The nurse gently manipulates the stabilizing disk daily to prevent skin breakdown.

The nurse is assessing an 80-year-old client for signs and symptoms of gastric cancer. The nurse differentiates which as a sign/symptom of gastric cancer in the geriatric client, but not in a client under the age of 75?

Agitation

A client reporting shortness of breath is admitted with a diagnosis of cirrhosis. A nursing assessment reveals an enlarged abdomen with striae, an umbilical hernia, and 4+ pitting edema of the feet and legs. What is the most important data for the nurse to monitor?

Albumin Hinkle, J.L., & Cheever, K.H., Brunner & Suddarth's Textbook of Medical-Surgical Nursing, 14th ed., Philadelphia, Wolters Kluwer, 2018, Chapter 49: Assessment and Management of Patients With Hepatic Disorders, p. 1385.

What intervention does the nurse anticipate providing for the patient with ascites that will help correct the decrease in effective arterial blood volume that leads to sodium retention?

Albumin infusion Hinkle, J.L., & Cheever, K.H., Brunner & Suddarth's Textbook of Medical-Surgical Nursing, 14th ed., Philadelphia, Wolters Kluwer, 2018, Chapter 49: Assessment and Management of Patients With Hepatic Disorders, Paracentesis, p. 1387.

A nurse is teaching a client and the client's family about chronic pancreatitis. Which are the major causes of chronic pancreatitis?

Alcohol consumption and smoking Hinkle, J.L., & Cheever, K.H., Brunner & Suddarth's Textbook of Medical-Surgical Nursing, 14th ed., Philadelphia, Wolters Kluwer, 2018, Chapter 50: Assessment and Management of Patients With Biliary Disorders, Chronic Pancreatitis, p. 1445.

A nurse is teaching a client about the types of chronic liver disease. The teaching is determined to be effective when the client correctly identifies which type of cirrhosis as being caused by scar tissue surrounding portal areas?

Alcoholic cirrhosis Hinkle, J.L., & Cheever, K.H., Brunner & Suddarth's Textbook of Medical-Surgical Nursing, 14th ed., Philadelphia, Wolters Kluwer, 2018, Chapter 49: Assessment and Management of Patients With Hepatic Disorders, p. 1406.

A client reports having increased incidence of constipation. What can cause constipation?

All options are correct.

The nurse is assessing a client with hepatic cirrhosis for mental deterioration. For what clinical manifestations will the nurse monitor? Select all that apply.

Alterations in mood Agitation Insomnia Chapter 49: Assessment and Management of Patients With Hepatic Disorders - Page 1394 The earliest symptoms of hepatic encephalopathy include both mental status changes and motor disturbances. The client appears confused and unkempt and has alterations in mood and sleep patterns. The client tends to sleep during the day and has restlessness and insomnia at night. To assess for mental deterioration, the nurse will assess general behavior, orientation, and speech as well as cognitive abilities and speech patterns.

A public health nurse plans teaching on the neurological morbid diseases and conditions associated with obesity. Which diseases or conditions will the nurse identify in the teaching? Select all that apply.

Alzheimer disease Anxiety Depression Stroke Hinkle, J.L., & Cheever, K.H., Brunner & Suddarth's Textbook of Medical-Surgical Nursing, 14th ed., Philadelphia, Wolters Kluwer, 2018, Chapter 48: Assessment and Management of Patients with Obesity, p. 1359.

A nurse researcher evaluates the epidemiology of obesity among gender and ethnicity of adults in the United States. Which findings correlate with data the nurse researcher finds? Select all that apply.

Among women, the highest prevalence of obesity is among non-Hispanic blacks. Among men, the highest prevalence of obesity is among Hispanics. Hinkle, J.L., & Cheever, K.H., Brunner & Suddarth's Textbook of Medical-Surgical Nursing, 14th ed., Philadelphia, Wolters Kluwer, 2018, Chapter 48: Assessment and Management of Patients with Obesity, p. 1359.

The nurse practitioner suspects that a patient may have a gastric ulcer after completing a history and physical exam. Select an indicator that can be used to help establish the distinction.

Amount of hydrochloric acid (HCL) secretion in the stomach

Which of the following digestive enzymes aids in the digesting of starch?

Amylase

The nurse is performing a rectal assessment and notices a longitudinal tear or ulceration in the lining of the anal canal. The nurse documents the finding as which condition?

Anal fissure

The American Cancer Society recommends routine screening to detect colorectal cancer. Which screening test for colorectal cancer should a nurse recommend?

Annual digital examination after age 40

A client is admitted with a new onset of pyloric obstruction. What client symptoms should the nurse anticipate? Select all that apply.

Anorexia Nausea and vomiting Epigastric fullness

A client is scheduled for magnetic resonance imaging (MRI). During the client teaching, what will the nurse discuss?

"Do you experience any claustrophobia?"

A nurse provides nutritional teaching to a client who is scheduled to have bariatric surgery. Which statements will the nurse include on fluid intake after the procedure? Select all that apply.

"Drink plenty of water." "Avoid drinking liquid calories." "Drink frequent, small amounts of liquids." "Drink liquids 30-60 minutes after eating." Hinkle, J.L., & Cheever, K.H., Brunner & Suddarth's Textbook of Medical-Surgical Nursing, 14th ed., Philadelphia, Wolters Kluwer, 2018, Chapter 48: Assessment and Management of Patients with Obesity, p. 1372.

An adolescent client with multiple dental caries is discussing diet with the nurse. What client statement identifies a risk factor for dental caries?

"I drink a can of carbonated soda at lunch every day."

A nurse is teaching an older adult client about good bowel habits. Which statement by the client indicates to the nurse that additional teaching is required?

"I need to use laxatives regularly to prevent constipation."

A nurse is caring for a client who will undergo bariatric surgery. Which nutritional recommendation will the nurse include in the client teaching?

"Increase your intake of plant-based proteins."

A client asks the nurse why the physician ordered the blood test carcinoembryonic antigen (CEA). The nurse answers:

"It indicates if a cancer is present."

A nurse reviews a client's medication list and notes the client takes orlistat for the treatment of obesity and to promote weight loss. Which client teaching will the nurse include that best promotes health in the client taking this medication?

"Make sure and take a daily multivitamin." Hinkle, J.L., & Cheever, K.H., Brunner & Suddarth's Textbook of Medical-Surgical Nursing, 14th ed., Philadelphia, Wolters Kluwer, 2018, Chapter 48: Assessment and Management of Patients with Obesity, Pharmacologic Therapy, p. 1364.

A client with a peptic ulcer is diagnosed with Helicobacter pylori infection. The nurse is teaching the client about the medications prescribed, including metronidazole, omeprazole, and clarithromycin. Which statement by the client indicates the best understanding of the medication regimen?

"The medications will kill the bacteria and stop the acid production."

A nurse cares for an older adult client and teaches the client about age-related changes of the biliary tract. What statements will the nurse include when discussing age-related changes that occur in the pancreas of the older adult? Select all that apply.

"The pancreas develops fibrous material with age." "The pancreas develops fatty deposits with age." "The pancreas decreases secretion of enzymes with age." "The pancreas decreases bicarbonate secretion with age." Hinkle, J.L., & Cheever, K.H., Brunner & Suddarth's Textbook of Medical-Surgical Nursing, 14th ed., Philadelphia, Wolters Kluwer, 2018, Chapter 50: Assessment and Management of Patients With Biliary Disorders, p. 1430.

A client with obesity is prescribed lorcaserin for weight loss. The client reports dry mouth. What is the nurse's best response?

"This is an expected finding with this medication." Hinkle, J.L., & Cheever, K.H., Brunner & Suddarth's Textbook of Medical-Surgical Nursing, 14th ed., Philadelphia, Wolters Kluwer, 2018, Chapter 48: Assessment and Management of Patients with Obesity, p. 1365.

The nurse is instructing a client prior to a colonoscopy. The client states, "Why do I have to drink this disgusting liquid?" The nurse is most correct to verbalize the goal of the oral preparation as which of the following?

"To cleanse the bowel to promote clear visualization of structures"

A client who has just been diagnosed with hepatitis A asks, "How did I get this disease?" What is the nurse's best response?

"You may have eaten contaminated restaurant food." Chapter 49: Assessment and Management of Patients With Hepatic Disorders - Page 1398 Hepatitis A virus typically is transmitted by the oral-fecal route — commonly by consuming food contaminated by infected food handlers. The virus isn't transmitted by the I.V. route, blood transfusions, or unprotected sex. Hepatitis B can be transmitted by I.V. drug use or blood transfusion. Hepatitis C can be transmitted by unprotected sex.

A nurse is caring for a client who has had extensive abdominal surgery and is in critical condition. The nurse notes that the complete blood count shows an 8 G/dL hemoglobin and a 30% hematocrit. Dextrose 5% in half-normal saline solution is infusing through a triple lumen central catheter at 125 mL/hr. The physician orders include the following: Gentamicin 80 mg IV piggyback in 50 mL D5W over 30 minutes Zantac 50 mg IV piggyback in 50 mL D5W over 30 minutes One unit of 250 mL of PRBCs (packed red blood cells) over 3 hours Flush the nasogastric tube with 30 mL normal saline every 2 hours. How many milliliters should the nurse record as the intake for the 8-hour shift? Enter the correct number ONLY.

1470

A nurse caring for a client who has had radical neck surgery notices an abnormal amount of serosanguineous secretions in the wound suction unit during the first postoperative day. What is an expected, normal amount of drainage?

Approximately 80 to 120 mL

A nurse caring for a client who has had radical neck surgery notices an abnormal amount of serosanguineous secretions in the wound suction unit during the first postoperative day. What is an expected, normal amount of drainage?`

Approximately 80 to 120 mL

While caring for a patient who has had radical neck surgery, the nurse notices an abnormal amount of serosanguineous secretions in the wound suction unit during the first postoperative day. What does the nurse know is an expected amount of drainage in the wound unit?

Approximately 80 to 120 mL

The nurse completing a plan of care for a client with cirrhosis who has ascites and 4+ pitting edema of the feet and legs identifies a nursing diagnosis of risk for impaired skin integrity. Which nursing intervention is appropriate for this problem?

Arrange for a low air loss bed.

A client has undergone a radical neck dissection. His skin graft site is pale. This indicates which condition?

Arterial thrombosis

The nurse fills a tube feeding bag with two 8-oz cans of commercially prepared formula. The client is to receive the formula at 80 mL/hour via continuous gastrostomy feeding tube and pump. How many hours will this bag of formula run before becoming empty? Record your answer using a whole number.

6

Pharmacologic therapy frequently is used to dissolve small gallstones. It takes about how many months of medication with UDCA or CDCA for stones to dissolve?

6 to 12 Hinkle, J.L., & Cheever, K.H., Brunner & Suddarth's Textbook of Medical-Surgical Nursing, 14th ed., Philadelphia, Wolters Kluwer, 2018, Chapter 50: Assessment and Management of Patients With Biliary Disorders, Pharmacologic Therapy, p. 1434.

A patient is receiving pharmacologic therapy with ursodeoxycholic acid or chenodeoxycholic-cholic acid for treatment of small gallstones. The patient asks the nurse how long the therapy will take to dissolve the stones. What is the best answer the nurse can give?

6 to 12 months Hinkle, J.L., & Cheever, K.H., Brunner & Suddarth's Textbook of Medical-Surgical Nursing, 14th ed., Philadelphia, Wolters Kluwer, 2018, Chapter 50: Assessment and Management of Patients With Biliary Disorders, Pharmacologic Therapy, p. 1434.

Ursodeoxycholic acid (UDCA) has been used to dissolve small, radiolucent gallstones. Which duration of therapy is required to dissolve the stones?

7 months Hinkle, J.L., & Cheever, K.H., Brunner & Suddarth's Textbook of Medical-Surgical Nursing, 14th ed., Philadelphia, Wolters Kluwer, 2018, Chapter 50: Assessment and Management of Patients With Biliary Disorders, p. 1434.

What is the primary nursing diagnosis for a client with a bowel obstruction?

Deficient fluid volume

An elderly client states, "I don't understand why I have so many caries in my teeth." What assessment made by the nurse places the client at risk for dental caries?

Exhibiting hemoglobin A1C 8.2

Residual content is checked before each intermittent tube feeding. The patient would be reassessed if the residual, on two occasions, was:

Greater than 200 mL.

The nurse is caring for a client during the postoperative period following radical neck dissection. Which finding should be reported to the physician?

High epigastric pain and/or discomfort

Which of the following is a risk factor for the development of diabetes mellitus? Select all that apply.

Hypertension Obesity Family history Age greater of 45 years or older History of gestational diabetes Hinkle, J.L., & Cheever, K.H., Brunner & Suddarth's Textbook of Medical-Surgical Nursing, 14th ed., Philadelphia, Wolters Kluwer, 2018, Chapter 48: Assessment and Management of Patients with Obesity, Chart 51-1, p. 1457.

A nurse is inserting a nasogastric tube in an alert client. During the procedure, the client begins to cough constantly and has difficulty breathing. The nurse suspects the nasogastric tube is

Inserted into the lungs

Which mouth condition is most associated with HIV infection?

Kaposi sarcoma

A nurse is caring for a client with a BMI of 35 kg/m2 who is wanting to lose weight. What is the initial recommendation the nurse will expect from the client's health care provider?

Lifestyle modification

A client with cirrhosis has a massive hemorrhage from esophageal varices. Balloon tamponade is used temporarily to control hemorrhage and stabilize the client. In planning care, the nurse gives the highest priority to which goal?

Maintaining the airway Hinkle, J.L., & Cheever, K.H., Brunner & Suddarth's Textbook of Medical-Surgical Nursing, 14th ed., Philadelphia, Wolters Kluwer, 2018, Chapter 49: Assessment and Management of Patients With Hepatic Disorders, p. 1390.

A client with cancer has a neck dissection and laryngectomy. An intervention that the nurse will do is:

Make a notation on the call light system that the client cannot speak.

Which of the following medications, used in the treatment of GERD, accelerate gastric emptying?

Metoclopramide (Reglan)

What part of the GI tract begins the digestion of food?

Mouth

Celiac sprue is an example of which category of malabsorption?

Mucosal disorders causing generalized malabsorption

A nurse admits a woman reporting severe right upper quadrant pain after eating dinner. What client risk factors lead the nurse to suspect gallbladder disease? Select all that apply.

Multiparous Obese Older than 40 years old History of diabetes mellitus Hinkle, J.L., & Cheever, K.H., Brunner & Suddarth's Textbook of Medical-Surgical Nursing, 14th ed., Philadelphia, Wolters Kluwer, 2018, Chapter 50: Assessment and Management of Patients With Biliary Disorders, p. 1431.

The nurse is admitting a client with a diagnosis of diverticulitis and assesses that the client has a board-like abdomen, no bowel sounds, and reports of severe abdominal pain. What is the nurse's first action?

Notify the health care provider.

A client has been taking a 10-day course of antibiotics for pneumonia. The client has been having white patches that look like milk curds in the mouth. What treatment will the nurse educate the client about?

Nystatin

A nurse is reviewing the health history of a client who has been diagnosed with endometrial cancer. Which of the following would the nurse identify as a factor increasing this client's risk? Select all that apply.

Obesity Nulliparity Hinkle, J.L., & Cheever, K.H., Brunner & Suddarth's Textbook of Medical-Surgical Nursing, 14th ed., Philadelphia, Wolters Kluwer, 2018, Chapter 48: Assessment and Management of Patients with Obesity, Chart 57-9, p. 1709.

A nurse is preparing a presentation for a local community group addressing the influences on health care delivery. Which of the following would the nurse include in presentation when describing disease patterns?

Obesity along with conditions associated with it has become a major health concern. Hinkle, J. L., Cheever, K. H. Brunner & Suddarth's Textbook of Medical-Surgical Nursing , 14th ed., Philadelphia: Wolters Kluwer Health/Lippincott Williams & Wilkins, 2018, Chapter 1: Health Care Delivery and Evidence-Based Nursing Practice, p. 6.

A client has a blockage of the passage of bile from a stone in the common bile duct. What type of jaundice does the nurse suspect this client has?

Obstructive jaundice

A nurse practitioner prescribes drug therapy for a patient with peptic ulcer disease. Choose the drug that can be used for 4 weeks and has a 90% chance of healing the ulcer.

Omeprazole

A client has undergone a liver biopsy. Which postprocedure position is appropriate?

On the right side Hinkle, J.L., & Cheever, K.H., Brunner & Suddarth's Textbook of Medical-Surgical Nursing, 14th ed., Philadelphia, Wolters Kluwer, 2018.

The nurse is caring for a client with intussusception of the bowel. What does the nurse understand occurs with this disorder?

One part of the intestine telescopes into another portion of the intestine.

A nurse researcher analyzes data trends of obesity in both the United States and globally. What statistics does the nurse determine is true regarding the epidemiology of obesity? Select all that apply.

Over 600 million adults have obesity worldwide The prevalence of obesity has more than doubled since 1975 The prevalence of obesity in the United States is the highest in the world Hinkle, J.L., & Cheever, K.H., Brunner & Suddarth's Textbook of Medical-Surgical Nursing, 14th ed., Philadelphia, Wolters Kluwer, 2018, Chapter 48: Assessment and Management of Patients with Obesity, p. 1358.

A nurse researcher studies the pathophysiology and etiology of obesity. What does the nurse discover is true regarding the "thrifty gene" theory of obesity?

Over time, we have become efficient in food storage and deposition of fat stores.

Which of the following is a leading health indicator to be used to measure the health of the nation?

Overweight and obesity Hinkle, J. L., Cheever, K. H. Brunner & Suddarth's Textbook of Medical-Surgical Nursing , 14th ed., Philadelphia: Wolters Kluwer Health/Lippincott Williams & Wilkins, 2018, Chapter 4: Health Education and Health Promotion, p. 57.

A male client in a wheelchair comes in for his yearly physical examination. He is unable to stand. The nurse retrieves the wheelchair scale to obtain an accurate weight. The nurse understands the importance of this assessment with this client. What is the nurse's reasoning for obtaining an accurate weight?

People with disabilities have an increased incidence of obesity.

The nurse is conducting a community education class on gastritis. The nurse includes that chronic gastritis caused by Helicobacter pylori is implicated in which disease/condition?

Peptic ulcers

A client undergoing a diagnostic examination for gastrointestinal disorder was given polyethylene glycol/electrolyte solution as a part of the test preparation. Which of the following measures should the nurse take once the solution is administered?

Permit the client to drink only clear liquids.

Which of the following manifestations are associated with a deficiency of vitamin B12? Select all that apply.

Pernicious anemia Macrocytic anemia Thrombocytopenia

A nurse cares for a client with obesity who reports taking "a medication of weight loss" but cannot remember the name of it. The client also reports nervousness and feeling "jittery". Which medication is the client most likely taking?

Phentermine

A nurse cares for an older adult client with obesity who also has glaucoma. Which obesity medication is contraindicated in this client?

Phentermine

A nurse teaches a client with obesity about the various medication options for the treatment of obesity. Which medication will the nurse include when teaching the client about the class of medications that stimulate noradrenergic receptors?

Phentermine Hinkle, J.L., & Cheever, K.H., Brunner & Suddarth's Textbook of Medical-Surgical Nursing, 14th ed., Philadelphia, Wolters Kluwer, 2018, Chapter 48: Assessment and Management of Patients with Obesity, p. 1365.

One difference between cholesterol stones (left) and the stones on the right are that the ones on the right account for only 10% to 25% of cases of stones in the United States. What is the name of the stones on the right?

Pigment Hinkle, J.L., & Cheever, K.H., Brunner & Suddarth's Textbook of Medical-Surgical Nursing, 14th ed., Philadelphia, Wolters Kluwer, 2018, Chapter 50: Assessment and Management of Patients With Biliary Disorders, Figure 50-2, p. 1431.

A nurse is caring for a client who is suspected to have developed a peptic ulcer hemorrhage. Which action would the nurse perform first?

Place the client in a recumbent position with the legs elevated.

The nurse is caring for a client who has a gastrostomy tube feeding. Upon initiating care, the nurse aspirates the gastrotomy tube for gastric residual volume (GRV) and obtains 200 mL of gastric contents. What is the priority action by the nurse?

Place the client in a semi-Fowler's position with the head of the bed at 45 degrees.

A nurse is interviewing a client about past medical history. Which preexisting condition may lead the nurse to suspect that a client has colorectal cancer?

Polyps

Which diagnostic produces images of the body by detecting the radiation emitted from radioactive substances?

Positron emission tomography (PET) Chapter 43: Assessment of Digestive and Gastrointestinal Function - Page 1236 PET produces images of the body by detecting the radiation emitted from radioactive substances. CT provides cross-sectional images of abdominal organs and structures. MRI uses magnetic fields and radio waves to produce an image of the area being studied. Fibroscopy of the upper GI tract allows direct visualization of the esophageal, gastric, and duodenal mucosa through a lighted endoscope.

The nurse is teaching a client with peptic ulcer disease who has been prescribed misoprostol. What information from the nurse would be most accurate about misoprostol?

Prevents ulceration in clients taking nonsteroidal anti-inflammatory drugs (NSAIDs)

The nurse is caring for a client who is scheduled for a percutaneous liver biopsy. Which diagnostic test is obtained prior?

Prothrombin time (PT) The client must have coagulation studies before the procedure such as a PT or PTT because a major complication after a liver biopsy is bleeding. Clients at risk for serious bleeding may receive precautionary vitamin K. A complete blood count and blood chemistry may be completed for baseline values. Typically, an ESR is not associated with the procedure.

A nurse practitioner, who is treating a patient with GERD, knows that responsiveness to this drug classification is validation of the disease. The drug classification is:

Proton pump inhibitors.

A client with oral cancer reports dryness of the mouth. What is the nurse's best response?

Provide a humidifier for the client to use while sleeping.

A client with a disorder of the oral cavity cannot tolerate tooth brushing or flossing. Which strategy should the nurse use to assist the client?

Provide the client with an irrigating solution of baking soda and warm water.

Which term refers to the symptom of gastroesophageal reflux disease (GERD), which is characterized by a burning sensation in the esophagus?

Pyrosis

The nurse cares for a client who is post op bariatric surgery and reports dysphagia. Which procedures are most likely to cause these symptoms? Select all that apply.

RYGB Gastric band Modified RYGB

A client who is postoperative from bariatric surgery reports foul-smelling, fatty stools. What is the nurse's understanding of the primary reason for this finding?

Rapid gastric dumping Hinkle, J.L., & Cheever, K.H., Brunner & Suddarth's Textbook of Medical-Surgical Nursing, 14th ed., Philadelphia, Wolters Kluwer, 2018, Chapter 48: Assessment and Management of Patients with Obesity, p. 1372.

A client is admitted to the health care facility with abdominal pain, a low-grade fever, abdominal distention, and weight loss. The physician diagnoses acute pancreatitis. What is the primary goal of nursing care for this client?

Relieving abdominal pain Hinkle, J.L., & Cheever, K.H., Brunner & Suddarth's Textbook of Medical-Surgical Nursing, 14th ed., Philadelphia, Wolters Kluwer, 2018, Chapter 50: Assessment and Management of Patients With Biliary Disorders, Pain Management, p. 1442.

The nurse determines a client scheduled to undergo an abdominal ultrasonography should receive which instruction?

Restrict eating of solid food for 6 to 8 hours before the test.

The nurse conducts education related to test preparation for a client scheduled to undergo an abdominal ultrasonography. The nurse should give the client which instruction?

Restrict eating of solid food for 8 to 12 hours before the test.

A nurse works in a bariatric clinic and cares for client with obesity who will or have undergone bariatric surgery. What is the nurse's understanding of how the procedure works?

Restricts the client's ability to eat. Hinkle, J.L., & Cheever, K.H., Brunner & Suddarth's Textbook of Medical-Surgical Nursing, 14th ed., Philadelphia, Wolters Kluwer, 2018, Chapter 48: Assessment and Management of Patients with Obesity, p. 1368.

The nurse cares for a client after a gastroscopy for which the client received sedation. The nurse should report which finding to the physician?

Resume regular diet.

In what location would the nurse palpate for the liver?

Right upper quadrant

Which of the following medications used for obesity improves cardiovascular disease risk factors in obese patients with metabolic syndrome?

Rimonabant (Acomplia)

The nurse observes the physician palpating the abdomen of a client that is suspected of having acute appendicitis. When the abdomen is pressed in the left lower quadrant the client complains of pain on the right side. What does the nurse understand this assessment technique is referred to?

Rovsing sign

Which category of laxatives draws water into the intestines by osmosis?

Saline agents (e.g., magnesium hydroxide)

A client is diagnosed with Zollinger-Ellison syndrome. The nurse knows to assess the client for which characteristic clinical feature of this syndrome?

Steatorrhea

Select the assessment finding that the nurse should immediately report, post radical neck dissection.

Stridor

A client is recently diagnosed with Crohn's disease and is beginning treatment. What first-line treatment does the nurse expect that the client will be placed on to decrease the inflammatory response?

Sulfasalazine

Which position should be used for a client undergoing a paracentesis?

Supine The client undergoing paracentesis should be placed in a supine or lateral decubitus position per the National Center for Biotechnology Information.

The nurse is preparing to examine the abdomen of a client who reports a change in bowel pattern. The nurse would place the client in which position?

Supine with knees flexed

Specific disease processes and ingestion of certain foods and medications may change the appearance of the stool. If blood is shed in sufficient quantities into the upper gastrointestinal (GI) tract, it produces which change in the stool appearance?

Tarry-black

A nurse is caring for a client who has experienced an acute exacerbation of Crohn's disease. Which statement best indicates that the disease process is under control?

The client exhibits signs of adequate GI perfusion.

A nurse cares for older adults with obesity and understands that this particular population is impacted differently than the general population. What does the nurse understand is true regarding older adults with obesity? Select all that apply.

They are at greater risk for falls. They are admitted to nursing homes more than other older adults. They have lower SpO2 levels.

A client informs the nurse that he is taking a stimulant laxative in order to be able to have a bowel movement daily. What should the nurse inform the client about taking a stimulant laxative?

They can be habit forming and will require increasing doses to be effective.

The nurse is assisting the health care provider with a colonoscopy for a client with rectal bleeding. The health care provider requests the nurse to administer glucagon during the procedure. Why is the nurse administering this medication during the procedure?

To relax colonic musculature and reduce spasm.

Which of the following is the most effective strategy to prevent hepatitis B infection?

Vaccine Hinkle, J.L., & Cheever, K.H., Brunner & Suddarth's Textbook of Medical-Surgical Nursing, 14th ed., Philadelphia, Wolters Kluwer, 2018, Chapter 49: Assessment and Management of Patients With Hepatic Disorders, Prevention, p. 1401.

A client with carcinoma of the head of the pancreas is scheduled for surgery. Which of the following should a nurse administer to the client before surgery?

Vitamin K Hinkle, J.L., & Cheever, K.H., Brunner & Suddarth's Textbook of Medical-Surgical Nursing, 14th ed., Philadelphia, Wolters Kluwer, 2018, Chapter 49: Assessment and Management of Patients With Hepatic Disorders, Medical Management, p. 1452.

Which statement correctly identifies a difference between duodenal and gastric ulcers?

Vomiting is uncommon in clients with duodenal ulcers.

A middle-aged female client is overweight and sedentary, has slightly elevated blood pressure, and is seeking ways to begin exercise. The client wishes to lose weight and improve her blood pressure. The nurse plans an exercise program in which the client

Walks for 30 minutes three times this week and is re-evaluated

Which group of clients is at high risk of developing breast cancer?

Women who are obese Hinkle, J.L., & Cheever, K.H., Brunner & Suddarth's Textbook of Medical-Surgical Nursing, 14th ed., Philadelphia, Wolters Kluwer, 2018, Chapter 48: Assessment and Management of Patients with Obesity, Table 58-3, p. 1359, 1731.

While completing an abdominal assessment, the nurse will use which landmark as the upper boundary for auscultating bowel sounds?

Xiphoid process

The primary source of microorganisms for catheter-related infections are the skin and the

catheter hub.

The term for a reddened circumscribed lesion that ulcerates and becomes crusted and is a primary lesion of syphilis is a(n)

chancre.

A client is admitted for suspected GI disease. Assessment data reveal muscle wasting, a decrease in chest and axillary hair, and increased bleeding tendency. The nurse suspects the client has:

cirrhosis. Hinkle, J.L., & Cheever, K.H., Brunner & Suddarth's Textbook of Medical-Surgical Nursing, 14th ed., Philadelphia, Wolters Kluwer, 2018, Chapter 49: Assessment and Management of Patients With Hepatic Disorders, Chart 49-10, p. 1407.

The nurse prepares a client for a barium enema. The nurse should place the client on which diet prior to the procedure?

clear liquids day before

The nurse collaborates with the physician and dietician to determine the best type of tube feeding for a client at risk for diarrhea due to hypertonic feeding solutions. Which type of feedings should the nurse suggest?

continuous feedings

What type of feedings should be administered to a client who is at risk of diarrhea due to hypertonic feeding solutions?

continuous feedings

A client with acute pancreatitis has jaundice with diminished bowel sounds and a tender distended abdomen. Additionally, lab results indicate hypovolemia. What will the physician order to treat the large amount of protein-rich fluid that has been released into the client's tissues and peritoneal cavity? Select all that apply.

diuretics albumin Chapter 49: Assessment and Management of Patients With Hepatic Disorders - Page 1442 Diuretics are given if circulating fluid is excessive. IV albumin may be given to pull fluid trapped in the peritoneum back into the circulation. Sodium would not be used to treat excessive fluid accumulation. Blood glucose levels can be elevated in clients with acute pancreatitis; therefore, glucose solutions would not be administered nor would they be used to treat excessive fluid accumulation.

Cystic fibrosis, a genetic disorder characterized by pulmonary and pancreatic dysfunction, usually appears in young children but can also affect adults. If the pancreas was functioning correctly, where would the bile and pancreatic enzymes enter the GI system?

duodenum

The most common symptom of esophageal disease is

dysphagia.

The nurse prepares to administer all of a client's medications via feeding tube. The nurse consults the pharmacist and/or physician when the nurse notes which type of oral medication on the client's medication administration record?

enteric-coated tablets

Rebound hypoglycemia is a complication of parenteral nutrition caused by

feedings stopped too abruptly.

A client reports severe pain and bleeding while having a bowel movement. Upon inspection, the health care provider notes a linear tear in the anal canal tissue. The client is diagnosed with a:

fissure.

A client is in the initial stages of oral cancer diagnosis and is frightened about the side effects of treatment and subsequent prognosis. The client has many questions regarding this type of cancer and asks where oral cancer typically occurs. What is the nurse's response?

floor of the mouth

The nurse cares for a client who receives continuous parenteral nutrition (PN) through a Hickman catheter and notices that the client's solution has run out. No PN solution is currently available from the pharmacy. What should the nurse do?

hang 10% dextrose and water

Diet therapy for clients diagnosed with irritable bowel syndrome (IBS) includes:

high-fiber diet.

The nurse assesses bowel sounds and hears one to two bowel sounds in 2 minutes. How should the nurse document the bowel sounds?

hypoactive

The nurse determines one or two bowel sounds in 2 minutes should be documented as

hypoactive.

The nurse caring for an older adult client diagnosed with diarrhea is administering and monitoring the client's medications. Because one of the client's medications is digitalis (digoxin), the nurse monitors the client closely for:

hypokalemia.

The nurse establishes a learning contract with an overweight client. The contract is best if it

includes an incremental goal of 1-2 pound weight loss this week

What is the recommended order for performing an abdominal examination?

inspection, auscultation, percussion, palpation

Which term describes a gastric secretion that combines with vitamin B12 so that it can be absorbed?

intrinsic factor

A nurse is reviewing lab results for a client with an intestinal obstruction, and infection is suspected. What would be an expected finding?

leukocytosis; elevated hematocrit; low sodium, potassium, and chloride

A client with a lengthy history of alcohol addiction is being seen for jaundice. The appearance of jaundice would most likely indicate:

liver disorder. Hinkle, J.L., & Cheever, K.H., Brunner & Suddarth's Textbook of Medical-Surgical Nursing, 14th ed., Philadelphia, Wolters Kluwer, 2018, Chapter 49: Assessment and Management of Patients With Hepatic Disorders, p. 1384.

The nurse conducts discharge education for a client who is to go home with parenteral nutrition (PN). The nurse determines the client understands the education when the client indicates a sign and/or symptom of metabolic complications is

loose, watery stools.

What is the recommended dietary treatment for a client with chronic cholecystitis?

low-fat diet Hinkle, J.L., & Cheever, K.H., Brunner & Suddarth's Textbook of Medical-Surgical Nursing, 14th ed., Philadelphia, Wolters Kluwer, 2018, Chapter 49: Assessment and Management of Patients With Hepatic Disorders, p. 1434.

A client has been diagnosed with cancer in the descending colon. Which symptoms would the nurse expect the client to report? Select all that apply.

narrowing of stools constipation

The healthcare provider of a client with oral cancer has ordered the placement of a GI tube to provide nutrition and to deliver medications. What would be the preferred route?

nasogastric intubation

A typical sign/symptom of appendicitis is:

nausea.

The nurse recognizes that the client diagnosed with a duodenal ulcer will likely experience

pain 2 to 3 hours after a meal.

A nurse is performing focused assessment on her clients. She expects to hear hypoactive bowel sounds in a client with:

paralytic ileus.

A nurse is caring for a client who is undergoing a diagnostic workup for a suspected gastrointestinal problem. The client reports gnawing epigastric pain following meals and heartburn. What would the nurse suspect this client has?

peptic ulcer disease

The nurse teaches an unlicensed caregiver about bathing clients who are receiving tube feedings. The most significant complication related to continuous tube feedings is the

potential risk for aspiration.

A client is admitted from the emergency department with complaints of severe abdominal pain and an elevated white blood cell count. The physician diagnoses appendicitis. The nurse knows the client is at greatest risk for:

rupture of the appendix.

A nurse is assessing a postoperative client for hemorrhage. What responses associated with the compensatory stage of shock should be reported to the healthcare provider?

tachycardia and tachypnea Hinkle, J.L., & Cheever, K.H., Brunner & Suddarth's Textbook of Medical-Surgical Nursing, 14th ed., Philadelphia, Wolters Kluwer, 2018, Chapter 49: Assessment and Management of Patients With Hepatic Disorders, p. 313.

The nurses assesses the client for blood in the stool due to an upper GI condition. The nurse understands that if there is blood in the stool, the stool will be which color?

tarry black

An obese client describes symptoms of palpitations, chronic fatigue, and dyspnea on exertion to the cardiologist. Upon completing the examination, the cardiologist schedules a procedure to confirm the suspected diagnosis. What diagnostic procedure would the nurse expect to be prescribed?

transesophageal echocardiography Hinkle, J.L., and Cheever, K.H. Brunner & Suddarth's Textbook of Medical-Surgical Nursing, 14th ed. Philadelphia: Lippincott Williams & Wilkins, 2018, Chapter 25: Assessment of Cardiovascular Function, p. 702.

Hickman and Groshong are examples of which type of central venous access device?

tunneled central catheters

The client describes a test previously completed to detect a small bowel obstruction prior to admission to the hospital. The client states that the test involved insertion of a tube through the nose and lasted over 6 hours. The nurse determines which test name should be documented?

upper GI enteroclysis

A client has recently been diagnosed with gastric cancer. On palpation, the nurse would note what two signs that confirm metastasis to the liver? Select all that apply.

Ascites Hepatomegaly

The nurse is preparing the client for an assessment of the abdomen. What should the nurse complete prior to this assessment?

Ask the client to empty the bladder.

A nurse applies an ostomy appliance to a client who is recovering from ileostomy surgery. Which intervention should the nurse utilize to prevent leakage from the appliance?

Ask the client to remain inactive for 5 minutes.

A client with anorexia reports constipation. Which nursing measure would be most effective in helping the client reduce constipation?

Assist client to increase dietary fiber.

The nurse is performing a focused abdominal assessment of a client with a history of bowel obstruction. The nurse has positioned the client appropriately and inspected the client's abdomen carefully. What action should the nurse perform next?

Auscultate the client's abdomen Chapter 43: Assessment of Digestive and Gastrointestinal Function - Page 1232 Abdominal auscultation is done before palpation because palpation disrupts normal bowel sounds. Percussion would have a similar disruptive effect.

A client presents to the emergency room with a possible diagnosis of appendicitis. The health care provider asks the nurse to assess for tenderness at McBurney's point. The nurse knows to palpate which area?

Between the umbilicus and the anterior superior iliac spine

A nurse cares for clients who undergo bariatric surgery due to obesity. When teaching the client on the most successful surgery in clients with very high BMI's, which procedure will the nurse mention?

Biliopancreatic diversion with duodenal switch

The nurse is performing an abdominal assessment for a patient with diarrhea and auscultates a loud rumbling sound in the left lower quadrant. What will the nurse document this sound as on the nurse's notes?

Borborygmus

Which of the following is a term used to describe intestinal rumbling?

Borborygmus

A patient is admitted to the hospital after not having had a bowel movement in several days. The nurse observes the patient is having small liquid stools, a grossly distended abdomen, and abdominal cramping. What complication can this patient develop related to this problem?

Bowel perforation

A patient with IBD would be encouraged to increase fluids, use vitamins and iron supplements, and follow a diet designed to reduce inflammation. Select the meal choice that would be recommended for a low-residue diet.

Broiled chicken with low-fiber pasta

A nurse cares for a client with obesity who is also diagnosed with depression. Which medication does the nurse expect the health care provider will prescribe, which also aids in weight loss?

Bupropion

A patient comes to the clinic with the complaint, "I think I have an ulcer." What is a characteristic associated with peptic ulcer pain that the nurse should inquire about? Select all that apply.

Burning sensation localized in the back or mid-epigastrium Feeling of emptiness that precedes meals from 1 to 3 hours Severe gnawing pain that increases in severity as the day progresses

Which term describes a reddened, circumscribed lesion that ulcerates and becomes crusted and is a primary lesion of syphilis?

Chancre

A nurse is reviewing the history and physical of a client admitted for a hemorrhoidectomy. Which predisposing condition does the nurse expect to see?

Constipation

A client suspected of having colorectal cancer requires which diagnostic study to confirm the diagnosis?

Sigmoidoscopy

When bowel sounds are heard about every 15 seconds, the nurse would record that the bowel sounds are

normal.

A nurse provides medication teaching for a client with obesity who is prescribed Lorcaserin. Which statement will the nurse include in the teaching?

"Avoid driving heavy equipment."

A nurse is caring for a client newly diagnosed with hepatitis A. Which statement by the client indicates the need for further teaching?

"How did this happen? I've been faithful my entire marriage." Hinkle, J.L., & Cheever, K.H., Brunner & Suddarth's Textbook of Medical-Surgical Nursing, 14th ed., Philadelphia, Wolters Kluwer, 2018, Chapter 49: Assessment and Management of Patients With Hepatic Disorders, Hepatitis A Virus, p. 1398.

The nurse is preparing to interview a client with cirrhosis. Based on an understanding of this disorder, which question would be most important to include?

"How often do you drink alcohol?"

After teaching a client who has had a Roux-en-Y gastric bypass, which client statement indicates the need for additional teaching?

"I need to drink 8 ounces of water before eating."

The nurse provides client education to a client about to undergo hydrogen breath testing. The nurse evaluates that the client understands the test when the client makes which statement?

"I should avoid antibiotics for 1 month before the test."

A nurse is caring for a client who has a history of sleep apnea. The client understands the disease process when he says:

"I should become involved in a weight loss program."

After teaching a client about the procedure for inserting a nontunneled central catheter, the nurse determines that the client has understood the instructions based on which statement?

"I will be lying on my back but my legs will be higher than my head."

A nurse is performing discharge teaching with a client who had a total gastrectomy. Which statement indicates the need for further teaching?

"I will have to take vitamin B12 shots up to 1 year after surgery."

A home care nurse is caring for a client with reports of epigastric discomfort who is scheduled for a barium swallow. Which statement by the client indicates an understanding of the test?

"I'll avoid eating or drinking anything 6 to 8 hours before the test."

A client is diagnosed with a hiatal hernia. Which statement indicates effective client teaching about hiatal hernia and its treatment?

"I'll eat frequent, small, bland meals that are high in fiber."

A patient has been diagnosed with acute gastritis and asks the nurse what could have caused it. What is the best response by the nurse? (Select all that apply.)

"It can be caused by ingestion of strong acids." "You may have ingested some irritating foods." "Is it possible that you are overusing aspirin."

Semi-Fowler position is maintained for at least which timeframe following completion of an intermittent tube feeding?

1 hour

The nurse is caring for a patient who has been diagnosed with gastritis. To promote fluid balance when treating gastritis, the nurse knows that what minimal daily intake of fluids is required?

1.5 L

The nurse is conducting a community education session on the prevention of oral cancers. The nurse includes which cancer as being a type of premalignant squamous cell skin cancer?

Actinic cheilitis

Blood flow to the GI tract is approximately what percentage of the total cardiac output?

20% Chapter 43: Assessment of Digestive and Gastrointestinal Function - Page 1225 Blood flow to the GI tract is about 20% of the total cardiac output and increases significantly after eating.

A nurse is reviewing the medical records of several patients and their risk for health problems. The nurse determines that the patient with which body mass index (BMI) would have the lowest risk?

23

Which client would be at greatest risk for the development of an anorectal fistula?

A 35-year-old female with Crohn's disease

What statement best describes a client with severe or extreme obesity?

A BMI >40 kg/m2

A nurse is taking health history data from a client. Use of which of the following medications would especially alert the nurse to an increased risk of hepatic dysfunction and disease in this client? Select all that apply.

Acetaminophen Ketoconazole Valproic acid Chapter 49: Assessment and Management of Patients With Hepatic Disorders - Page 1380 Many medications (including acetaminophen, ketoconazole, and valproic acid) are responsible for hepatic dysfunction and disease. A thorough medication history should address all current and past prescription medications, over-the-counter medications, herbal remedies, and dietary supplements.

The nurse is examining the mouth of a client who is HIV positive. On the inner side of the lip, the nurse sees a shallow ulcer with a yellow center and red border. The client says the area has been painful for about 5 days or so. Which condition is most consistent with these findings?

Aphthous stomatitis

A nurse is completing an assessment on a client with a postoperative neck dissection. The nurse notices excessive bleeding from the dressing site and suspects possible carotid artery rupture. What action should the nurse take first?

Apply pressure to the bleeding site

A client in the emergency department reports that a piece of meat became stuck in the throat while eating. The nurse notes the client is anxious with respirations at 30 breaths/min, frequent swallowing, and little saliva in the mouth. An esophagogastroscopy with removal of foreign body is scheduled for today. What would be the first activity performed by the nurse?

Assess lung sounds bilaterally.

A client experienced surgical resection of a tumor of the esophagus. After recovery from the anesthesia, what will the nurse include in the postoperative care plans? Select all that apply.

Assess lung sounds every 4 hours and prn. Monitor drainage in the closed chest drainage system. Verify rhythm on the cardiac monitoring system.

An older adult client is admitted to an acute care facility for treatment of an acute flare-up of a chronic gastrointestinal condition. In addition to assessing the client for complications of the current illness, the nurse monitors for age-related changes in the gastrointestinal tract. Which age-related change increases the risk of anemia?

Atrophy of the gastric mucosa

A client had a central line inserted for parenteral nutrition and is awaiting transport to the radiology department for catheter placement verification. The client reports feeling anxious and has a respiratory rate of 28 breaths/minute. What is the next action of the nurse?

Auscultate lung sounds

The nurse is creating a plan of care for a client who is not able to tolerate brushing his teeth. The nurse includes which mouth irrigation in the plan of care?

Baking soda and water

A nurse is preparing to assist a health care provider with a peripherally inserted central catheter. The nurse demonstrates understanding of this procedure by preparing which insertion site?

Basilic vein

The nurse cares for clients with obesity and understands that causes are multifactorial. What factors contribute to the development of obesity? Select all that apply.

Behavior Environment Physiology Genetics

A client has been receiving intermittent tube feedings for several days at home. The nurse notes the findings as shown in the accompanying documentation. The nurse reports the following as an adverse reaction to the tube feeding:

Fasting blood glucose level

A nurse is reviewing the health care provider orders for a client admitted with acute pancreatitis. The health care provider has ordered intravenous calcium chloride infusions for the client. What does the nurse understand is the reason for this order?

Calcium binds to fatty acids when auto digestion of the pancreas occurs. Hinkle, J.L., & Cheever, K.H., Brunner & Suddarth's Textbook of Medical-Surgical Nursing, 14th ed., Philadelphia, Wolters Kluwer, 2018, Chapter 50: Assessment and Management of Patients With Biliary Disorders, p. 1448.

A patient is scheduled for a fiberoptic colonoscopy. What does the nurse know that fiberoptic colonoscopy is most frequently used to diagnose?

Cancer

Which ulcer is associated with extensive burn injury?

Curling ulcer

A nurse is aware that both the sympathetic and parasympathetic portions of the autonomic nervous system affect GI motility. What are the actions of the sympathetic nervous system? Select all that apply.

Decreases gastric motility Causes blood vessel constriction Creates an inhibitory effect on the GI tract

A nurse cares for a client with sepsis who had bariatric surgery two weeks ago. What is the most likely source of the sepsis?

Disruption at the site of anastomosis. Hinkle, J.L., & Cheever, K.H., Brunner & Suddarth's Textbook of Medical-Surgical Nursing, 14th ed., Philadelphia, Wolters Kluwer, 2018, Chapter 48: Assessment and Management of Patients with Obesity, p. 1371.

The nurse is caring for an older adult who reports xerostomia. The nurse evaluates for use of which medication?

Diuretics

Which tube is a nasoenteric feeding tube?

Dobbhoff

The nurse is to administer a cyclic feeding through a gastric tube. It is most important for the nurse to

Elevate the head of the bed to 45 degrees. Chapter 44: Digestive and Gastrointestinal Treatment Modalities - Page 1246 All the options are things that the nurse will do when administering a cyclic tube feeding. Elevating the head of the bed to 30 to 45 degrees assists in preventing aspiration into the lungs. This is a priority according to Maslow's hierarchy of needs.

A client is seeing the physician for a suspected tumor of the liver. What laboratory study results would indicate that the client may have a primary malignant liver tumor?

Elevated alpha-fetoprotein Hinkle, J.L., & Cheever, K.H., Brunner & Suddarth's Textbook of Medical-Surgical Nursing, 14th ed., Philadelphia, Wolters Kluwer, 2018, Chapter 49: Assessment and Management of Patients With Hepatic Disorders, Assessment and Diagnostic Findings, p. 1417.

The nurse is caring for a patient who has malabsorption syndrome with an undetermined cause. What procedure will the nurse assist with that is the best diagnostic test for this illness?

Endoscopy with mucosal biopsy

A client underwent a continent ileostomy. Within which time frame should the client expect to empty the reservoir?

Every 4 to 6 hours

Which of the following is accurate regarding regional enteritis?

Exacerbations and remissions

A patient is receiving nasogastric tube feedings. The intake and output record for the past 24 hours reveals an intake of 3100 mL and an output of 2400 mL. The nurse identifies which nursing diagnosis as most likely?

Excess fluid volume

A group of students is reviewing information about the liver and associated disorders. The group demonstrates understanding of the information when they identify which of the following as a primary function of the liver?

Excrete bile Hinkle, J.L., & Cheever, K.H., Brunner & Suddarth's Textbook of Medical-Surgical Nursing, 14th ed., Philadelphia, Wolters Kluwer, 2018, Chapter 49: Assessment and Management of Patients With Hepatic Disorders, Anatomic and Physiologic Overview, p. 1378.

A nurse cares for a client who is postoperative bariatric surgery and has experienced frequent episodes of dumping syndrome. The client now reports anorexia. What is the primary reason for the client's report of anorexia?

Fear of eating

A nurse is caring for a client with mild acute pancreatitis. Which health care provider prescriptions will the nurse question as it relates to evidence based practices in the treatment of acute pancreatitis? Select all that apply.

Full liquid diet as tolerated. Nasogastric tube to intermittent wall suction for removal of gastric secretions. Initiate parenteral feedings first and advance to enteral feedings as tolerated. Hinkle, J.L., & Cheever, K.H., Brunner & Suddarth's Textbook of Medical-Surgical Nursing, 14th ed., Philadelphia, Wolters Kluwer, 2018, Chapter 50: Assessment and Management of Patients With Biliary Disorders, p. 1442.

Which of the following dietary guidelines should be followed following bariatric surgery? Select all that apply.

Include two protein snacks per day. Eat slowly. Eat three meals per day.

Which of the following dietary guidelines should be followed following bariatric surgery? Select all that apply.

Include two protein snacks per day. Eat slowly. Eat three meals per day. Hinkle, J.L., & Cheever, K.H., Brunner & Suddarth's Textbook of Medical-Surgical Nursing, 14th ed., Philadelphia, Wolters Kluwer, 2018.

When examining the abdomen of a client with reports of nausea and vomiting, what would the nurse do first?

Inspection

A client who is post op from bariatric surgery reports constipation. What is the most likely cause of the client's symptoms?

Insufficient water intake

Vomiting results in which of the following acid-base imbalances?

Metabolic alkalosis

A client diagnosed with colon cancer presents with the characteristic symptoms of a left-sided lesion. Which symptoms are indicative of this disorder? Select all that apply.

Narrowing stools Constipation Abdominal distention

A client with obesity is prescribed liraglutide for weight loss. Which common side effect will the nurse include in the client teaching?

Nausea

A nurse is caring for a client with active upper GI bleeding. What is the appropriate diet for this client during the first 24 hours after admission?

Nothing by mouth

Which of the following is considered an early symptom of gastric cancer?

Pain relieved by antacids

A client is in the hospital for the treatment of peptic ulcer disease. The client reports vomiting and a sudden severe pain in the abdomen. The nurse then assesses a board-like abdomen. What does the nurse suspect these symptoms indicate?

Perforation of the peptic ulcer

A group of students are studying for an examination on the gastrointestinal (GI) system and are reviewing the structures of the esophagus and stomach. The students demonstrate understanding of the material when they identify which of the following as the opening between the stomach and duodenum?

Pyloric sphincter Chapter 43: Assessment of Digestive and Gastrointestinal Function - Page 1225 The pyloric sphincter is the opening between the stomach and duodenum. The cardiac sphincter is the opening between the esophagus and the stomach. The hypopharyngeal sphincter or upper esophageal sphincter prevents food or fluids from re-entering the pharynx. The ileocecal valve is located at the distal end of the small intestine and regulates flow of intestinal contents into the large intestine.

When a central venous catheter dressing becomes moist or loose, what should a nurse do first?

Remove the dressing, clean the site, and apply a new dressing.

A client with obesity is interested in trying orlistat for weight loss. Which disease or condition in the client's medical history alert the nurse of potential complications if the client uses this medication?

Renal insufficiency

A nurse cares for a client who is post op open cholecystectomy and has a T-tube in place. Which clinical situation will the nurse notify the health care provider about as a possible complication of the surgery?

Significantly reduced bile output from the T-tube. Hinkle, J.L., & Cheever, K.H., Brunner & Suddarth's Textbook of Medical-Surgical Nursing, 14th ed., Philadelphia, Wolters Kluwer, 2018, Chapter 50: Assessment and Management of Patients With Biliary Disorders, p. 1435.

A nurse is inserting a nasogastric tube for feeding a client. Place in order the steps from 1 to 6 for correctly inserting the tube.

Sit the client in an upright position Apply gloves to the nurse's hands Measure the length of the tube that will be inserted Apply water-soluble lubricant to the tip of the tube Tilt the client's nose upward Instruct the client to lower the head and swallow

An older adult patient who has been living at home alone is diagnosed with parotitis. What causative bacteria does the nurse suspect is the cause of the parotitis?

Staphylococcus aureus

A client with acute pancreatitis reports muscle cramping in the lower extremities. What pathophysiology concept represents the reason the client is reporting this?

Tetany related to hypocalcemia Hinkle, J.L., & Cheever, K.H., Brunner & Suddarth's Textbook of Medical-Surgical Nursing, 14th ed., Philadelphia, Wolters Kluwer, 2018, Chapter 50: Assessment and Management of Patients With Biliary Disorders, p. 1448.

A healthcare provider prescribes a combination of three drugs to treat reoccurring peptic ulcer disease, and the client asks the nurse the reason for all the medications. What teaching should the nurse review with the client?

The bismuth salts, antibiotics, and proton pump inhibitors will work together to suppress or eradicate H. pylori.

A client is being seen in the clinic for reports of painful hemorrhoids. The nurse assesses the client and observes the hemorrhoids are prolapsed but able to be placed back in the rectum manually. The nurse documents the hemorrhoids as what degree?

Third degree

Which nursing instruction is correct to provide the client following a barium enema?

The stools may be a white or clay colored.

The nurse is inserting a sump tube in a patient with Crohn's disease who is suspected of having a bowel obstruction. What does the nurse understand is the benefit of the gastric (Salem) sump tube in comparison to some of the other tubes?

The tube is radiopaque.

A nurse examines the socioeconomic impact of obesity among Americans. Which statements does the nurse understand is true? Select all that apply.

Those with less education are impacted at a greater prevalence of disease. Those with less income are impacted at a greater prevalence of disease.

A positive nitrogen balance indicates which condition?

Tissue growth Chapter 44: Digestive and Gastrointestinal Treatment Modalities - Page 1256 A positive nitrogen balance exists when nitrogen intake exceeds nitrogen output and indicates tissue growth. A negative nitrogen balance exists with fever, starvation, and burn injury.

Which symptom characterizes regional enteritis?

Transmural thickening

When describing the role of the pancreas to a client with a pancreatic dysfunction, the nurse would identify which substance as being acted on by pancreatic lipase?

Triglycerides Chapter 43: Assessment of Digestive and Gastrointestinal Function - Page 1226 Pancreatic lipase acts on lipids, especially triglycerides. Salivary amylase and pancreatic amylase act on starch. Pepsin and hydrochloric acid in the stomach and trypsin from the pancreas act on proteins. Insulin acts on glucose.

The nurse is caring for a patient with ascites due to cirrhosis of the liver. What position does the nurse understand will activate the renin-angiotensin aldosterone and sympathetic nervous system and decrease responsiveness to diuretic therapy?

Upright Hinkle, J.L., & Cheever, K.H., Brunner & Suddarth's Textbook of Medical-Surgical Nursing, 14th ed., Philadelphia, Wolters Kluwer, 2018, Chapter 49: Assessment and Management of Patients With Hepatic Disorders, Bed Rest, p. 1387.

A client with severe and chronic liver disease is showing manifestations related to inadequate vitamin intake and metabolism. He reports difficulty driving at night because he cannot see well. Which of the following vitamins is most likely deficient for this client?

Vitamin A Hinkle, J.L., & Cheever, K.H., Brunner & Suddarth's Textbook of Medical-Surgical Nursing, 14th ed., Philadelphia, Wolters Kluwer, 2018, Chapter 49: Assessment and Management of Patients With Hepatic Disorders, Vitamin Deficiency, p. 1397.

The nurse is caring for a client with chronic gastritis. The nurse monitors the client knowing that this client is at risk for which vitamin deficiency?

Vitamin B12

Which condition indicates an overdose of lactulose?

Watery diarrhea Hinkle, J.L., & Cheever, K.H., Brunner & Suddarth's Textbook of Medical-Surgical Nursing, 14th ed., Philadelphia, Wolters Kluwer, 2018, Chapter 49: Assessment and Management of Patients With Hepatic Disorders, Quality and Safety Nursing Alert, p. 1395.

A client is admitted to the emergency department with reports right lower quadrant pain. Blood specimens are drawn and sent to the laboratory. Which laboratory finding should be reported to the health care provider immediately?

White blood cell (WBC) count 22.8/mm3

Which neuroregulator increase gastric acid secretion?

acetylcholine

Which venous access device can be used for less than 6 weeks in clients requiring parenteral nutrition?

nontunneled catheters

A nurse is teaching a group of middle-aged men about peptic ulcers. When discussing risk factors for peptic ulcers, the nurse should mention:

alcohol abuse and smoking.

Total parental nutrition (TPN) should be used cautiously in clients with pancreatitis because they

cannot tolerate a high glucose concentration. Hinkle, J.L., and Cheever, K.H. Brunner & Suddarth's Textbook of Medical-Surgical Nursing, 14th ed. Philadelphia: Lippincott Williams & Wilkins, 2017, Chapter 50: Pancreatitis: Medical Management

Total parental nutrition (TPN) should be used cautiously in clients with pancreatitis because such clients:

cannot tolerate high-glucose concentration. Hinkle, J.L., & Cheever, K.H., Brunner & Suddarth's Textbook of Medical-Surgical Nursing, 14th ed., Philadelphia, Wolters Kluwer, 2018, Chapter 49: Assessment and Management of Patients With Hepatic Disorders, p. 1444.

To ensure patency of central venous line ports, diluted heparin flushes are used

daily when not in use.

A client receives a local anesthetic to suppress the gag reflex for a diagnostic procedure of the upper GI tract. The nurse determines which nursing intervention is advised for this client?

do not give any food and fluids until the gag reflex returns

A nurse is caring for a client who underwent a subtotal gastrectomy. To manage dumping syndrome, the nurse should advise the client to:

drink liquids only between meals.

When gastric analysis testing reveals excess secretion of gastric acid, the nurse recognizes which medical diagnoses is supported?

duodenal ulcer

Which response is a parasympathetic response in the GI tract?

increased peristalsis Chapter 43: Assessment of Digestive and Gastrointestinal Function - Page 1226 Increased peristalsis is a parasympathetic response in the GI tract. Decreased gastric secretion, blood vessel constriction, and decreased motility are sympathetic responses in the GI tract.

The nurse prepares to administer the lavage solution to a client having a colonoscopy completed. The nurse stops and notifies the physician when noting that the client has which condition?

inflammatory bowel disease

The nurse performs an abdominal assessment. The nurse should perform the assessment in which order?

inspection, auscultation, percussion, palpation

A client has symptoms suggestive of peritonitis. Nursing management would not include:

limiting analgesics to avoid the formation of paralytic ileus.

The nurse provides health teaching to inform the client with oral cancer that

many oral cancers produce no symptoms in the early stages.

The most significant complication related to continuous tube feedings is

the increased potential for aspiration.

A client has received a diagnosis of oral cancer. During client education, the client expresses dismay at not having recognized any early signs or symptoms of the disease. The nurse tells the client that in early stages of this disease:

there are usually no symptoms.

Which enzyme aids in the digestion of protein?

trypsin

The nurse is assessing a client for constipation. To identify the cause of constipation, the nurse should begin by reviewing the client's:

usual pattern of elimination.

A client is admitted to the hospital with an exacerbation of chronic gastritis. When assessing the client's nutritional status, the nurse should expect to find what type of deficiency?

vitamin B12

A client with acute gastritis asks the nurse what might have caused the problem. What is a possible cause of acute gastritis?

Dietary indiscretion Excessive alcohol intake Radiation therapy

A client is diagnosed with dumping syndrome after bariatric surgery. Which findings on the nursing assessment correlate with this diagnosis? Select all that apply.

Dizziness Sweating Tachycardia

A client receives a local anesthetic to suppress the gag reflex for a diagnostic procedure of the upper GI tract. Which nursing intervention is advised for this client?

Do not give any food and fluids until the gag reflex returns.

A nurse cares for a client with obesity who is taking phentermine/topiramate ER and reports tingling around the mouth and fingertips. What is the priority nursing action?

Document the finding. Chapter 48: Assessment and Management of Patients with Obesity - Page 1365 A common side effect of phentermine/topiramate ER is tingling around the mouth and fingertips. This is not a life threatening side effect and does not need to be immediately reported to the health care provider and the client's vital signs and laboratory results do not need to obtained right away.

A nurse teaches a client who will undergo bariatric surgery the importance of avoiding problematic foods post surgery. Which foods will the nurse suggest the client avoid? Select all that apply.

Doughy breads Overcooked meats Pasta Rice Hinkle, J.L., & Cheever, K.H., Brunner & Suddarth's Textbook of Medical-Surgical Nursing, 14th ed., Philadelphia, Wolters Kluwer, 2018, Chapter 48: Assessment and Management of Patients with Obesity, p. 1371.

The nurse is conducting discharge teaching for a client with diverticulosis. Which instruction should the nurse include in the teaching?

Drink 8 to 10 glasses of fluid daily.

A nurse researcher investigates why clients with obesity who have lost weight often regain the weight. The nurse looks at the relationship of leptin and its role in weight gain. What does the nurse determine is true regarding leptin and weight gain? Select all that apply.

Drops in leptin level increases hunger. Leptin signals satiety in the hypothalamus. Hinkle, J.L., & Cheever, K.H., Brunner & Suddarth's Textbook of Medical-Surgical Nursing, 14th ed., Philadelphia, Wolters Kluwer, 2018, Chapter 48: Assessment and Management of Patients with Obesity, p. 1360.

Clients with Type O blood are at higher risk for which of the following GI disorders?

Duodenal ulcers

Which term refers to the first portion of the small intestine?

Duodenum

Increased appetite and thirst may indicate that a client with chronic pancreatitis has developed diabetes mellitus. Which of the following explains the cause of this secondary diabetes?

Dysfunction of the pancreatic islet cells Hinkle, J.L., & Cheever, K.H., Brunner & Suddarth's Textbook of Medical-Surgical Nursing, 14th ed., Philadelphia, Wolters Kluwer, 2018, Chapter 50: Assessment and Management of Patients With Biliary Disorders, Assessment and Diagnostic Findings, p. 1445.

The nurse is obtaining a history on a patient who comes to the clinic. What symptom described by the patient is one of the first symptoms associated with esophageal disease?

Dysphagia

A patient is diagnosed with mild acute pancreatitis. What does the nurse understand is characteristic of this disorder?

Edema and inflammation Hinkle, J.L., & Cheever, K.H., Brunner & Suddarth's Textbook of Medical-Surgical Nursing, 14th ed., Philadelphia, Wolters Kluwer, 2018, Chapter 50: Assessment and Management of Patients With Biliary Disorders, Clinical Manifestations, pp. 1441-1442.

A client is receiving vasopressin for the urgent management of active bleeding due to esophageal varices. What most serious complication should the nurse assess the client for after the administration?

Electrocardiogram changes Hinkle, J.L., & Cheever, K.H., Brunner & Suddarth's Textbook of Medical-Surgical Nursing, 14th ed., Philadelphia, Wolters Kluwer, 2018, Chapter 49: Assessment and Management of Patients With Hepatic Disorders, Pharmacologic Therapy, p. 1390.

A nurse is preparing a client with Crohn's disease for a barium enema. What should the nurse do the day before the test?

Encourage plenty of fluids.

A health care provider suspects that a client has peptic ulcer disease. With which diagnostic procedure would the nurse most likely prepare to assist?

Endoscopy

A client has a cheesy white plaque in the mouth. The plaque looks like milk curds and can be rubbed off. What is the nurse's best intervention?

Instruct the client to swish prescribed nystatin solution for 1 minute.

A client is scheduled to receive a 25% dextrose solution of parenteral nutrition. What actions are a priority for the nurse to perform prior to administration? Select all that apply.

Ensure availability of an infusion pump Ensure completion of baseline monitoring of the complete blood count (CBC) and chemistry panel Place a 1.5-micron filter on the tubing

A surgeon is discussing surgery with a client diagnosed with colon cancer. The client is visibly shaken over the possibility of a colostomy. Based on the client's response, the surgeon should collaborate with which health team member?

Enterostomal nurse

A client has a new order for metoclopramide. What potential side effects should the nurse educate the client about?

Extrapyramidal

The nurse is performing a community screening for colorectal cancer. Which characteristic should the nurse include in the screening?

Familial polyposis

Which characteristic is a risk factor for colorectal cancer?

Familial polyposis

Which medication is classified as a histamine-2 receptor antagonist?

Famotidine

A client is recovering from gastric surgery. What is the correct position for the nurse to place this client?

Fowler's

A client has a new order for metoclopramide. The nurse identifies that this medication can be safely administered for which condition?

Gastroesophageal reflux disease

A patient describes a burning sensation in the esophagus, pain when swallowing, and frequent indigestion. What does the nurse suspect that these clinical manifestations indicate?

Gastroesophageal reflux disease

Gastrostomy feedings are preferred to nasogastric feedings in the comatose patient, because the:

Gastroesophageal sphincter is intact, lessening the possibility of regurgitation and aspiration.

The nurse is caring for a comatose patient and administering gastrostomy feedings. What does the nurse understand is the reason that gastrostomy feedings are preferred to nasogastric (NG) feedings in the comatose patient?

Gastroesophageal sphincter is intact, lessening the possibility of regurgitation.

A patient is experiencing painful, inflamed, and swollen gums, and when brushing the teeth, the gums bleed. What common disease of the oral tissue does the nurse understand these symptoms indicate?

Gingivitis

The following appears on the medical record of a male patient receiving parenteral nutrition: WBC: 6500/cu mm Potassium 4.3 mEq/L Magnesium 2.0 mg/dL Calcium 8.8 mg/dL Glucose 190 mg/dL Which finding would alert the nurse to a problem?

Glucose level

A client with suspected biliary obstruction due to gallstones reports changes to the color of his stools. Which stool color does the nurse recognize as common to biliary obstruction?

Gray Hinkle, J.L., & Cheever, K.H., Brunner & Suddarth's Textbook of Medical-Surgical Nursing, 14th ed., Philadelphia, Wolters Kluwer, 2018, Chapter 50: Assessment and Management of Patients With Biliary Disorders, Changes in Urine and Stool Color, p. 1432.

A client with human immunodeficiency virus (HIV) comes to the clinic and is experiencing white patches on the lateral border of the tongue. What type of lesions does the nurse document?

Hairy leukoplakia

The nurse is assessing a client who is stating gastrointestinal upset and a feeling of bloating. Which type of meal would the nurse anticipate causing these types of symptoms?

Hamburger and French fries

A patient is receiving parenteral nutrition. The current solution is nearing completion, and a new solution is to be hung, but it has not arrived from the pharmacy. Which action by the nurse would be most appropriate?

Hang a solution of dextrose 10% and water until the new solution is available. Chapter 44: Digestive and Gastrointestinal Treatment Modalities - Page 1260 The infusion rate of the solution should not be increased or decreased; if the solution is to run out, a solution of 10% dextrose and water is used until the next solution is available. Having someone go to the pharmacy would be appropriate, but there is no way to determine if the person will arrive back before the solution runs out. Starting another infusion would be inappropriate. Additionally, the infusion needs to be maintained through the central venous access device to maintain patency.

The client with osteoarthritis is seen in the clinic. Which assessment finding indicates the client is having difficulty implementing self-care?

Has a weight gain of 5 pounds

The nurse is assessing a client with an ulcer for signs and symptoms of hemorrhage. The nurse interprets which condition as a sign/symptom of possible hemorrhage?

Hematemesis

The nurse is cautiously assessing a client admitted with peptic ulcer disease because the most common complication that occurs in 10% to 20% of clients is:

Hemorrhage

Which of the following is the most common complication associated with peptic ulcer?

Hemorrhage

The nurse is caring for a client who has just returned from the PACU after surgery for peptic ulcer disease. For what potential complications does the nurse know to monitor? Select all that apply.

Hemorrhage Perforation Penetration Pyloric obstruction

A nurse is caring for a patient with a Salem sump gastric tube attached to low intermittent suction for decompression. The patient asks, "What's this blue part of the tube for?" Which response by the nurse would be most appropriate?

"It is a vent that prevents backflow of the secretions."

A nurse is providing dietary instructions to a client with a history of pancreatitis. Which instruction is correct?

"Maintain a high-carbohydrate, low-fat diet." Hinkle, J.L., & Cheever, K.H., Brunner & Suddarth's Textbook of Medical-Surgical Nursing, 14th ed., Philadelphia, Wolters Kluwer, 2018, Chapter 50: Assessment and Management of Patients With Biliary Disorders, Improving Nutritional Status, p. 1444.

A client with obesity will undergo intragastric balloon therapy for weight loss. What teaching will the nurse include when educating the client about this procedure? Select all that apply.

"The balloon will remain in place for 3 to 6 months." "Nausea and vomiting are the most commonly reported side effects." Hinkle, J.L., & Cheever, K.H., Brunner & Suddarth's Textbook of Medical-Surgical Nursing, 14th ed., Philadelphia, Wolters Kluwer, 2018, Chapter 48: Assessment and Management of Patients with Obesity, p. 1366.

A client with obesity is prescribed orlistat for weight loss. The client asks the nurse, "I understand the medication prevents digestion of fat, but what happens if I eat fat?" What is the nurse's best response?

"The fat is passed in your stools."

The nurse is assisting a client to drain his continent ileostomy (Kock pouch). The nurse should insert the catheter how far through the nipple/valve?

2 in.

The nurse is caring for a client recovering from an esophagogastroduodenoscopy (EGD). Which of the following client symptoms would require further nursing assessment?

Abdominal distention

A client is diagnosed with colon cancer, located in the lower third of the rectum. What does the nurse understand will be the surgical treatment option for this client?

Abdominoperineal resection

What is the most common cause of small-bowel obstruction?

Adhesions

A client's health history is suggestive of inflammatory bowel disease. Which of the following would suggest Crohn's diseterm-158ase, rather than ulcerative colitis, as the cause of the client's signs and symptoms?

An absence of blood in stool

The nurse is caring for a patient with cirrhosis of the liver and observes that the patient is having hand-flapping tremors. What does the nurse document this finding as?

Asterixis Hinkle, J.L., & Cheever, K.H., Brunner & Suddarth's Textbook of Medical-Surgical Nursing, 14th ed., Philadelphia, Wolters Kluwer, 2018, Chapter 49: Assessment and Management of Patients With Hepatic Disorders, Physical Assessment, p. 1380.

Which drug is considered a stimulant laxative?

Bisacodyl

Which of the following would a nurse expect to assess in a client with peritonitis?

Board-like abdomen

Which of the following appears to be a significant factor in the development of gastric cancer?

Diet

A client is being treated for diverticulosis. Which points should the nurse include in this client's teaching plan? Select all that apply.

Do not suppress the urge to defecate. Drink at least 8 to 10 large glasses of fluid every day. Use bulk-forming laxatives Encourage an individualized exercise program

The nurse is performing an assessment of a patient. During the assessment the patient informs the nurse of some recent "stomach trouble." What does the nurse know is the most common symptom of patients with GI dysfunction?

Dyspepsia

The nurse is assessing the skin graft site of a client who has undergone a radical neck dissection. The skin graft site is pink. The nurse documents which result?

Healthy graft

Which diagnostic test would be used first to evaluate a client with upper GI bleeding?

Hemoglobin and hematocrit

A client has developed drug-induced hepatitis from a drug reaction to antidepressants. What treatment does the nurse anticipate the client will receive to treat the reaction?

High-dose corticosteroids Hinkle, J.L., & Cheever, K.H., Brunner & Suddarth's Textbook of Medical-Surgical Nursing, 14th ed., Philadelphia, Wolters Kluwer, 2018, Chapter 49: Assessment and Management of Patients With Hepatic Disorders, Drug-Induced Hepatitis, p. 1405.

When reviewing the history of a client with pancreatic cancer, the nurse would identify which of the following as a possible risk factor?

History of pancreatitis Hinkle, J.L., & Cheever, K.H., Brunner & Suddarth's Textbook of Medical-Surgical Nursing, 14th ed., Philadelphia, Wolters Kluwer, 2018, Chapter 50: Assessment and Management of Patients With Biliary Disorders, Clinical Manifestations, p. 1450.

Upon receiving the dinner tray for a client admitted with acute gallbladder inflammation, the nurse will question which of the following foods on the tray?

Hot roast beef sandwich with gravy Hinkle, J.L., & Cheever, K.H., Brunner & Suddarth's Textbook of Medical-Surgical Nursing, 14th ed., Philadelphia, Wolters Kluwer, 2018, Chapter 50: Assessment and Management of Patients With Biliary Disorders, Nutritional and Supportive Therapy, p. 1434.

Review the following four examples of ideal body weight (IBW), actual weight, and body mass index (BMI). Using three criteria for each example, select the body weight that indicates morbid obesity.

IBW = 145 lbs; weight = 290 lbs; BMI = 31 kg/m2

A nurse is preparing a presentation for a local community group about hepatitis. Which of the following would the nurse include?

Hepatitis C increases a person's risk for liver cancer. Chapter 49: Assessment and Management of Patients With Hepatic Disorders - Page 1417 Infection with hepatitis C increases the risk of a person developing hepatic (liver) cancer. Hepatitis A is transmitted primarily by the oral-fecal route; hepatitis B is frequently spread by sexual contact and infected blood. Hepatitis E is similar to hepatitis A whereas hepatitis G is similar to hepatitis C.

Which term refers to a protrusion of the intestine through a weakened area in the abdominal wall?

Hernia

The nurse is caring for a patient who has had an appendectomy. What is the best position for the nurse to maintain the patient in after the surgery?

High Fowler's

A patient diagnosed with IBS is advised to eat a diet that is:

High in fiber.

Which is the most prominent signs of inflammatory bowel disease?

Intermittent pain and diarrhea

A patient visited a nurse practitioner because he had diarrhea for 2 weeks. He described his stool as large and greasy. The nurse knows that this description is consistent with a diagnosis of:

Intestinal malabsorption.

The nurse is planning care for a client with painful oral lesions. Which food should be included in the client's diet?

Jell-O

Clinical manifestations of common bile duct obstruction include all of the following except:

Light-colored urine Hinkle, J.L., & Cheever, K.H., Brunner & Suddarth's Textbook of Medical-Surgical Nursing, 14th ed., Philadelphia, Wolters Kluwer, 2018, Chapter 50: Assessment and Management of Patients With Biliary Disorders, Clinical Manifestations, p. 1452.

A client is being prepared to undergo laboratory and diagnostic testing to confirm the diagnosis of cirrhosis. Which test would the nurse expect to be used to provide definitive confirmation of the disorder?

Liver biopsy Chapter 49: Assessment and Management of Patients With Hepatic Disorders - Page 1408 A liver biopsy which reveals hepatic fibrosis is the most conclusive diagnostic procedure. Coagulation studies provide information about liver function but do not definitively confirm the diagnosis of cirrhosis. Magnetic resonance imaging and radioisotope liver scan help to support the diagnosis but do not confirm it. These tests provide information about the liver's enlarged size, nodular configuration, and distorted blood flow.

After teaching a group of students about irritable bowel syndrome and antidiarrheal agents, the instructor determines that the teaching was effective when the students identify which of the following as an example of an opiate-related antidiarrheal agent?

Loperamide

A nurse is planning care for a client who will be arriving to the unit postoperatively from bariatric surgery. In an effort to decrease the risk of venous thromboembolism (VTE), which health care provider orders does the nurse anticipate?

Mechanical compression and prophylactic anticoagulation

The nurse cares for a client who receives continuous enteral tube feedings and who is at low risk for aspiration. The nurse assesses the gastric residual volume to be 350 mL. The nurse determines which action is correct?

Monitoring the feeding closely.

Which is a true statement regarding gastric cancer?

Most clients are asymptomatic during the early stage of the disease.

A client undergoes total gastrectomy. Several hours after surgery, the nurse notes that the client's nasogastric (NG) tube has stopped draining. How should the nurse respond?

Notify the health care provider.

Peptic ulcer disease occurs more frequently in people with which blood type?

O

Which medication classification represents a proton (gastric acid) pump inhibitor?

Omeprazole

A health care provider recommends behavior interventions for a client with obesity. What does the nurse understand is most effective behavioral intervention for clients with obesity?

High intensity Hinkle, J.L., & Cheever, K.H., Brunner & Suddarth's Textbook of Medical-Surgical Nursing, 14th ed., Philadelphia, Wolters Kluwer, 2018, Chapter 48: Assessment and Management of Patients with Obesity, p. 1362.

A client with a diagnosis of acute appendicitis is awaiting surgical intervention. The nurse listens to bowel sounds and hears none and observes that the abdomen is rigid and board-like. What complication does the nurse determine may be occurring at this time?

Peritonitis

When caring for a client with acute pancreatitis, the nurse should use which comfort measure?

Positioning the client on the side with the knees flexed Hinkle, J.L., & Cheever, K.H., Brunner & Suddarth's Textbook of Medical-Surgical Nursing, 14th ed., Philadelphia, Wolters Kluwer, 2018, Chapter 50: Assessment and Management of Patients With Biliary Disorders, Chart 50-4, p. 1447.

It is important for the nurse to monitor serum electrolytes in a patient with acute diarrhea. Select the electrolyte result that should be immediately reported.

Potassium of 2.8 mEq/L

The patient admitted with acute pancreatitis has passed the acute stage and is now able to tolerate solid foods. What type of diet will increase caloric intake without stimulating pancreatic enzymes beyond the ability of the pancreas to respond?

High-carbohydrate, low-protein, low-fat diet Hinkle, J.L., & Cheever, K.H., Brunner & Suddarth's Textbook of Medical-Surgical Nursing, 14th ed., Philadelphia, Wolters Kluwer, 2018, Chapter 50: Assessment and Management of Patients With Biliary Disorders, Chart 50-4, p. 1447.

A client who is postoperative open RYGB bariatric surgery is scheduled for discharge and will have a Jackson-Pratt drain to care for while at home. Which teaching will the nurse include specific to this? Select all that apply.

How to empty the drain Recording drainage amount When to contact the health care provider How to measure the drainage amount

A client is admitted with a diagnosis of acute appendicitis. When assessing the abdomen, the nurse would expect to find rebound tenderness at which location?

Right lower quadrant

The nurse is preparing a care plan for a client with hepatic cirrhosis. Which nursing diagnoses are appropriate? Select all that apply.

Risk for injury related to altered clotting mechanisms Activity intolerance related to fatigue, general debility, muscle wasting, and discomfort Disturbed body image related to changes in appearance, sexual dysfunction, and role function Chapter 49: Assessment and Management of Patients With Hepatic Disorders - Page 1409 Risks for injury, activity intolerance, and disturbed body image are priority nursing diagnoses. The appropriate nursing diagnosis related to nutrition would be altered nutrition, less than body requirements, related to chronic gastritis, decreased gastrointestinal motility, and anorexia. Urinary incontinence is not generally a concern with hepatic cirrhosis.

A client recovers from an episode of gallbladder inflammation and the client's diet is advanced from a low-fat liquid diet. Which meals will the nurse recommend to the client? Select all that apply.

Roasted chicken, mashed potatoes, and green beans. Marinated lean steak, steamed rice, and roasted zucchini. Oatmeal with a cooked fruit compote and hot tea. Hinkle, J.L., & Cheever, K.H., Brunner & Suddarth's Textbook of Medical-Surgical Nursing, 14th ed., Philadelphia, Wolters Kluwer, 2018, Chapter 50: Assessment and Management of Patients With Biliary Disorders, Nutritional and Supportive Therapy, p. 1434.

Which of the following are characteristics associated with the Zollinger-Ellison syndrome (ZES)? Select all that apply.

Severe peptic ulcers Extreme gastric hyperacidity Gastrin-secreting tumors of the pancreas

An older client is diagnosed with parotitis. What bacterial infection does the nurse suspect caused the client's parotitis?

Staphylococcus aureus

A client has had a gastrostomy tube inserted. What does the nurse anticipate the initial fluid nourishment will be after the insertion of the gastrostomy tube?

Sterile water at 30 mL/h

Place the pathophysiological steps in correct order for the development of dumping syndrome.

Hypertonic food bolus from the stomach to the small intestine Release of metabolic peptides Tachycardia, dizziness, sweating, nausea, vomiting, bloating, abdominal cramping, and diarrhea Rapid rise of glucose, release of insulin, reactive hypoglycemia Hinkle, J.L., & Cheever, K.H., Brunner & Suddarth's Textbook of Medical-Surgical Nursing, 14th ed., Philadelphia, Wolters Kluwer, 2018, Chapter 48: Assessment and Management of Patients with Obesity, p. 1373.

A nurse is caring for a client who needs a nasogastric (NG) tube for a tube feeding. What is the safe method for the nurse to use to measure the appropriate length of the NG tube?

The distance measured from the tip of the nose to the earlobe and from the earlobe to the xiphoid process

An elderly client diagnosed with diarrhea is taking digoxin. Which electrolyte imbalance should the nurse be alert to?

Hypokalemia

A client presents to the emergency department with complaints of acute GI distress, bloody diarrhea, weight loss, and fever. Which condition in the family history is most pertinent to the client's current health problem?

Ulcerative colitis

A patient is admitted to the hospital with possible cholelithiasis. What diagnostic test of choice will the nurse prepare the patient for?

Ultrasonography Hinkle, J.L., & Cheever, K.H., Brunner & Suddarth's Textbook of Medical-Surgical Nursing, 14th ed., Philadelphia, Wolters Kluwer, 2018, Chapter 50: Assessment and Management of Patients With Biliary Disorders, Ultrasonography, p. 1432.

Which of the following is an age-related change in the esophagus?

Weakened gag reflex

A graduate nurse is cleaning a central venous access device (CVAD) and is being evaluated by the preceptor nurse. The preceptor nurse makes a recommendation for relearning the skill when she notes the graduate nurse does the following action:

Wipes catheter ports from distal end to insertion site

A nurse is caring for a client with obesity and diabetes after abdominal surgery. What is the client at increased risk for?

Wound dehiscence

The nurse is creating a discharge plan of care for a client with a peptic ulcer. The nurse tells the client to avoid

decaffeinated coffee.

The nurse cares for a client who receives parenteral nutrition (PN). The nurse notes on the care plan that the catheter will need to be removed 6 weeks after insertion and that the client's venous access device is a

nontunneled central catheter.

A client reports diarrhea after having bariatric surgery. What nonpharmacologic treatment can the nurse suggest to decrease the incidence of diarrhea?

Increase the fiber content in the diet.

A client with a peptic ulcer is about to begin a therapeutic regimen that includes a bland diet, antacids, and famotidine. Before the client is discharged, the nurse should provide which instruction?

"Avoid aspirin and products that contain aspirin."

Alcohol, which is toxic to the liver, is a common cause of hepatic disorders. As part of health teaching, the nurse advises a group of women that the amount of daily alcohol use should generally be limited to the equivalent of:

1 drink Hinkle, J.L., & Cheever, K.H., Brunner & Suddarth's Textbook of Medical-Surgical Nursing, 14th ed., Philadelphia, Wolters Kluwer, 2018, Chapter 49: Assessment and Management of Patients With Hepatic Disorders, Medical Management, p. 1408.

A client with peptic ulcer disease must begin triple medication therapy. For how long will the client follow this regimen?

10 to 14 days

A nurse is preparing to administer a 500 mL bolus tube feeding to a patient. The nurse anticipates administering this feeding over which time frame?

10 to 15 minutes Chapter 44: Digestive and Gastrointestinal Treatment Modalities - Page 1249 Typically a bolus tube feeding of 300 to 500 mL requires about 10 to 15 minutes to complete.

The nurse administers a tube feeding to a client via the intermittent gravity drip method. The nurse should administer the feeding over at least which period of time?

30 minutes Chapter 44: Digestive and Gastrointestinal Treatment Modalities - Page 1249 Tube feedings administered via intermittent gravity drip should be administered over 30 minutes or longer.

A client weighs 215 lbs and is 5' 8" tall. The nurse calculate this client's body mass index (BMI) as what?

32.7

Calculate the BMI of a client who is 180 pounds and is 5 feet 2 inches tall. Round to one decimal point.

32.9

Ursodeoxycholic acid (UDCA) has been used to dissolve small, radiolucent gallstones. Which duration of therapy is required to dissolve the stones?

6 to 12 months Hinkle, J.L., & Cheever, K.H., Brunner & Suddarth's Textbook of Medical-Surgical Nursing, 14th ed., Philadelphia, Wolters Kluwer, 2018, Chapter 50: Assessment and Management of Patients With Biliary Disorders, p. 1434.

A client comes to the clinic after developing a headache, abdominal pain, nausea, hiccupping, and fatigue about 2 hours ago. The client tells the nurse that the last food was buffalo chicken wings and beer. Which medical condition does the nurse find to be most consistent with the client's presenting problems?

Acute gastritis

A nurse plans care for a client who is post op bariatric surgery. Which nursing diagnosis will be the priority?

Acute pain related to surgical procedure Hinkle, J.L., & Cheever, K.H., Brunner & Suddarth's Textbook of Medical-Surgical Nursing, 14th ed., Philadelphia, Wolters Kluwer, 2018, Chapter 48: Assessment and Management of Patients with Obesity, p. 1371.

Which of the following would the nurse expect to assess in a client with hepatic encephalopathy?

Asterixis Hinkle, J.L., & Cheever, K.H., Brunner & Suddarth's Textbook of Medical-Surgical Nursing, 14th ed., Philadelphia, Wolters Kluwer, 2018, Chapter 49: Assessment and Management of Patients With Hepatic Disorders, Physical Assessment, p. 1380.

The nurse teaches the client with gastroesophageal reflux disease (GERD) which measure to manage the disease?

Avoid eating or drinking 2 hours before bedtime

The nurse is evaluating a client's ulcer symptoms to differentiate ulcer as duodenal or gastric. Which symptom should the nurse at attribute to a duodenal ulcer?

Awakening in pain

Which is the most common presenting symptom of colon cancer?

Change in bowel habits

A client comes into the emergency department with reports of abdominal pain. What should the nurse ask first?

Characteristics and duration of pain

The nurse is caring for a group of clients. Which client(s) would be a candidate for total parenteral nutrition (TPN)? Select all that apply.

Child with short bowel syndrome Middle-aged man with acute pancreatitis Man with two-thirds of his colon removed

If a client's central venous catheter accidentally becomes disconnected, what should a nurse do first?

Clamp the catheter.

A nurse cares for a client who is 5 feet 11 inches tall and weighs 225 pounds. What statement describes the client's BMI?

Class I obesity

The nurse is working with a client who has difficulty controlling her blood sugar. The overweight client does not adhere to a low-calorie diet and forgets to take medications and check her blood glucose level. The client's glycohemoglobin is 8.5%. When establishing a goal for the client, the nurse first

Collaborates with the client to establish an agreed-upon goal Hinkle, J. L., Cheever, K. H. Brunner & Suddarth's Textbook of Medical-Surgical Nursing , 14th ed Philadelphia: Wolters Kluwer Health/Lippincott Williams & Wilkins, 2018, Chapter 9: Chronic Illness and Disability, p. 147.

Which of the following is the diagnostic of choice if the suspected diagnosis is diverticulitis?

Computed tomography scan

A nurse cares for a client who is post op open cholecystectomy. Upon assessment, the nurse notes the client's abdomen feels firm to palpation. What is the nurse's priority action?

Contact the health care provider Hinkle, J.L., & Cheever, K.H., Brunner & Suddarth's Textbook of Medical-Surgical Nursing, 14th ed., Philadelphia, Wolters Kluwer, 2018, Chapter 50: Assessment and Management of Patients With Biliary Disorders, p. 1439.

A nurse is preparing to administer a client's scheduled parenteral nutrition (PN). Upon inspecting the bag, the nurse notices that small amounts of white precipitate are present in the bag. What is the nurse's best action?

Contact the pharmacy to obtain a new bag of PN.

A morbidly obese client asks the nurse if medications are available to assist with weight loss. The nurse knows that the client would not be a candidate for phentermine if the following is part of the client's health history:

Coronary artery disease

A morbidly obese client asks the nurse if medications are available to assist with weight loss. The nurse knows that the client would not be a candidate for phentermine if the following is part of the client's health history:

Coronary artery disease Hinkle, J.L., & Cheever, K.H., Brunner & Suddarth's Textbook of Medical-Surgical Nursing, 14th ed., Philadelphia, Wolters Kluwer, 2018, Chapter 46: Management of Patients With Gastric and Duodenal Disorders, Pharmacologic Therapy, p. 1364.

A client with gastric cancer is having a resection. What is the nursing management priority for this client?

Correcting nutritional deficits

A client is having a diagnostic workup for reports of frequent diarrhea, right lower abdominal pain, and weight loss. The nurse is reviewing the results of the barium study and notes the presence of "string sign." What does the nurse understand that this is significant of?

Crohn's disease

A mother brings her teenage son to the clinic, where tests show that he has hepatitis A virus (HAV). They ask the nurse how this could have happened. Which of the following explanations would the nurse correctly identify as possible causes? Select all that apply.

Infection at school Suboptimal sanitary habits Consumption of sewage-contaminated water or shellfish Sexual activity Chapter 49: Assessment and Management of Patients With Hepatic Disorders - Page 1398 Typically, a child or a young adult acquires the infection at school through poor hygiene, hand-to-mouth contact, or close contact during play. The virus is carried home, where haphazard sanitary habits spread it through the family. An infected food handler can spread the disease, and people can contract it by consuming water or shellfish from sewage-contaminated waters. Outbreaks have occurred in day care centers and institutions as a result of poor hygiene among people with developmental disabilities. Hepatitis A can be transmitted during sexual activity. It is not contracted through the consumption of undercooked beef.

A nurse is assessing a client who reports abdominal pain, nausea, and diarrhea. When examining the client's abdomen, which sequence should the nurse use?

Inspection, auscultation, percussion, and palpation

Which is a true statement regarding the nursing considerations in administration of metronidazole?

It leaves a metallic taste in the mouth.

A nurse cares for a client who is post op bariatric surgery. Which position will the nurse place the client in order to best promote comfort?

Low Fowler's

A client who reports increasing difficulty swallowing, weight loss, and fatigue is diagnosed with esophageal cancer. Because this client has difficulty swallowing, what should the nurse assign highest priority to?

Maintaining a patent airway

Rebleeding may occur from a peptic ulcer and often warrants surgical interventions. Signs of bleeding include which of the following?

Mental confusion

Which of the following is considered a bulk-forming laxative?

Metamucil

A nurse is giving a client barium swallow test. What is the most important assessment a nurse would make to ensure that a client does not retain any barium after a barium swallow?

Monitoring the stool passage and its color.

The nurse working in the ED is evaluating a client for signs and symptoms of appendicitis. Which of the client's signs/symptoms should the nurse report to the physician?

Nausea

A patient who had a recent myocardial infarction was brought to the emergency department with bleeding esophageal varices and is presently receiving fluid resuscitation. What first-line pharmacologic therapy does the nurse anticipate administering to control the bleeding from the varices?

Octreotide (Sandostatin) Chapter 49: Assessment and Management of Patients With Hepatic Disorders - Page 1390 Octreotide (Sandostatin), a synthetic analogue of the hormone somatostatin, is effective in decreasing bleeding from esophageal varices, and lacks the vasoconstrictive effects of vasopressin. Because of this safety and efficacy profile, octreotide is considered the preferred treatment regimen for immediate control of variceal bleeding.

A nurse cares for clients with obesity who undergo bariatric surgery. Which clients will the nurse identify as having increased risk of infection after surgery? Select all that apply.

Older adults Adults with greater body mass Males Hinkle, J.L., & Cheever, K.H., Brunner & Suddarth's Textbook of Medical-Surgical Nursing, 14th ed., Philadelphia, Wolters Kluwer, 2018, Chapter 48: Assessment and Management of Patients with Obesity, p. 1372.

Which of the following the are early manifestations of liver cancer? Select all that apply.

Pain Continuous aching in the back Chapter 49: Assessment and Management of Patients With Hepatic Disorders - Page 1417 Early manifestations of liver cancer include pain and continuous dull aching in the right upper quadrant epigastrium or back. Weight loss, anorexia, and anemia may occur. Jaundice is present only if the larger bile ducts are occluded by the pressure of malignant nodules in the hilum of the liver. Fever and vomiting are not associated manifestations.

The nurse is working with clients with digestive tract disorders. Which of the following organs does the nurse realize has effects as an exocrine gland and an endocrine gland?

Pancreas

During assessment of a client for a malabsorption disorder, the nurse notes a history of abdominal pain and weight loss, marked steatorrhea, azotorrhea, and frequent glucose intolerance. Based on these clinical features, what diagnosis will the nurse suspect?

Pancreatic insufficiency

A patient reports an inflamed salivary gland below the right ear. The nurse documents probable inflammation of which gland?

Parotid

The nurse is preparing to assess the donor site of a client who underwent a myocutaneous flap after a radical neck dissection. The nurse prepares to assess the most commonly used muscle for this surgery. Which muscle should the nurse assess?

Pectoralis major

The nurse is assisting the health care provider with a gastric acid stimulation test for a client. What medication should the nurse prepare to administer subcutaneously to stimulate gastric secretions?

Pentagastrin

The client has returned to the floor following a radical neck dissection. Anesthesia has worn off. What is the nurse's priority action?

Place the client in the Fowler's position.

A client is being treated for prolonged diarrhea. Which foods should the nurse encourage the client to consume?

Potassium-rich foods

Which term is used to describe stone formation in a salivary gland, usually the submandibular gland?

Sialolithiasis

A client is recovering from gastric surgery. Toward what goal should the nurse progress the client's enteral intake?

Six small meals daily with 120 mL fluid between meals

What would the nurse recognize as preventing a client from being able to take a fecal occult blood test (FOBT)?

The client has hemorrhoidal bleeding

The nurse is inserting a nasogastric tube and the patient begins coughing and is unable to speak. What does the nurse suspect has occurred?

The nurse has inadvertently inserted the tube into the trachea.

A nurse cares for clients with obesity. Which clinical measurements use quantified measurements to diagnose obesity? Select all that apply.

Weight BMI Waist circumference

After teaching nursing students about methods to assess gastric tube placement, the instructor determines that the teaching was successful when the group identifies which of the following as the most accurate method?

X-ray visualization

A patient has been NPO for two days anticipating surgery which has been repeatedly delayed. In addition to risks of nutritional and fluid deficits, the nurse determines that this patient is at the greatest risk for:

altered oral mucous membranes.

The nurse attempts to unclog a client's feeding tube. Attempts with warm water agitation and milking the tube are unsuccessful. The nurse uses evidence-based practice principles when subsequently using which technique to unclog the tube?

digestive enzymes and sodium bicarbonate

The major carbohydrate that tissue cells use as fuel is

glucose.

An enzyme that begins the digestion of starches is

ptyalin.

The nurse teaches the client whose surgery will result in a sigmoid colostomy that the feces expelled through the colostomy will be

solid.

The nurse recognizes that blood shed in sufficient quantities into the upper GI tract produces which color of stool?

tarry black

The nurse is reviewing the results of a hemoccult test with the client. Which question asked by the nurse is important in screening for the potential of a false-positive result. Select all that apply.

"Are you prescribed regular strength aspirin daily?" "Can you tell me the amount of alcohol that you drink on an average week?" "When was the last time that you included red meat in your diet?"

A client reports having red stools lately. What will the nurse ask during assessment questioning?

"Have you been eating beets?"

When preparing a client for magnetic resonance imaging (MRI) of the abdomen, which statement would indicate the need to notify the health care provider?

"I really don't like to be in small, enclosed spaces."

A nurse is teaching a group of adults on the risks of obesity on neurological health. What statement will the nurse include in the teaching regarding obesity and the risk for developing Alzheimer disease?

"Individuals with obesity are twice as likely to develop Alzheimer disease than those who do not have obesity."

A nurse cares for a client with interstitial pancreatitis. What client teaching will the nurse include when planning care for the client?

"Inflammation is confined to only the pancreas." Hinkle, J.L., & Cheever, K.H., Brunner & Suddarth's Textbook of Medical-Surgical Nursing, 14th ed., Philadelphia, Wolters Kluwer, 2018, Chapter 50: Assessment and Management of Patients With Biliary Disorders, Acute Pancreatitis, pp. 1440-1441.

A nurse cares for a client who is obese. The health care provider prescribes orlistat in an effort to help client lose weight, along with diet and exercise. When teaching the client about this medication, what will the nurse include?

"It binds with enzymes to help prevent digestion of fat." Hinkle, J.L., & Cheever, K.H., Brunner & Suddarth's Textbook of Medical-Surgical Nursing, 14th ed., Philadelphia, Wolters Kluwer, 2018, Chapter 48: Assessment and Management of Patients with Obesity, p. 1365.

The nurse is conducting a health instruction program on oral cancer. The nurse determines that the participants understand the instructions when they state

"Many oral cancers produce no symptoms in the early stages."

Upon hearing that the small intestine lining has thinned, an elderly client asks, "What can this lead to?" What is the best response by the nurse?

"You may frequently experience constipation."

The nurse is providing medication administration teaching for a client with obesity who is prescribed liraglutide for weight loss. What will the nurse include in the teaching?

"You will be injecting the medication on a daily basis."

A client presented with gastrointestinal bleeding 2 days ago and continues to have problems. The health care provider has ordered a visualization of the small intestine via a capsule endoscopy. What will the nurse include in the client education about this procedure?

"You will need to swallow a capsule."

When completing a nutritional assessment of a patient who is admitted for a GI disorder, the nurse notes a recent history of dietary intake. This is based on the knowledge that a portion of digested waste products can remain in the rectum for how many days after a meal is digested?

3 days

A nurse caring for adults with obesity recognizes that obesity is classified based on BMI. Which BMI does the nurse recognize as Class II obesity?

35 kg/m2

As part of the process of checking the placement of a nasogastric tube, the nurse checks the pH of the aspirate. Which pH finding would indicate to the nurse that the tube is in the stomach?

4

A client has been prescribed a protein intake of 0.6 g/kg of body weight. The client weighs 154 pounds. The nurse calculates the daily protein intake to be how many grams? Enter the correct number ONLY.

42

A client who is being treated for pyloric obstruction has a nasogastric (NG) tube in place to decompress the stomach. The nurse routinely checks for obstruction which would be indicated by what amount?

450 mL

A nurse working in a bariatric clinic assesses various clients with obesity. Which clients will the nurse recognize as meeting the selection criteria for bariatric surgery? Select all that apply.

48-year-old female with BMI 36 k/m2 and uncontrolled type 2 diabetes. 34-year-old male with BMI 30 k/m2 and metabolic syndrome with hypertension. Hinkle, J.L., & Cheever, K.H., Brunner & Suddarth's Textbook of Medical-Surgical Nursing, 14th ed., Philadelphia, Wolters Kluwer, 2018, Chapter 48: Assessment and Management of Patients with Obesity, p. 1369.

When discussing lifestyle modifications with a client who has obesity, what caloric deficit should the nurse recommend in order for the client to safely lose weight?

500-1,000 calories Hinkle, J.L., & Cheever, K.H., Brunner & Suddarth's Textbook of Medical-Surgical Nursing, 14th ed., Philadelphia, Wolters Kluwer, 2018, Chapter 48: Assessment and Management of Patients with Obesity, p. 1363.

The nurse inserts a nasogastric tube into the right nares of a patient. When testing the tube aspirate for pH to confirm placement, what does the nurse anticipate the pH will be if placement is in the lungs?

6

A patient is receiving a continuous tube feeding. The nurse notes that the feeding tube was last irrigated at 2 p.m. The nurse would plan to irrigate the tube again at which time?

6 p.m. to 8 p.m. Chapter 44: Digestive and Gastrointestinal Treatment Modalities - Page 1246 The recommendation is to irrigate the feeding tube of patients receiving continuous tube feedings every 4 to 6 hours. For this patient, the nurse would irrigate the tube next at 6 p.m. to 8 p.m.

This example of cholesterol gallstones (left side of picture) is the result of decreased bile acid synthesis and increased cholesterol synthesis in the liver, which in turn, form stones. Cholesterol stones account for what percentage of cases of gallbladder disease in the United States?

75% Hinkle, J.L., & Cheever, K.H., Brunner & Suddarth's Textbook of Medical-Surgical Nursing, 14th ed., Philadelphia, Wolters Kluwer, 2018, Chapter 50: Assessment and Management of Patients With Biliary Disorders, Pathophysiology, p. 1430.

A client is recovering from a neck dissection. What volume of serosanguineous secretions would the nurse expect to drain over the first 24 hours?

80 to 120 mL

A client is instructed to follow a low-fat diet after an inflammatory attack of the gallbladder. Which vitamins will the nurse recommend the client supplement due to the client's dietary restrictions? Select all that apply.

A D K Essential fatty acids Hinkle, J.L., & Cheever, K.H., Brunner & Suddarth's Textbook of Medical-Surgical Nursing, 14th ed., Philadelphia, Wolters Kluwer, 2018, Chapter 50: Assessment and Management of Patients With Biliary Disorders, p. 1439.

When assessing a client during a routine checkup, the nurse reviews the history and notes that the client had aphthous stomatitis at the time of the last visit. How is aphthous stomatitis best described by the nurse?

A canker sore of the oral soft tissues

A nurse is preparing a presentation for a local community group of older adults about colon cancer. What would the nurse include as the primary characteristic associated with this disorder?

A change in bowel habits

A client is suspected of having cirrhosis of the liver. What diagnostic procedure will the nurse prepare the client for in order to obtain a confirmed diagnosis?

A liver biopsy Hinkle, J.L., & Cheever, K.H., Brunner & Suddarth's Textbook of Medical-Surgical Nursing, 14th ed., Philadelphia, Wolters Kluwer, 2018, Chapter 49: Assessment and Management of Patients With Hepatic Disorders, Assessment and Diagnostic Findings, p. 1408.

At its most fundamental level, what does obesity result from?

A metabolic imbalance Chapter 48: Assessment and Management of Patients with Obesity - Page 1360 At it's most fundamental level, obesity results from a metabolic imbalance characterized by an excess of caloric consumption relative to caloric expenditures.

A patient is scheduled for a Billroth I procedure for ulcer management. What does the nurse understand will occur when this procedure is performed?

A partial gastrectomy is performed with anastomosis of the stomach segment to the duodenum.

Which of the following is the primary function of the small intestine?

Absorption

A patient tells the nurse that it feels like food is "sticking" in the lower portion of the esophagus. What motility disorder does the nurse suspect these symptoms indicate?

Achalasia

A nurse is performing an assessment for a client who presents to the clinic with an erythemic, fissuring lip lesion with white hyperkeratosis. What does the nurse suspect that these findings are characteristic of?

Actinic cheilitis

While stripping wax from surfboards, a client accidentally ingested a refrigerated strong base cleaning solution, thinking it was water. What interventions would the nurse anticipate including in this client's care plan? Select all that apply.

Administer medication for report of pain. Insert an intravenous (IV) catheter for administration of IV fluids. Maintain nothing by mouth status. Assess respiratory status every 4 hours and prn.

The nurse is inserting a nasogastric tube for a patient with pancreatitis. What intervention can the nurse provide to allow facilitation of the tube insertion?

Allow the patient to sip water as the tube is being inserted.

The single modality of pharmacologic therapy for chronic type B viral hepatitis is:

Alpha-interferon Hinkle, J.L., & Cheever, K.H., Brunner & Suddarth's Textbook of Medical-Surgical Nursing, 14th ed., Philadelphia, Wolters Kluwer, 2018, Chapter 49: Assessment and Management of Patients With Hepatic Disorders, Medical Management, p. 1402.

Lactulose (Cephulac) is administered to a patient diagnosed with hepatic encephalopathy to reduce which of the following?

Ammonia Hinkle, J.L., & Cheever, K.H., Brunner & Suddarth's Textbook of Medical-Surgical Nursing, 14th ed., Philadelphia, Wolters Kluwer, 2018, Chapter 49: Assessment and Management of Patients With Hepatic Disorders, Medical Management, p. 1395.

A patient is suspected to have pancreatic carcinoma and is having diagnostic testing to determine insulin deficiency. What would the nurse determine is an indicator for insulin deficiency in this patient? (Select all that apply).

An abnormal glucose tolerance Glucosuria Hyperglycemia Hinkle, J.L., & Cheever, K.H., Brunner & Suddarth's Textbook of Medical-Surgical Nursing, 14th ed., Philadelphia, Wolters Kluwer, 2018, Chapter 50: Assessment and Management of Patients With Biliary Disorders, Clinical Manifestations, p. 1450.

A client with gastric cancer is scheduled to undergo a Billroth II procedure. The client's spouse asks how much of the client's stomach will be removed. What would be the most accurate response from the nurse?

Approximately 75%

The nurse is teaching a group of clients with obesity about the risks of disease associated with obesity. Which respiratory conditions or diseases will the nurse include in the teaching, which are associated with obesity? Select all that apply.

Asthma Infection Obstructive sleep apnea Hinkle, J.L., & Cheever, K.H., Brunner & Suddarth's Textbook of Medical-Surgical Nursing, 14th ed., Philadelphia, Wolters Kluwer, 2018, Chapter 48: Assessment and Management of Patients with Obesity, p. 1359.

When assisting with preparing a client scheduled for a barium swallow, which of the following would be appropriate to include?

Avoid smoking for at least 12 to 24 hours before the procedure.

When assisting with preparing a client scheduled for a barium swallow, what nursing instruction would be appropriate to include?

Avoid smoking for at least a day before the procedure. Chapter 43: Assessment of Digestive and Gastrointestinal Function - Page 1235 The nurse should instruct the client to avoid smoking for at least a day before the procedure of barium swallow because smoking stimulates gastric motility. The client is advised to take vitamin K before a liver biopsy and instructed to take three cleansing enemas before a barium enema. Instruction to avoid red meat would be appropriate for a client who is having a Hemoccult test.

The nurse instructs the client with gastroesophageal reflux disease (GERD) regarding dietary measures. Which action by the client demonstrates that the client has understood the recommended dietary changes?

Avoiding chocolate and coffee.

A client with a history of IV drug use is being treated for hepatitis, and presents today with jaundice and arthralgias. This client most likely has hepatitis:

B. Chapter 49: Assessment and Management of Patients With Hepatic Disorders - Page 1398 The client's presentation is most similar to hepatitis B. Mode of transmission is from infected blood or plasma, needles, syringes, surgical or dental equipment contaminated with infected blood; also sexually transmitted through vaginal secretions and semen of carriers or those actively infected. Mode of transmission for hepatitis C is similar to HBV, although less severe and without jaundice. Mode of transmission for hepatitis A is the oral route from feces and saliva of infected persons. The mode of transmission for hepatitis E is similar to HAV.

A client is scheduled for removal of the lower portion of the antrum of the stomach and a small portion of the duodenum and pylorus. What surgical procedure will the nurse prepare the client for?

Billroth I

Which term refers to intestinal rumbling?

Borborygmus

A nurse researcher is reviewing data obtained from a developing nation on nutrition and metabolism issues facing that country. What is the nurse's understanding of the "double-burden" many developing nations now face?

Both undernutrition and obesity Hinkle, J.L., & Cheever, K.H., Brunner & Suddarth's Textbook of Medical-Surgical Nursing, 14th ed., Philadelphia, Wolters Kluwer, 2018, Chapter 48: Assessment and Management of Patients with Obesity, p. 1358.

During a colonoscopy with moderate sedation, the patient groans with obvious discomfort and begins bleeding from the rectum. The patient is diaphoretic and has an increase in abdominal girth from distention. What complication of this procedure is the nurse aware may be occurring?

Bowel perforation

Which clinical manifestation is not associated with hemorrhage?

Bradycardia

When inspecting the abdomen of a client with cirrhosis, the nurse observes that the veins over the abdomen are dilated. The nurse documents this finding as which of the following?

Caput medusae Hinkle, J.L., & Cheever, K.H., Brunner & Suddarth's Textbook of Medical-Surgical Nursing, 14th ed., Philadelphia, Wolters Kluwer, 2018, Chapter 49: Assessment and Management of Patients With Hepatic Disorders, Gastrointestinal Varices, p. 1407.

A nurse cares for a client with obesity who takes anticonvulsant medication for the treatment of epilepsy. Which anticonvulsant medications may be contributing to the client's obesity? Select all that apply.

Carbamazepine Gabapentin

The primary source of microorganisms for catheter-related infections are the skin and which of the following?

Catheter hub

A client with calculi in the gallbladder is said to have

Cholelithiasis Hinkle, J.L., & Cheever, K.H., Brunner & Suddarth's Textbook of Medical-Surgical Nursing, 14th ed., Philadelphia, Wolters Kluwer, 2018, Chapter 50: Assessment and Management of Patients With Biliary Disorders, Disorders of the Gallbladder, p. 1430.

A patient is suspected to have diverticulosis without symptoms of diverticulitis. What diagnostic test does the nurse anticipate educating the patient about prior to scheduling?

Colonoscopy

Which nursing assessment is most important in a client diagnosed with ascites?

Daily measurement of weight and abdominal girth Hinkle, J.L., & Cheever, K.H., Brunner & Suddarth's Textbook of Medical-Surgical Nursing, 14th ed., Philadelphia, Wolters Kluwer, 2018, Chapter 49: Assessment and Management of Patients With Hepatic Disorders, p. 1386.

A nurse is assessing a client receiving tube feedings and suspects dumping syndrome. What would lead the nurse to suspect this? Select all that apply.

Diarrhea Tachycardia Diaphoresis

A client with active schizophrenia has developed acute gastritis after ingesting a strongly alkaline solution during a psychotic episode. Which emergency treatments should the nurse anticipate using with the client? Select all that apply.

Diluted lemon juice Diluted vinegar

The nurse reviews dietary guidelines with a client who had a gastric banding. Which teaching points are included? Select all that apply.

Do not eat and drink at the same time. Drink plenty of water, from 90 minutes after each meal to 15 minutes before each meal. Avoid fruit drinks and soda.

What test should the nurse prepare the client for that will locate stones that have collected in the common bile duct?

Endoscopic retrograde cholangiopancreatography (ERCP) Hinkle, J.L., & Cheever, K.H., Brunner & Suddarth's Textbook of Medical-Surgical Nursing, 14th ed., Philadelphia, Wolters Kluwer, 2018, Chapter 49: Assessment and Management of Patients With Hepatic Disorders, Endoscopic Retrograde Cholangiopancreatography, p. 1433.

A nurse is applying an ostomy appliance to the ileostomy of a client with ulcerative colitis. Which action is appropriate?

Gently washing the area surrounding the stoma using a facecloth and mild soap

A nurse working in a cardiac health care office notes increased risk of certain cardiac conditions as a result of obesity. Which conditions can be associated with obesity? Select all that apply.

Hypertension Coronary artery disease Heart failure Myocardial infarction

Gastrin has which of the following effects on gastrointestinal (GI) motility?

Increased motility of the stomach

The nurse is assessing a patient with appendicitis. The nurse is attempting to elicit a Rovsing's sign. Where should the nurse palpate for this indicator of acute appendicitis?

Left lower quadrant

Immediate medical and nursing management is necessary for a patient who has ingested a corrosive substance and experienced a chemical burn. Select the first response.

Maintain a patent airway.

The nurse is caring for an older adult patient experiencing fecal incontinence. When planning the care of this patient, what should the nurse designate as a priority goal?

Maintaining skin integrity

A client with obesity taking lorcaserin reports feeling agitated lately and has had diarrhea for several days. What is the nurse's priority response?

Notify the health care provider.

A client is receiving continuous tube feedings at 75 mL/h. When the nurse checked the residual volume 4 hours ago, it was 250 mL, and now the residual volume is 325 mL. What is the priority action by the nurse?

Notify the healthcare provider.

Which of the following medications is classified as a proton pump inhibitor (PPI)?

Omeprazole

The nurse is teaching a client who was admitted to the hospital with acute hepatic encephalopathy and ascites about an appropriate diet. The nurse determines that the teaching has been effective when the client chooses which food choice from the menu?

Pancakes with butter and honey, and orange juice

Which condition is the major cause of morbidity and mortality in clients with acute pancreatitis?

Pancreatic necrosis Hinkle, J.L., & Cheever, K.H., Brunner & Suddarth's Textbook of Medical-Surgical Nursing, 14th ed., Philadelphia, Wolters Kluwer, 2018, Chapter 50: Assessment and Management of Patients With Biliary Disorders, p. 1444.

The nurse is caring for a client with hepatitis. Which of the following would lead the nurse to suspect that the client is in the prodromal phase?

Rash Hinkle, J.L., & Cheever, K.H., Brunner & Suddarth's Textbook of Medical-Surgical Nursing, 14th ed., Philadelphia, Wolters Kluwer, 2018, Chapter 49: Assessment and Management of Patients With Hepatic Disorders, Table 49-4, p. 1398.

A client with obesity has been taking lorcaserin for several months and presents to the health care provider's office reporting fever and diarrhea. Which life-threatening condition does the nurse suspect?

Serotonin syndrome

The nurse is instructing the client who was newly diagnosed with peptic ulcers. Which of the following diagnostic studies would the nurse anticipate reviewing with the client?

Serum antibodies for H. pylori

The nurse is asking the client with acute pancreatitis to describe the pain. What pain symptoms does the client describe related to acute pancreatitis?

Severe mid-abdominal to upper abdominal pain radiating to both sides and to the back Hinkle, J.L., & Cheever, K.H., Brunner & Suddarth's Textbook of Medical-Surgical Nursing, 14th ed., Philadelphia, Wolters Kluwer, 2018.

The client has a chancre on the lips. What instruction should the nurse provide?

Take measures to prevent spreading the lesion to other people.

Which outcome indicates effective client teaching to prevent constipation?

The client reports engaging in a regular exercise regimen.

The nurse is caring for a geriatric client experiencing diarrhea. When teaching about the site in the body where water and electrolytes are absorbed, the nurse is most correct to instruct on which location?

The large intestine

The presence of mucus and pus in the stools suggests which condition?

Ulcerative colitis

The nurse is assessing a client for constipation. Which factor should the nurse review first to identify the cause of constipation?

Usual pattern of elimination

Which of the following is the most common type of diverticulum?

Zenker's diverticulum

The digestion of carbohydrates is aided by

amylase. Hinkle, J.L., & Cheever, K.H., Brunner & Suddarth's Textbook of Medical-Surgical Nursing, 14th ed., Philadelphia, Wolters Kluwer, 2018, Chapter 50: Assessment and Management of Patients With Biliary Disorders, p. 1429.

A longitudinal tear or ulceration in the lining of the anal canal is termed a(n):

anal fissure.

When caring for a client with cirrhosis, which symptoms should a nurse report immediately? Select all that apply.

change in mental status signs of GI bleeding Hinkle, J.L., & Cheever, K.H., Brunner & Suddarth's Textbook of Medical-Surgical Nursing, 14th ed., Philadelphia, Wolters Kluwer, 2018, Chapter 49: Assessment and Management of Patients With Hepatic Disorders, p. 1407.

A client is scheduled to have an endoscopic retrograde cholangiopancreatography. Which structures are visualized during this procedure?

common bile duct, pancreatic duct, and biliary tree Chapter 43: Assessment of Digestive and Gastrointestinal Function - Page 1237 With the use of endoscopy, dye is injected through a catheter into the common bile duct and the pancreatic duct, permitting visualization and evaluation of the biliary tree. The common bile duct, the pancreatic duct, and the biliary tree are visualized.

A client who can't tolerate oral feedings begins receiving intermittent enteral feedings. When monitoring for evidence of intolerance to these feedings, what must the nurse remain alert for?

diaphoresis, vomiting, and diarrhea.

A client is scheduled to undergo rhinoplasty in the morning, and reports medications used on a daily basis, which the nurse records on the client's chart. Which daily medications have the potential to result in constipation?

laxative

The client is on a continuous tube feeding. The nurse determines the tube placement should be checked every

shift.

The nurse is caring for client scheduled to undergo radical neck dissection. During preoperative teaching, the nurse states that an associated complication is

shoulder drop.

The nurse recognizes which change of the GI system is an age-related change?

weakened gag reflex

A nurse is educating a client who will undergo bariatric surgery on methods to prevent dysphagia. What teaching will the nurse include? Select all that apply.

"Eat slowly." "Chew your food thoroughly." "Avoid eating tough foods." "Avoid eating overcooked meats."

A client who has occasional gastric symptoms is receiving teaching on how to prevent gastroesophageal reflux disease (GERD). Which statement indicates the client understands the teaching?

"Eliminating bothersome foods will help."

A nurse cares for a client who wants to know more information about bariatric surgery. The client asks the nurse, "What weight loss can I expect?" What is the nurse's best response?

"Expect to lose 10-35% of total body weight 2 to 3 years postoperatively."

A client is readmitted with an exacerbation of celiac disease 2 weeks after discharge. Which statement by the client indicates the need for a dietary consult?

"I didn't eat anything I shouldn't have; I just ate roast beef on rye bread."

When preparing a client for magnetic resonance imaging (MRI) of the abdomen, which statement would indicate the need to notify the physician?

"I really don't like to be in small, enclosed spaces."

After undergoing a total cystectomy and urinary diversion, a client has a Kock pouch (continent internal reservoir) in place. Which statement by the client indicates a need for further teaching?

"I'll have to wear an external collection pouch for the rest of my life."

A nurse is teaching a client who has experienced an episode of acute gastritis. The nurse knows further education is necessary when the client makes which statement?

"My appetite should come back tomorrow."

A nurse cares for a client with a BMI of 36 kg/m2 and nonalcoholic fatty liver disease. The client asks the nurse if he is a candidate for bariatric surgery. How should the nurse respond to the client?

"Yes, your BMI and chronic condition meets the criteria for bariatric surgery."

A nurse cares for a female client of childbearing age who will undergo bariatric surgery. When teaching the client about precautions after surgery, which teaching will the nurse include that is specific to this population?

"You should avoid pregnancy for at least 18 months after surgery." Hinkle, J.L., & Cheever, K.H., Brunner & Suddarth's Textbook of Medical-Surgical Nursing, 14th ed., Philadelphia, Wolters Kluwer, 2018, Chapter 48: Assessment and Management of Patients with Obesity, p. 1373.

A client with end-stage liver disease is scheduled to undergo a liver transplant. The client tells the nurse, "I am worried that my body will reject the liver." Which statement is the nurse's best response to the client?

"You will need to take daily medication to prevent rejection of the transplanted liver. The new liver has a good chance of survival with the use of these drugs." Hinkle, J.L., & Cheever, K.H., Brunner & Suddarth's Textbook of Medical-Surgical Nursing, 14th ed., Philadelphia, Wolters Kluwer, 2018, Chapter 49: Assessment and Management of Patients With Hepatic Disorders, p. 1423.

A nurse researches the cost and financial impact of obesity in America. What is the annual health care cost tied to obesity?

$147 billion

A patient has a gastric sump tube attached to low intermittent suction. The nurse empties the suction collection chamber and records an output of 320 mL for this 8-hour shift. The record shows that the tube had been irrigated with 20 mL of normal saline twice this shift. What would be the actual output of the gastric sump tube?

280 Chapter 44: Digestive and Gastrointestinal Treatment Modalities - Page 1246 The output measured includes the two 20 mL irrigations. To determine the actual output, the nurse would subtract the amount of irrigation used (in this case 40 mL total) from the total output (in this case 320 mL) and arrive at an output of 280 mL.

A nurse should monitor blood glucose levels for a patient diagnosed with hyperinsulinism. What blood value does the nurse recognize as inadequate to sustain normal brain function?

30 mg/dL Hinkle, J.L., & Cheever, K.H., Brunner & Suddarth's Textbook of Medical-Surgical Nursing, 14th ed., Philadelphia, Wolters Kluwer, 2018, Chapter 50: Assessment and Management of Patients With Biliary Disorders, Hyperinsulinism, p. 1453.

After assessing a client with peritonitis, how would the nurse most likely document the client's bowel sounds?

Absent

A client is postoperative following a graft reconstruction of the neck. What intervention is the most important for the nurse to complete with the client?

Assess the graft for color and temperature.

Which of the following is considered the gold standard for the diagnosis of liver disease?

Biopsy

The nurse is to obtain a stool specimen from a client who reported that he is taking iron supplements. The nurse would expect the stool to be which color?

Black

When assessing a client with cirrhosis of the liver, which of the following stool characteristics is the client likely to report?

Clay-colored or whitish Hinkle, J.L., & Cheever, K.H., Brunner & Suddarth's Textbook of Medical-Surgical Nursing, 14th ed., Philadelphia, Wolters Kluwer, 2018.

Which of the following clients is at highest risk for peptic ulcer disease?

Client with blood type O

A client has given a confirmed diagnosis of gastric cancer. Two more procedures may be performed to assess tumor depth and lymph node involvement and surgical respectability. Which two are the procedures? Choose the two that apply.

Computed tomography (CT) Endoscopic ultrasound

A nurse cares for a client who has secondary obesity. Which condition is the most likely to result in secondary obesity?

Cushing's disease

A client comes to the emergency department complaining of pain in the right leg. When obtaining the history, the nurse learns that the client has a history of obesity and hypertension. Based on this information the nurse anticipates the client having which musculoskeletal disorder?

Degenerative joint disease

Which is the primary symptom of achalasia?

Difficulty swallowing

The nurse is caring for a client with suspected chronic pancreatitis. Which diagnostic test or imaging does the nurse recognize as the most useful in diagnosing this condition?

ERCP Hinkle, J.L., & Cheever, K.H., Brunner & Suddarth's Textbook of Medical-Surgical Nursing, 14th ed., Philadelphia, Wolters Kluwer, 2018, Chapter 50: Assessment and Management of Patients With Biliary Disorders, p. 1445.

When caring for a client with the impaired swallowing related to neuromuscular impairment, what is the nurse's priority intervention?

Elevate the head of the bed 90 degrees during meals.

Postoperatively, a client with a radical neck dissection should be placed in which position?

Fowler

What pathophysiological concept is related to the increase in the hormone leptin, as it relates to satiety and hunger?

Increased adipose stores

A nurse caring for clients with obesity understands these clients are at increased risk for developing pressure ulcers. What does the nurse recognize increases the client's risk for developing pressure ulcers? Select all that apply.

Increased adipose tissue decreases the supply of blood, oxygen, and nutrients to peripheral tissue. Skin folds are associated with more moisture and friction.

A client is admitted with increased ascites related to cirrhosis. Which nursing diagnosis should receive top priority?

Ineffective breathing pattern Hinkle, J.L., & Cheever, K.H., Brunner & Suddarth's Textbook of Medical-Surgical Nursing, 14th ed., Philadelphia, Wolters Kluwer, 2018, Chapter 49: Assessment and Management of Patients With Hepatic Disorders, Chart 49-11, p. 1414.

The client is receiving a 25% dextrose solution of parenteral nutrition. The infusion machine is beeping, and the nurse determines the intravenous (IV) bag is empty. The nurse finds there is no available bag to administer. What is the priority action by the nurse?

Infuse a solution containing 10% dextrose and water.

A client is diagnosed with megaloblastic anemia caused by vitamin B12 deficiency. The health care provider begins the client on cyanocobalamin (Betalin-12), 100 mcg I.M. daily. Which substance influences vitamin B12 absorption?

Intrinsic factor

A patient with irritable bowel syndrome has been having more frequent symptoms lately and is not sure what lifestyle changes may have occurred. What suggestion can the nurse provide to identify a trigger for the symptoms?

Keep a 1- to 2-week symptom and food diary to identify food triggers.

Which enzyme aids in the digestion of fats?

Lipase Hinkle, J.L., & Cheever, K.H., Brunner & Suddarth's Textbook of Medical-Surgical Nursing, 14th ed., Philadelphia, Wolters Kluwer, 2018, Chapter 50: Assessment and Management of Patients With Biliary Disorders, p. 1429.

Diet modifications for patient diagnosed with chronic inflammatory bowel disease include which of the following?

Low residue

Which of the following is an enzyme secreted by the gastric mucosa?

Pepsin

A client who is post op bariatric surgery reports diarrhea. What is the most likely cause of the client's symptoms?

Poor dietary choices Hinkle, J.L., & Cheever, K.H., Brunner & Suddarth's Textbook of Medical-Surgical Nursing, 14th ed., Philadelphia, Wolters Kluwer, 2018, Chapter 48: Assessment and Management of Patients with Obesity, p. 1371.

A nurse cares for an obese client taking phentermine/topiramate-ER. Which laboratory findings will the nurse recognize as most concerning and will report to the health care provider? Select all that apply.

Potassium 3.3 mEq/L Bicarbonate 19 mEq/L Hinkle, J.L., & Cheever, K.H., Brunner & Suddarth's Textbook of Medical-Surgical Nursing, 14th ed., Philadelphia, Wolters Kluwer, 2018, Chapter 48: Assessment and Management of Patients with Obesity, p. 1365.

The nurse is caring for a client with cirrhosis. Which assessment findings indicate that the client has deficient vitamin K absorption caused by this hepatic disease?

Purpura and petechiae Hinkle, J.L., & Cheever, K.H., Brunner & Suddarth's Textbook of Medical-Surgical Nursing, 14th ed., Philadelphia, Wolters Kluwer, 2018, Chapter 49: Assessment and Management of Patients With Hepatic Disorders, Chart 49-10, p. 1407.

When interviewing a client with internal hemorrhoids, what would the nurse expect the client to report?

Rectal bleeding

Which of the following is the most common symptom of a polyp?

Rectal bleeding

The nurse advises the patient who has just been diagnosed with acute gastritis to:

Refrain from food until the GI symptoms subside.

Which of the following medications used for obesity improves cardiovascular disease risk factors in obese patients with metabolic syndrome?

Rimonabant (Acomplia) Hinkle, J.L., & Cheever, K.H., Brunner & Suddarth's Textbook of Medical-Surgical Nursing, 14th ed., Philadelphia, Wolters Kluwer, 2018.

Which of the following refers to a bacterial or viral infection of the salivary glands?

Sialadenitis

A client is admitted with a gastrointestinal bleed. What client symptom may indicate a peptic ulcer perforation to the nurse?

Sudden, severe upper abdominal pain

A client taking metronidazole for the treatment of H. pylori states that the medication is causing nausea. What teaching should the nurse provide to the client to alleviate the nausea?

Take the medication with meals to decrease the nausea.

A client is admitted to the hospital for diagnostic testing to rule out colorectal cancer. Which intervention should the nurse include on the plan of care?

Test all stools for occult blood.

A nurse is educating a client who has been treated for hepatic encephalopathy about dietary restrictions to prevent ammonia accumulation. What should the nurse include in the dietary teaching?

The amount of protein is not restricted in the diet. Hinkle, J.L., & Cheever, K.H., Brunner & Suddarth's Textbook of Medical-Surgical Nursing, 14th ed., Philadelphia, Wolters Kluwer, 2018, Chapter 49: Assessment and Management of Patients With Hepatic Disorders, Pathophysiology, p. 1396.

A client with dysphagia is being prepared for discharge. Which outcome indicates that the client is ready for discharge?

The client is free from esophagitis and achalasia.

A nurse cares for a client who is post op bariatric surgery and the nurse offers the client a sugar-free beverage. What is the primary purpose of offering a sugar-free beverage?

These are less likely to cause dumping syndrome.

A client receives tube feedings after an oral surgery. The nurse manages tube feedings to minimize the risk of aspiration. Which measure should the nurse include in the care plan to reduce the risk of aspiration?

Use semi-Fowler position during, and 60 minutes after, an intermittent feeding.

A client with chronic pancreatitis is treated for uncontrolled pain. Which complication does the nurse recognize is most common in the client with chronic pancreatitis?

Weight loss Hinkle, J.L., & Cheever, K.H., Brunner & Suddarth's Textbook of Medical-Surgical Nursing, 14th ed., Philadelphia, Wolters Kluwer, 2018, Chapter 50: Assessment and Management of Patients With Biliary Disorders, Clinical Manifestations, p. 1445.

What is a major concern for the nurse when caring for a patient with chronic pancreatitis?

Weight loss Hinkle, J.L., & Cheever, K.H., Brunner & Suddarth's Textbook of Medical-Surgical Nursing, 14th ed., Philadelphia, Wolters Kluwer, 2018, Chapter 50: Assessment and Management of Patients With Biliary Disorders, Clinical Manifestations, p. 1445.

A client with newly diagnosed type 2 diabetes is admitted to the metabolic unit. The primary goal for this admission is education. Which goal should the nurse incorporate into her teaching plan?

Weight reduction through diet and exercise Hinkle, J.L., & Cheever, K.H., Brunner & Suddarth's Textbook of Medical-Surgical Nursing, 14th ed., Philadelphia, Wolters Kluwer, 2018, Chapter 48: Assessment and Management of Patients with Obesity, Lifestyle Modification, p. 1364.

A client with enteritis reports frequent diarrhea. What assessment should the nurse should anticipate?

metabolic acidosis

Which condition is caused by improper catheter placement and inadvertent puncture of the pleura?

pneumothorax

Which client requires immediate nursing intervention? The client who:

presents with a rigid, board-like abdomen.

A client tells the nurse that the stool was colored yellow. The nurse assesses the client for

recent foods ingested.

A client has been experiencing lower GI difficulties that have increased in severity, and the gastroenterologist is concerned that the client's bowel is not functioning properly. What function of the lower GI tract is most likely to be affected by the client's disorder?

water and electrolyte absorption

A client who is 24 hours post op from laparoscopic cholecystectomy calls the nurse and reports pain in the right shoulder. How should the nurse respond to the client's report of symptoms?

"Apply a heating pad to your shoulder for 15 minutes hourly as needed." Hinkle, J.L., & Cheever, K.H., Brunner & Suddarth's Textbook of Medical-Surgical Nursing, 14th ed., Philadelphia, Wolters Kluwer, 2018, Chapter 50: Assessment and Management of Patients With Biliary Disorders, p. 1437.

To prevent gastroesophageal reflux in a client with hiatal hernia, the nurse should provide which discharge instruction?

"Avoid coffee and alcoholic beverages."

A nurse is providing discharge instruction for a client who is postoperative bariatric surgery. What statement will the nurse include when providing teaching aimed at decreasing the risk of gastric ulcers?

"Avoid taking non-steroidal anti-inflammatory drugs."

A client is prescribed tetracycline to treat peptic ulcer disease. Which instruction would the nurse give the client?

"Be sure to wear sunscreen while taking this medicine."

A nurse is teaching a client with gastritis about the need to avoid the intake of caffeinated beverages. The client asks why this is so important. Which explanation from the nurse would be most accurate?

"Caffeine stimulates the central nervous system and thus gastric activity and secretions, which need to be minimized to promote recovery."

A client who is post op from bariatric surgery tells the nurse, "I love cereal for breakfast. Do you have recommendations for which kind I should eat?" How should the nurse respond?

"Choose a cereal with less than 2 grams of fiber per serving."

A client admitted for treatment of a gastric ulcer is being prepared for discharge. The client will follow a regimen of antacid therapy. Discharge teaching should include which instructions? Choose all that apply.

"Continue to take antacids even if your symptoms subside." "You may be prescribed H2-receptor antagonists for up to 1 year."

A nurse cares for an obese client taking phentermine for weight loss. What client teaching will the nurse include when discussing precautions about the medication?

"Do not drink alcohol while taking this medication."

A nurse evaluates the effectiveness of discharge teaching for a client who is postoperative from bariatric surgery. Which statement made by the client indicates to the nurse that additional teaching is necessary?"

"For minor pain, I should take prescribed ibuprofen."

The nurse is caring for a man who has experienced a spinal cord injury. Throughout his recovery, the client expects to gain control of his bowels. The nurse's best response to this client would be which of the following?

"Having a bowel movement is a spinal reflex requiring intact nerve fibers. Yours are not intact."

A nurse is interviewing a patient to determine suitability for home parenteral nutrition. Which patient statement would alert the nurse to a potential problem?

"I have a telephone, but it has been shut off because my bill is overdue."

The nurse determines that teaching for the client with peptic ulcer disease has been effective when the client makes which statement?

"I have learned some relaxation strategies that decrease my stress."

After teaching a client who has had a Roux-en-Y gastric bypass, which client statement indicates the need for additional teaching?

"I need to drink 8 oz of water before eating."

A nurse manager prepares teaching for staff nurses who care for clients with diabetes. Which statements will the nurse manager include when discussing the differences between the endocrine and exocrine functions of the pancreas? Select all that apply.

"Internal secretion of hormones is the function of the endocrine pancreas." "The endocrine pancreas secretes hormones through a ductless gland." "The exocrine pancreas secretes hormones from excretory ducts." "The exocrine pancreas secretes pancreatic enzymes into the GI tract." Hinkle, J.L., & Cheever, K.H., Brunner & Suddarth's Textbook of Medical-Surgical Nursing, 14th ed., Philadelphia, Wolters Kluwer, 2018, Chapter 50: Assessment and Management of Patients With Biliary Disorders, p. 1429.

A nurse is preparing a client for endoscopic retrograde cholangiopancreatography (ERCP). The client asks what this test is used for. Which statements by the nurse explains how ERCP can determine the difference between pancreatitis and other biliary disorders? Select all that apply.

"It can evaluate the presence and location of ductal stones and aid in stone removal." "It can assess the anatomy of the pancreas and the pancreatic and biliary ducts." "It can detect unhealthy tissues in the pancreas and assess for abscesses and pseudocysts." Hinkle, J.L., & Cheever, K.H., Brunner & Suddarth's Textbook of Medical-Surgical Nursing, 14th ed., Philadelphia, Wolters Kluwer, 2018, Chapter 50: Assessment and Management of Patients With Biliary Disorders, p. 1433.

A nurse reviews with the client the various types of medications used to treat diabetes. Which statement will the nurse use when teaching the client about liraglutide's mechanism of action?

"It causes delayed gastric emptying." Hinkle, J.L., & Cheever, K.H., Brunner & Suddarth's Textbook of Medical-Surgical Nursing, 14th ed., Philadelphia, Wolters Kluwer, 2018, Chapter 48: Assessment and Management of Patients with Obesity, p. 1365.

A nurse cares for a client with obesity who is scheduled to undergo vagal blocking therapy. When teaching the client about the procedure or device, which statements will the nurse include? Select all that apply.

"It is a pacemaker-type device that is implanted under your skin." "A pre-programed pulsating signal is delivered." "Recharge the device two times per week."

A client comes to the clinic reporting pain in the epigastric region. What statement by the client suggests the presence of a duodenal ulcer?

"My pain resolves when I have something to eat."

The nurse asks a client to point to where pain is felt. The client asks why this is important. What is the nurse's best response?

"Often the area of pain is referred from another area."

A nurse prepares community teaching on healthy lifestyle modifications to a group of older adults. When discussing obesity rates of older adults in comparison with the rest of the population, what will the nurse include?

"Older adults have a slightly higher prevalence of obesity in comparison to the general population." Hinkle, J.L., & Cheever, K.H., Brunner & Suddarth's Textbook of Medical-Surgical Nursing, 14th ed., Philadelphia, Wolters Kluwer, 2018, Chapter 48: Assessment and Management of Patients with Obesity, p. 1366.

A client with hepatic cirrhosis questions the nurse about the possible use of an herbal supplement—milk thistle—to help heal the liver. Which is the most appropriate response by the nurse?

"Silymarin from milk thistle has anti-inflammatory and antioxidant properties that may have beneficial effects, especially in hepatitis. However, you should always notify your primary care provider of any herbal remedies being used so drug interactions can be evaluated." Hinkle, J.L., & Cheever, K.H., Brunner & Suddarth's Textbook of Medical-Surgical Nursing, 14th ed., Philadelphia, Wolters Kluwer, 2018, Chapter 49: Assessment and Management of Patients With Hepatic Disorders, p. 1408.

A client is scheduled for a Roux-en-Y bariatric surgery. When teaching the client about the surgical procedure, which statement will the nurse use?

"The stomach is stapled to create a very small pouch and part of the small intestine is rerouted."

A nurse cares for a client who is post op cholecystectomy due to the development of cholesterol stones. The client asks the nurse, "Why did I develop these stones?" What is the nurse's best response?

"The stones formed from a component in the bile and are caused by bile acid and fat abnormalities in the bloodstream." Hinkle, J.L., & Cheever, K.H., Brunner & Suddarth's Textbook of Medical-Surgical Nursing, 14th ed., Philadelphia, Wolters Kluwer, 2018, Chapter 50: Assessment and Management of Patients With Biliary Disorders, pp. 1430-1431.

A client has a nasogastric (NG) tube for suction and is NPO after a pancreaticoduodenectomy. Which explanation made by the nurse is the major purpose of this treatment?

"The tube allows the gastrointestinal tract to rest." Hinkle, J.L., & Cheever, K.H., Brunner & Suddarth's Textbook of Medical-Surgical Nursing, 14th ed., Philadelphia, Wolters Kluwer, 2018, Chapter 50: Assessment and Management of Patients With Biliary Disorders, p. 1452.

A community health nurse prepares teaching regarding gut health and the impact on obesity. What will the nurse include on how the "Western diet" impacts gut health? Select all that apply.

"This diet decreases the diversity of gut microbes." "This diet decreases the gut microbes that lead to a leaner body." Hinkle, J.L., & Cheever, K.H., Brunner & Suddarth's Textbook of Medical-Surgical Nursing, 14th ed., Philadelphia, Wolters Kluwer, 2018, Chapter 48: Assessment and Management of Patients with Obesity, p. 1361.

A client discharged after a laparoscopic cholecystectomy calls the surgeon's office reporting severe right shoulder pain 24 hours after surgery. Which statement is the correct information for the nurse to provide to this client?

"This pain is caused from the gas used to inflate your abdominal area during surgery. Sitting upright in a chair, walking, or using a heating pad may ease the discomfort." Hinkle, J.L., & Cheever, K.H., Brunner & Suddarth's Textbook of Medical-Surgical Nursing, 14th ed., Philadelphia, Wolters Kluwer, 2018, Chapter 50: Assessment and Management of Patients With Biliary Disorders, p. 1437.

A client who is 6 months postoperative bariatric surgery tells the nurse, "I hate what my body looks like now. All these skin folds really bother me." What is the nurse's best response?

"You are not alone in having these feelings." Hinkle, J.L., & Cheever, K.H., Brunner & Suddarth's Textbook of Medical-Surgical Nursing, 14th ed., Philadelphia, Wolters Kluwer, 2018, Chapter 48: Assessment and Management of Patients with Obesity, p. 1372.

A client who is obese and the nurse have established a goal for the client to achieve a weight loss of 1 pound each week. One month later, the nurse evaluates that the client has lost 2 pounds. The nurse first states

"You have succeeded in making positive progress." Hinkle, J. L., Cheever, K. H. Brunner & Suddarth's Textbook of Medical-Surgical Nursing , 14th ed Philadelphia: Wolters Kluwer Health/Lippincott Williams & Wilkins, 2018, Chapter 9: Chronic Illness and Disability, p. 148.

A student accepted into a nursing program must begin receiving the hepatitis B series of injections. The student asks when the next two injections should be administered. What is the best response by the instructor?

"You must have the second one in 1 month and the third in 6 months." Hinkle, J.L., & Cheever, K.H., Brunner & Suddarth's Textbook of Medical-Surgical Nursing, 14th ed., Philadelphia, Wolters Kluwer, 2018, Chapter 49: Assessment and Management of Patients With Hepatic Disorders, Medical Management, p. 1402.

The nurse cares for a client who is post op bariatric surgery whose bowel sounds have returned and is tolerating oral intake. What teaching will the nurse provide the client about the nutrition required at this time? Select all that apply.

"You need to drink plenty of water in between meals." "Eat slowly." "You will have six small meals per day."

Medical management of a patient with peritonitis includes fluid, electrolyte, and colloid replacement. The nurse knows to prepare the initial, most appropriate intravenous solution. Which of the following is the correct solution?

0.9% NS

A nurse is responsible for monitoring the diet of a client with hepatic encephalopathy. Which daily protein intake should this 185-pound (84-kilogram) male consume?

100 to 126 grams Chapter 49: Assessment and Management of Patients With Hepatic Disorders - Page 1396 Daily protein intake should be maintained at 1.2 to 1.5 g/kg per day. To calculate protein intake, multiply the 84 kg by the low and high ranges: (84 kg × 1.2 = 100 g; 84 kg × 1.5 = 126 g). This client would require a daily protein range from 100 to 126 grams.

A patient has a gastric sump tube inserted and attached to low intermittent suction. The physician has ordered the tube to be irrigated with 30 mL of normal saline every 6 hours. When reviewing the patient's intake and output record for the past 24 hours, the nurse would expect to note that the patient received how much fluid with the irrigation?

120

A nursing student is preparing a teaching plan about peptic ulcer disease. The student knows to include teaching about the percentage of clients with peptic ulcers who experience bleeding. The percentage is

15%

A nurse measures the residual gastric volume of a patient receiving intermittent tube feedings. The patient's last residual volume was 250 mL. Which finding would lead the nurse to notify the physician?

225 mL Chapter 44: Digestive and Gastrointestinal Treatment Modalities - Page 1251 If a residual volume greater than 200 mL is obtained twice, the nurse would need to notify the physician. A single residual volume of 200 mL or more does not indicate a need to withhold a feeding. Feedings may be continued in patients as long as there is close monitoring of gastric residual volume trends, x-ray study results, and the patient's physical status.

Calculate the BMI of a client who weighs 160 lbs and is 5 feet 6 inches tall. Round to one decimal.

25.8

What is the body mass index (BMI) of a client who weighs 195 lbs and is 6 feet tall? Round to the nearest whole number, and enter the correct number ONLY.

26

Calculate the BMI of a client who is 6 feet 1 inch tall and weighs 200 pounds. Round to one decimal.

26.4

The client has the intake and output shown in the accompanying chart for an 8-hour shift. What is the positive fluid balance?

260

A patient is receiving continuous tube feedings via a small bore feeding tube. The nurse irrigates the tube after administering medication to maintain patency. Which size syringe would the nurse use?

30-mL Chapter 44: Digestive and Gastrointestinal Treatment Modalities - Page 1250 When small-bore feeding tubes for continuous tube feedings are used and irrigated after administration of medications, a 30-mL or larger syringe is necessary, because the pressure generated by smaller syringes could rupture the tube.

The nurse knows that the serum amylase concentration returns to normal within which time frame?

48 hours Hinkle, J.L., & Cheever, K.H., Brunner & Suddarth's Textbook of Medical-Surgical Nursing, 14th ed., Philadelphia, Wolters Kluwer, 2018, Chapter 50: Assessment and Management of Patients With Biliary Disorders, p. 1442.

A client has developed an anorectal abscess. Which client is at most risk for the development of this type of abscess?

A client with Crohn's disease

The nurse is educating a patient with cirrhosis about the importance of maintaining a low-sodium diet. What food item would be permitted on a low-sodium diet?

A pear Hinkle, J.L., & Cheever, K.H., Brunner & Suddarth's Textbook of Medical-Surgical Nursing, 14th ed., Philadelphia, Wolters Kluwer, 2018, Chapter 49: Assessment and Management of Patients With Hepatic Disorders, Improving Nutritional Status, p. 1408.

The nurse is assisting the physician with a procedure to remove ascitic fluid from a client with cirrhosis. What procedure does the nurse ensure the client understands will be performed?

Abdominal paracentesis Hinkle, J.L., & Cheever, K.H., Brunner & Suddarth's Textbook of Medical-Surgical Nursing, 14th ed., Philadelphia, Wolters Kluwer, 2018, Chapter 49: Assessment and Management of Patients With Hepatic Disorders, Paracentesis, p. 1387.

An important message for any nurse to communicate is that drug-induced hepatitis is a major cause of acute liver failure. The medication that is the leading cause is:

Acetaminophen Hinkle, J.L., & Cheever, K.H., Brunner & Suddarth's Textbook of Medical-Surgical Nursing, 14th ed., Philadelphia, Wolters Kluwer, 2018, Chapter 49: Assessment and Management of Patients With Hepatic Disorders, Drug-Induced Hepatitis, p. 1405.

A client is evaluated for severe pain in the right upper abdominal quadrant, which is accompanied by nausea and vomiting. The physician diagnoses acute cholecystitis and cholelithiasis. For this client, which nursing diagnosis takes top priority?

Acute pain related to biliary spasms Hinkle, J.L., & Cheever, K.H., Brunner & Suddarth's Textbook of Medical-Surgical Nursing, 14th ed., Philadelphia, Wolters Kluwer, 2018, Chapter 50: Assessment and Management of Patients With Biliary Disorders, Pain and Biliary Colic, pp. 1431-1432.

The charge nurse was discussing with the nursing student that studies have been published that suggest inflammation increases the risk of heart disease. Which modifiable factor would the nursing student target in teaching clients about prevention of inflammation that can lead to atherosclerosis?

Addressing obesity Hinkle, J.L., & Cheever, K.H., Brunner & Suddarth's Textbook of Medical-Surgical Nursing, 14th ed., Philadelphia, Wolters Kluwer, 2018, Chapter 27: Management of Patients with Coronary Vascular Disorders, Risk Factors, p. 851.

A client with newly diagnosed hypertension asks what to do to decrease the risk for related cardiovascular problems. Which risk factor is not modifiable by the client?

Age Hinkle, J.L., & Cheever, K.H., Brunner & Suddarth's Textbook of Medical-Surgical Nursing, 14th ed., Philadelphia, Wolters Kluwer, 2018, Chapter 31: Assessment and Management of Patients With Hypertension, Chart 31-1, p. 887.

A nursing student has been assigned to care for a client with pancreatic cancer. The student is aware that the risk for pancreatic cancer is most directly proportional to

Age Hinkle, J.L., & Cheever, K.H., Brunner & Suddarth's Textbook of Medical-Surgical Nursing, 14th ed., Philadelphia, Wolters Kluwer, 2018, Chapter 50: Assessment and Management of Patients With Biliary Disorders, Cancer of the Pancreas, p. 1450.

A client has a family history of stomach cancer. Which factor would further increase the client's risk for developing gastric cancer? Select all that apply.

Age 55 years Previous infection with H. pylori

A client with liver and renal failure has severe ascites. On initial shift rounds, his primary nurse finds his indwelling urinary catheter collection bag too full to store more urine. The nurse empties more than 2,000 ml from the collection bag. One hour later, she finds the collection bag full again. The nurse notifies the physician, who suspects that a bladder rupture is allowing the drainage of peritoneal fluid. The physician orders a urinalysis to be obtained immediately. The presence of which substance is considered abnormal?

Albumin Hinkle, J.L., & Cheever, K.H., Brunner & Suddarth's Textbook of Medical-Surgical Nursing, 14th ed., Philadelphia, Wolters Kluwer, 2018, Chapter 49: Assessment and Management of Patients With Hepatic Disorders, Ascites, p. 1385.

Which term is used to describe a chronic liver disease in which scar tissue surrounds the portal areas?

Alcoholic cirrhosis Chapter 49: Assessment and Management of Patients With Hepatic Disorders - Page 1406 This type of cirrhosis is due to chronic alcoholism and is the most common type of cirrhosis. In postnecrotic cirrhosis, there are broad bands of scar tissue, which are a late result of a previous acute viral hepatitis. In biliary cirrhosis, scarring occurs in the liver around the bile ducts. Compensated cirrhosis is a general term given to the state of liver disease in which the liver continues to be able to function effectively.

Management of a patient with ascites includes nutritional modifications and diuretic therapy. Which of the following interventions would a nurse expect to be part of patient care? Select all that apply.

Aldactone, an aldosterone-blocking agent would be used. Daily salt intake would be restricted to 2 grams or less. The diuretic will be held if the serum sodium level decreases to <134 m Eq/L. Hinkle, J.L., & Cheever, K.H., Brunner & Suddarth's Textbook of Medical-Surgical Nursing, 14th ed., Philadelphia, Wolters Kluwer, 2018, Chapter 50: Assessment and Management of Patients With Biliary Disorders, Monitoring and Managing Potential Complications, p. 1444.

What concepts does the nurse understand about gerontologic considerations related to acute pancreatitis? Select all that apply.

As the client ages, there is an increased mortality rate for acute pancreatitis. As the client ages, there is an increased risk for the development of multiple organ dysfunction syndrome. As the client ages, the pattern of complications related to acute pancreatitis changes.

A client with obesity is suspected of having nonalcoholic fatty liver disease. Which diagnostic labs does the nurse anticipate the client needing? Select all that apply.

Aspartate aminotransferase Alanine aminotransferase Hinkle, J.L., & Cheever, K.H., Brunner & Suddarth's Textbook of Medical-Surgical Nursing, 14th ed., Philadelphia, Wolters Kluwer, 2018, Chapter 48: Assessment and Management of Patients with Obesity, p. 1362.

A client who underwent abdominal surgery and has a nasogastric (NG) tube in place begins to complain of abdominal pain that he describes as "feeling full and uncomfortable." Which assessment should the nurse perform first?

Assess patency of the NG tube.

When caring for a client with an acute exacerbation of a peptic ulcer, the nurse finds the client doubled up in bed with severe pain in the right shoulder. What is the initial appropriate action by the nurse?

Assess the client's abdomen and vital signs.

A client with achalasia recently underwent pneumatic dilation. The nurse intervenes after the procedure by

Assessing lung sounds

Which intervention should be included in the plan of care for a client who has undergone a cholecystectomy?

Assessing the color of the sclera every shift Hinkle, J.L., & Cheever, K.H., Brunner & Suddarth's Textbook of Medical-Surgical Nursing, 14th ed., Philadelphia, Wolters Kluwer, 2018, Chapter 50: Assessment and Management of Patients With Biliary Disorders, p. 1439.

The nurse is planning care for a client following an incisional cholecystectomy for cholelithiasis. Which intervention is the highest nursing priority for this client?

Assisting the client to turn, cough, and deep breathe every 2 hours Hinkle, J.L., & Cheever, K.H., Brunner & Suddarth's Textbook of Medical-Surgical Nursing, 14th ed., Philadelphia, Wolters Kluwer, 2018, Chapter 50: Assessment and Management of Patients With Biliary Disorders, p. 1438.

Which of the following terms describes the involuntary flapping movements of the hands associated with metabolic liver dysfunction?

Asterixis Hinkle, J.L., & Cheever, K.H., Brunner & Suddarth's Textbook of Medical-Surgical Nursing, 14th ed., Philadelphia, Wolters Kluwer, 2018, Chapter 49: Assessment and Management of Patients With Hepatic Disorders, Physical Assessment, p. 1380.

A client is receiving a parenteral nutrition admixture that contains carbohydrates, electrolytes, vitamins, trace minerals, and sterile water and is now scheduled to receive an intravenous fat emulsion (Intralipid). What is the best action by the nurse?

Attaches the fat emulsion tubing to a Y connector close to the infusion site Chapter 44: Digestive and Gastrointestinal Treatment Modalities - Page 1257 An intravenous fat emulsion is attached to a Y connector close to the infusion site. The fat emulsion is administered simultaneously with the parenteral nutrition admixture. A separate peripheral IV site is not necessary. The fat emulsion is not administered through a filter.

A nonresponsive client has a nasogastric tube to low intermittent suction due to gastrointestinal bleeding. It is most important for the nurse to

Auscultate lung sounds every 4 hours.

Health teaching for a patient with GERD is directed toward decreasing lower esophageal sphincter pressure and irritation. The nurse instructs the patient to do which of the following? Select all that apply.

Avoid beer, especially in the evening. Elevate the head of the bed on 6- to 8-inch blocks. Elevate the upper body on pillows.

A nurse prepares nutrition education for a client who will undergo bariatric surgery. What nutrition suggestion best indicates a beneficial effect on the number and quality of bowel movements the client may have after surgery?

Avoid high-fat foods Hinkle, J.L., & Cheever, K.H., Brunner & Suddarth's Textbook of Medical-Surgical Nursing, 14th ed., Philadelphia, Wolters Kluwer, 2018, Chapter 48: Assessment and Management of Patients with Obesity, p. 1371.

A white female client is admitted to an acute care facility with a diagnosis of stroke. Her history reveals bronchial asthma, exogenous obesity, and iron-deficiency anemia. Which history finding is a risk factor for stroke?

Being obese Hinkle, J.L., & Cheever, K.H., Brunner & Suddarth's Textbook of Medical-Surgical Nursing, 14th ed., Philadelphia, Wolters Kluwer, 2018, Chapter 48: Assessment and Management of Patients with Obesity, Chart 67-2, p. 2014.

A client who had developed jaundice 2 months earlier is brought to the ED after attending a party and developing excruciating pain that radiated over the abdomen and into the back. Upon assessment, which additional symptom would the nurse expect this client to have?

Bile-stained vomiting Hinkle, J.L., & Cheever, K.H., Brunner & Suddarth's Textbook of Medical-Surgical Nursing, 14th ed., Philadelphia, Wolters Kluwer, 2018, Chapter 50: Assessment and Management of Patients With Biliary Disorders, p. 1442.

A client is admitted to the health care facility with a diagnosis of a bleeding gastric ulcer. The nurse expects the client's stools to have which description?

Black and tarry

The nurse is assessing a client with a bleeding gastric ulcer. When examining the client's stool, which characteristic would the nurse be most likely to find?

Black and tarry appearance

The nurse is assessing a client with advanced gastric cancer. The nurse anticipates that the assessment will reveal which finding?

Bloating after meals

The nurse is caring for a client with a history of bulimia. The client complains of retrosternal pain and dysphagia after forcibly causing herself to vomit after a large meal. The nurse suspects which condition?

Boerhaave syndrome

The nurse is performing and documenting the findings of an abdominal assessment. When the nurse hears intestinal rumbling and the client then experiences diarrhea, the nurse documents the presence of which condition?

Borborygmus

In women, which of the following types of cancer exceeds colorectal cancer?

Breast

The nurse cares for a client with obesity and discusses the increased risk of certain cancers related to obesity. Which cancers will the nurse include in the teaching? Select all that apply.

Breast Colorectal Cervical

A client has been receiving radiation therapy to the lungs and now has erythema, edema, and pain of the mouth. What instruction will the nurse give to the client?

Brush and floss daily.

A nurse inspects the Stensen duct of the parotid gland to determine inflammation and possible obstruction. What area in the oral cavity would the nurse examine?

Buccal mucosa next to the upper molars

The nurse is caring for a client with a biliary disorder who has an elevated amylase level. If this elevation correlates to dysfunction, which body process does the nurse recognize may be impaired?

Carbohydrate digestion Hinkle, J.L., & Cheever, K.H., Brunner & Suddarth's Textbook of Medical-Surgical Nursing, 14th ed., Philadelphia, Wolters Kluwer, 2018, Chapter 50: Assessment and Management of Patients With Biliary Disorders, The Exocrine Pancreas, p. 1429.

The nurse is talking with a group of clients who are older than age 50 years about the recognition of colon cancer to access early intervention. What should the nurse inform the clients to report immediately to their primary care provider?

Change in bowel habits

When caring for a client with cirrhosis, which symptom(s) should the nurse report immediately?

Change in mental status Hinkle, J.L., & Cheever, K.H., Brunner & Suddarth's Textbook of Medical-Surgical Nursing, 14th ed., Philadelphia, Wolters Kluwer, 2018, Chapter 49: Assessment and Management of Patients With Hepatic Disorders, p. 1407.

When caring for a client with advanced cirrhosis and hepatic encephalopathy, which assessment finding should the nurse report immediately?

Change in the client's handwriting and/or cognitive performance Hinkle, J.L., & Cheever, K.H., Brunner & Suddarth's Textbook of Medical-Surgical Nursing, 14th ed., Philadelphia, Wolters Kluwer, 2018, Chapter 49: Assessment and Management of Patients With Hepatic Disorders, p. 1395.

A nurse discusses risk factors of cholelithiasis with a client. Which risk factors will the nurse include in the teaching? Select all that apply.

Changes in weight Cystic fibrosis Diabetes Obesity Hinkle, J.L., & Cheever, K.H., Brunner & Suddarth's Textbook of Medical-Surgical Nursing, 14th ed., Philadelphia, Wolters Kluwer, 2018, Chapter 50: Assessment and Management of Patients With Biliary Disorders, p. 1431.

A nurse is providing home care to a client receiving intermittent tube feedings. The client wants to take an over-the-counter allergy medication. The medication would need to be given via feeding tube because the client has difficulty swallowing. The nurse checks the medication and finds that it is a timed-release tablet. Which action by the nurse would be most appropriate?

Check with the pharmacy for an alternative formulation for the drug.

When examining the skin of a client who is dehydrated due to fluid losses from the gastrointestinal tract, which of the following would be most important?

Checking if the mucous membranes are dry

Which foods should be avoided following acute gallbladder inflammation?

Cheese Hinkle, J.L., & Cheever, K.H., Brunner & Suddarth's Textbook of Medical-Surgical Nursing, 14th ed., Philadelphia, Wolters Kluwer, 2018, Chapter 50: Assessment and Management of Patients With Biliary Disorders, p. 1434.

A client with gallstones tells the nurse, "The doctor has to do something. Isn't there something he can give me to dissolve them?" What medication does the nurse know may help dissolve the gallstones?

Chenodiol Hinkle, J.L., & Cheever, K.H., Brunner & Suddarth's Textbook of Medical-Surgical Nursing, 14th ed., Philadelphia, Wolters Kluwer, 2018, Chapter 49: Assessment and Management of Patients With Hepatic Disorders, Pharmacologic Therapy, p. 1434.

Which term most precisely refers to the incision of the common bile duct for removal of stones?

Choledocholithotomy Hinkle, J.L., and Cheever, K.H. Brunner & Suddarth's Textbook of Medical-Surgical Nursing, 14th ed. Philadelphia: Lippincott Williams & Wilkins, 2017, Chapter 50: Cholecystitis

A very ill client with acute cholecystitis is scheduled for surgery. The surgeon plans to create an incision in the common bile duct to remove stones. The nurse correctly documents this surgery in the electronic medical record using which term?

Choledochostomy Hinkle, J.L., & Cheever, K.H., Brunner & Suddarth's Textbook of Medical-Surgical Nursing, 14th ed., Philadelphia, Wolters Kluwer, 2018, Chapter 50: Assessment and Management of Patients With Biliary Disorders, p. 1437.

A client comes to the clinic and informs the nurse that he is there to see the physician for right upper abdominal discomfort, nausea, and frequent belching especially after eating a meal high in fat. What disorder do these symptoms correlate with?

Cholelithiasis Chapter 49: Assessment and Management of Patients With Hepatic Disorders - Page 1431 Initially, with cholelithiasis clients experience belching, nausea, and right upper quadrant discomfort, with pain or cramps after high-fat meal. Symptoms become acute when a stone blocks bile flow from the gallbladder. With acute cholecystitis, clients usually are very sick with fever, vomiting, tenderness over the liver, and severe pain called biliary colic. The symptoms do not correlate with hepatitis.

A client informs the nurse that he has been having abdominal pain that is relieved when having a bowel movement. The client states that the physician told him he has irritable bowel syndrome. What does the nurse recognize as characteristic of this disorder?

Chronic constipation with sporadic bouts of diarrhea

Which is an accurate statement regarding cancer of the esophagus?

Chronic irritation of the esophagus is a known risk factor.

The nurse is irrigating a client's colostomy when the client begins to report cramping. What is the appropriate action by the nurse?

Clamp the tubing and allow client to rest.

The nurse is irrigating a colostomy when the patient says, "You will have to stop, I am cramping so badly." What is the priority action by the nurse?

Clamp the tubing and give the patient a rest period.

A patient is admitted to the hospital with a possible common bile duct obstruction. What clinical manifestations does the nurse understand are indicators of this problem? (Select all that apply.)

Clay-colored feces Pruritus Jaundice Hinkle, J.L., & Cheever, K.H., Brunner & Suddarth's Textbook of Medical-Surgical Nursing, 14th ed., Philadelphia, Wolters Kluwer, 2018, Chapter 50: Assessment and Management of Patients With Biliary Disorders, Clinical Manifestations, p. 1452.

A nurse is caring for a client with cholelithiasis. Which sign indicates obstructive jaundice?

Clay-colored stools Hinkle, J.L., & Cheever, K.H., Brunner & Suddarth's Textbook of Medical-Surgical Nursing, 14th ed., Philadelphia, Wolters Kluwer, 2018, Chapter 49: Assessment and Management of Patients With Hepatic Disorders, Obstructive Jaundice, p. 1385.

Which is a clinical manifestation of cholelithiasis?

Clay-colored stools Hinkle, J.L., & Cheever, K.H., Brunner & Suddarth's Textbook of Medical-Surgical Nursing, 14th ed., Philadelphia, Wolters Kluwer, 2018, Chapter 50: Assessment and Management of Patients With Biliary Disorders, p. 1432.

A client has a tumor of the head of the pancreas. What clinical manifestations will the nurse assess? Select all that apply.

Clay-colored stools Dark urine Jaundice Hinkle, J.L., & Cheever, K.H., Brunner & Suddarth's Textbook of Medical-Surgical Nursing, 14th ed., Philadelphia, Wolters Kluwer, 2018, Chapter 50: Assessment and Management of Patients With Biliary Disorders, Clinical Manifestations, p. 1452.

Sixty to eighty percent of pancreatic tumors occur in the head of the pancreas. Tumors in this region obstruct the common bile duct. Which of the following clinical manifestations would indicate a common bile duct obstruction associated with a tumor in the head of the pancreas? Choose all that apply.

Clay-colored stools Dark urine Jaundice Pruritis Hinkle, J.L., & Cheever, K.H., Brunner & Suddarth's Textbook of Medical-Surgical Nursing, 14th ed., Philadelphia, Wolters Kluwer, 2018.

A nurse is caring for a client with a long-term central venous catheter. Which care principle is correct?

Clean the port with an alcohol pad before administering I.V. fluid through the catheter.

A nurse is planning care for a client who will undergo bariatric surgery in a week. What goals are acceptable during this point in the client's care? Select all that apply.

Client will become knowledgeable about the procedure. Client will understand preoperative and postoperative dietary restrictions. Client will have decreased anxiety about the procedure.

Which clients will the nurse recognize as being at high risk for complication if taking Lorcaserin for obesity? Select all that apply.

Client with type 2 diabetes Client taking medication for migraines Client taking medication for depression Hinkle, J.L., & Cheever, K.H., Brunner & Suddarth's Textbook of Medical-Surgical Nursing, 14th ed., Philadelphia, Wolters Kluwer, 2018, Chapter 48: Assessment and Management of Patients with Obesity, p. 1365.

A client is scheduled for several diagnostic tests to evaluate gastrointestinal function. After teaching the client about these tests, the nurse determines that the client has understood the teaching when the client identifies which test as not requiring the use of a contrast medium?

Colonoscopy

A nurse cares for a client with a BMI of 40 kg/m2. Which gastrointestinal condition or disease does the nurse recognize is associated with the client's BMI?

Colorectal cancer

Which statement provides accurate information regarding cancer of the colon and rectum?

Colorectal cancer is the third most common site of cancer in the United States.

A client has a nasogastric tube for continuous tube feeding. The nurse does all the following every shift to verify placement (select all options that apply):

Compares exposed tube length with original measurement Visually assesses the color of the aspirate Checks the pH of the gastric contents

The nurse plans care for a client with obesity. What does the nurse recognize is the primary pathophysiological reason clients with obesity are at greater risk for developing thromboembolism?

Compromised peripheral blood flow

A nurse administered a full strength feeding with an increased osmolality through a jejunostomy tube to a client. Immediately following the feeding, the client expelled a large amount of liquid brown stool and exhibited a blood pressure of 86/58 and pulse rate of 112 beats/min. The nurse

Consults with the physician about decreasing the feeding to half-strength Chapter 44: Digestive and Gastrointestinal Treatment Modalities - Page 1248 The osmolality of normal body fluids is 300 mOsm/kg. A feeding with a higher osmolality may cause dumping syndrome. The client may report a feeling of fullness, nausea, or both and may exhibit diarrhea, hypotension, and tachycardia. The nurse needs to take steps to prevent dumping syndrome. Increasing the amount of the feeding, administering the feeding at an extreme temperature, or increasing the osmolality of the feedings will continue dumping syndrome. The nurse needs to decrease the osmolality of the feeding as in administering a half-strength solution.

A nurse providing care to a patient who is receiving nasogastric tube feedings finds that the tube is clogged. Which of the following would be least appropriate to use to unclog the tube?

Cranberry juice Chapter 44: Digestive and Gastrointestinal Treatment Modalities - Page 1246 To unclog a feeding tube, air insufflation, digestive enzymes mixed with warm water, or a commercial enzyme product could be used. Cola and cranberry juice are no longer advocated for use in clearing a clogged tube.

A client presents with an infection in the area between the internal and external sphincters. In which chronic disease is this condition commonly seen?

Crohn's disease

A client with a 10-year history of Crohn's disease is seeing the physician due to increased diarrhea and fatigue. Additionally, the client has developed arthritis and conjunctivitis. What is the most likely cause of the latest symptoms?

Crohn's disease

A client sustained second- and third-degree burns over 30% of the body surface area approximately 72 hours ago. What type of ulcer should the nurse be alert for while caring for this client?

Curling's ulcer

A client with obesity reports taking orlistat in order to aid in weight loss. Which medication order for the client will the nurse question?

Cyclosporine Hinkle, J.L., & Cheever, K.H., Brunner & Suddarth's Textbook of Medical-Surgical Nursing, 14th ed., Philadelphia, Wolters Kluwer, 2018, Chapter 48: Assessment and Management of Patients with Obesity, p. 1365.

A nurse is caring for a client receiving parenteral nutrition at home. The client was discharged from the acute care facility 4 days ago. What would the nurse include in the client's plan of care? Select all that apply.

Daily weights Intake and output monitoring Calorie counts for oral nutrients

An older adult client seeks help for chronic constipation. What factor related to aging can cause constipation in elderly clients?

Decreased abdominal strength

Which is an age-related change of the hepatobiliary system?

Decreased blood flow Hinkle, J.L., & Cheever, K.H., Brunner & Suddarth's Textbook of Medical-Surgical Nursing, 14th ed., Philadelphia, Wolters Kluwer, 2018, Chapter 49: Assessment and Management of Patients With Hepatic Disorders, Chart 49-1, p. 1380.

A 70-year-old client is admitted with acute pancreatitis. The nurse understands that the mortality rate associated with acute pancreatitis increases with advanced age and attributes this to which gerontologic consideration associated with the pancreas?

Decreases in the physiologic function of major organs Hinkle, J.L., & Cheever, K.H., Brunner & Suddarth's Textbook of Medical-Surgical Nursing, 14th ed., Philadelphia, Wolters Kluwer, 2018, Chapter 50: Assessment and Management of Patients With Biliary Disorders, p. 1441.

The nurse is preparing a client for a test that involves inserting a thick barium paste into the rectum with radiographs taken as the client expels the barium. What test will the nurse prepare the client for?

Defecography

What is the most appropriate nursing diagnosis for the client with acute pancreatitis?

Deficient fluid volume Hinkle, J.L., & Cheever, K.H., Brunner & Suddarth's Textbook of Medical-Surgical Nursing, 14th ed., Philadelphia, Wolters Kluwer, 2018, Chapter 50: Assessment and Management of Patients With Biliary Disorders, Intensive Care, p. 1442.

An obese Hispanic client, age 65, is diagnosed with type 2 diabetes. Which statement about diabetes mellitus is true?

Diabetes mellitus is more common in Hispanics and Blacks than in Whites. Hinkle, J.L., & Cheever, K.H., Brunner & Suddarth's Textbook of Medical-Surgical Nursing, 14th ed., Philadelphia, Wolters Kluwer, 2018, Chapter 48: Assessment and Management of Patients with Obesity, Figure 48-1, p. 1359.

A client with a gastrojejunostomy is beginning to take solid food. Which finding would lead the nurse to suspect that the client is experiencing dumping syndrome?

Diarrhea

Which is one of the primary symptoms of irritable bowel syndrome (IBS)?

Diarrhea

What would the nurse identify as a characteristic finding when assessing a client for pilonidal sinus?

Dilated pits of hair follicles in the cleft

A nurse cares for a client who is 36 hours postoperative from bariatric surgery and is suspected of having a hemorrhage related to the procedure. What is the nurse's understanding of the most likely cause of the hemorrhage?

Disruption of a staple Hinkle, J.L., & Cheever, K.H., Brunner & Suddarth's Textbook of Medical-Surgical Nursing, 14th ed., Philadelphia, Wolters Kluwer, 2018, Chapter 48: Assessment and Management of Patients with Obesity, p. 1372.

What information should the nurse include in the teaching plan for a client being treated for diverticulosis?

Drink at least 8 to 10 large glasses of fluid every day

The nurse is teaching a client with an ostomy how to change the pouching system. Which information should the nurse include when teaching a client with no peristomal skin irritation?

Dry skin thoroughly after washing

A client who had a Roux-en-Y bypass procedure for morbid obesity ate a chocolate chip cookie after a meal. After ingestion of the cookie, the client reported cramping pains, dizziness, and palpitation. After having a bowel movement, the symptoms resolved. What should the nurse educate the client about regarding this event?

Dumping syndrome

A nurse is providing follow-up teaching at a clinic visit for a client recovering from gastric resection. The client reports sweating, diarrhea, nausea, palpitations, and the desire to lie down 15 to 30 minutes after meals. Based on the client's assessment, what will the nurse suspect?

Dumping syndrome

The nursing student approaches his instructor to discuss the plan of care for his client diagnosed with peptic ulcer disease. The student asks what is the most common site for peptic ulcer formation? The instructor would state which one of the following?

Duodenum

A nurse is assessing a client for GI dysfunction. What is the most common symptom in a client with GI dysfunction?

Dyspepsia Chapter 43: Assessment of Digestive and Gastrointestinal Function - Page 1229 Dyspepsia is a condition that usually involves a combination of symptoms: abdominal pain, bloating, distention, nausea, and belching. Dyspepsia refers to altered digestion that is not associated with a pathologic condition.

The nurse is performing an assessment of a patient. During the assessment the patient informs the nurse of some recent "stomach trouble." What does the nurse know is the most common symptom of patients with GI dysfunction?

Dyspepsia Chapter 43: Assessment of Digestive and Gastrointestinal Function - Page 1229 Dyspepsia, upper abdominal discomfort associated with eating (commonly called indigestion), is the most common symptom of patients with GI dysfunction. Indigestion is an imprecise term that refers to a host of upper abdominal or epigastric symptoms such as pain, discomfort, fullness, bloating, early satiety, belching, heartburn, or regurgitation; it occurs in approximately 25% of the adult population (Harmon & Peura, 2010).

The nurse cares for a client with gallstones that need to be removed but is not a surgical candidate or endoscopic candidate. What procedure does the nurse recognize as being a possible treatment option for the client?

ESWL Hinkle, J.L., & Cheever, K.H., Brunner & Suddarth's Textbook of Medical-Surgical Nursing, 14th ed., Philadelphia, Wolters Kluwer, 2018, Chapter 50: Assessment and Management of Patients With Biliary Disorders, p. 1435.

A nurse has admitted a client suspected of having acute pancreatitis. The nurse knows that mild acute pancreatitis is characterized by:

Edema and inflammation Hinkle, J.L., & Cheever, K.H., Brunner & Suddarth's Textbook of Medical-Surgical Nursing, 14th ed., Philadelphia, Wolters Kluwer, 2018, Chapter 50: Assessment and Management of Patients With Biliary Disorders, Clinical Manifestations, p. 1441.

A nurse is caring for a client in the emergency department who is reporting severe abdominal pain. The client is diagnosed with acute pancreatitis. Which laboratory value indicates further investigation?

Elevated troponin Hinkle, J.L., & Cheever, K.H., Brunner & Suddarth's Textbook of Medical-Surgical Nursing, 14th ed., Philadelphia, Wolters Kluwer, 2018, Chapter 50: Assessment and Management of Patients With Biliary Disorders, Chart 50-3, p. 1441.

A client is admitted to the healthcare facility suspected of having acute pancreatitis and undergoes laboratory testing. Which of the following would the nurse expect to find?

Elevated urine amylase levels Hinkle, J.L., & Cheever, K.H., Brunner & Suddarth's Textbook of Medical-Surgical Nursing, 14th ed., Philadelphia, Wolters Kluwer, 2018, Chapter 50: Assessment and Management of Patients With Biliary Disorders, Assessment and Diagnostic Findings, p. 1442.

Which dietary modification is used for a client diagnosed with acute pancreatitis?

Elimination of coffee Hinkle, J.L., & Cheever, K.H., Brunner & Suddarth's Textbook of Medical-Surgical Nursing, 14th ed., Philadelphia, Wolters Kluwer, 2018, Chapter 50: Assessment and Management of Patients With Biliary Disorders, Chart 50-4, p. 1447.

A nursing student has learned about many collaborative interventions to achieve pain relief for clients with acute pancreatitis. Which of the following are appropriate? Choose all that apply.

Encourage bed rest to decrease the client's metabolic rate. Teach the client about the correlation between alcohol intake and pain. Withhold oral feedings to limit the release of secretin. Hinkle, J.L., & Cheever, K.H., Brunner & Suddarth's Textbook of Medical-Surgical Nursing, 14th ed., Philadelphia, Wolters Kluwer, 2018, Chapter 50: Assessment and Management of Patients With Biliary Disorders, Pain Management, p. 1442.

A client has been recently diagnosed with an anorectal condition. The nurse is reviewing interventions that will assist the client with managing the therapeutic regimen. What would not be included?

Encourage the client to avoid exercise.

When caring for the patient with acute pancreatitis, the nurse must consider pain relief measures. What nursing interventions could the nurse provide? (Select all that apply.)

Encouraging bed rest to decrease the metabolic rate Withholding oral feedings to limit the release of secretin Administering parenteral opioid analgesics as ordered Hinkle, J.L., & Cheever, K.H., Brunner & Suddarth's Textbook of Medical-Surgical Nursing, 14th ed., Philadelphia, Wolters Kluwer, 2018, Chapter 50: Assessment and Management of Patients With Biliary Disorders, Nursing Management, p. 1443.

A nurse caring for a client who is post op bariatric surgery notes that the client has no bowel sounds and suspects the client may have a bowel obstruction. What treatment or therapy does the nurse expect the client will require?

Endoscopy Hinkle, J.L., & Cheever, K.H., Brunner & Suddarth's Textbook of Medical-Surgical Nursing, 14th ed., Philadelphia, Wolters Kluwer, 2018, Chapter 48: Assessment and Management of Patients with Obesity, p. 1373.

A client is given a diagnosis of hepatic cirrhosis. The client asks the nurse what findings led to this determination. Which of the following clinical manifestations would the nurse correctly identify? Select all that apply.

Enlarged liver size Ascites Hemorrhoids Chapter 49: Assessment and Management of Patients With Hepatic Disorders - Page 1381 Early in the course of cirrhosis, the liver tends to be large, and the cells are loaded with fat. The liver is firm and has a sharp edge that is noticeable on palpation. Portal obstruction and ascites, late manifestations of cirrhosis, are caused partly by chronic failure of liver function and partly by obstruction of the portal circulation. The obstruction to blood flow through the liver caused by fibrotic changes also results in the formation of collateral blood vessels in the GI system and shunting of blood from the portal vessels into blood vessels with lower pressures. These distended blood vessels form varices or hemorrhoids, depending on their location. Because of inadequate formation, use, and storage of certain vitamins (notably vitamins A, C, and K), signs of deficiency are common, particularly hemorrhagic phenomena associated with vitamin K deficiency. Additional clinical manifestations include deterioration of mental and cognitive function with impending hepatic encephalopathy and hepatic coma, as previously described.

The nurse is investigating a client's report of pain in the duodenal area. Where should the nurse perform the assessment?

Epigastric area and consider possible radiation of pain to the right subscapular region Chapter 43: Assessment of Digestive and Gastrointestinal Function - Page 1231 Indigestion is an imprecise term that refers to a host of upper abdominal or epigastric symptoms such as pain, discomfort, fullness, bloating, early satiety, belching, heartburn, or regurgitation; it occurs in approximately 25% of the adult population (Harmon & Peura, 2010).

An elderly client seeks medical attention for a vague complaint of difficulty swallowing. Which of the following assessment findings is most significant as related to this symptom?

Esophageal tumor

The nurse is working in a diagnostic testing unit focusing on gastrointestinal studies. For which testing procedure is the nurse correct to assess the gag reflex before offering fluids?

Esophagogastroduodenoscopy

Cancer of the esophagus is most often diagnosed by which of the following?

Esophagogastroduodenoscopy (EGD) with biopsy and brushings

The nurse is providing community education at the mall. The nurse is instructing on the muscular tube that connects the mouth to the stomach. The nurse outlines this structure on a drawing and labels it with which of the following?

Esophagus

As a nurse completes the admission assessment of a client admitted for gastric bypass surgery, the client states, "Finally! I'll be thin and able to eat without much concern." How should the nurse intervene?

Evaluate the client's understanding of the procedure.

A client has been brought into the ED via ambulance, reporting acute generalized abdominal pain, nausea, fever, and constipation. The healthcare provider suspects appendicitis, but testing has not been performed yet to make a definitive diagnosis. What will the nurse most likely do while initially caring for this client?

Explain to the client why analgesics are being withheld.

When assessing whether a client is a candidate for home parenteral nutrition, what would be important to address? Select all that apply.

Family support Telephone access Motivation for learning Health status

A client has been taking famotidine at home. What teaching should the nurse include with the client?

Famotidine will inhibit gastric acid secretions.

Place the physiological processes in order that are involved in feelings of satiety and hunger.

Fasting Release of ghrelin and neuropeptide y Stimulation of appetite through the CNS Eating behaviors signaled Hinkle, J.L., & Cheever, K.H., Brunner & Suddarth's Textbook of Medical-Surgical Nursing, 14th ed., Philadelphia, Wolters Kluwer, 2018, Chapter 48: Assessment and Management of Patients with Obesity, p. 1360.

The nurse is caring for a client with a suspected megacolon. The nurse anticipates that one of the findings of assessment will be

Fecal incontinence

The mode of transmission of hepatitis A virus (HAV) includes which of the following?

Fecal-oral Hinkle, J.L., & Cheever, K.H., Brunner & Suddarth's Textbook of Medical-Surgical Nursing, 14th ed., Philadelphia, Wolters Kluwer, 2018, Chapter 49: Assessment and Management of Patients With Hepatic Disorders, Hepatitis A Virus, p. 1398.

The nurse in an extended-care facility reports that a resident has clinical manifestations of fecal incontinence. The health care provider orders a diagnostic study to rule out inflammation. Which study will the nurse prepare the client for?

Flexible sigmoidoscopy

The nurse assesses a patient who recently had a nasoenteric intubation. Symptoms of oliguria, lethargy, and tachycardia in the patient would indicate to the nurse what common complication?

Fluid volume deficit

The nurse is to discontinue a nasogastric tube that had been used for decompression. What is the first action the nurse should take?

Flush with 10 mL of water.

Which type of deficiency results in macrocytic anemia?

Folic acid Hinkle, J.L., & Cheever, K.H., Brunner & Suddarth's Textbook of Medical-Surgical Nursing, 14th ed., Philadelphia, Wolters Kluwer, 2018, Chapter 49: Assessment and Management of Patients With Hepatic Disorders, Vitamin Deficiency, p. 1397.

The nurse teaches a client scheduled for a colonoscopy. Which instruction should be included as part of the preparation for the procedure?

Follow the dietary and fluid restrictions and bowel preparation procedures.

Which of the following is a function of the stomach? Select all that apply.

Food storage Secretion of digestive fluids Propels partially digested food into small intestine

When the nurse is caring for a patient with acute pancreatitis, what intervention can be provided in order to prevent atelectasis and prevent pooling of respiratory secretions?

Frequent changes of positions Hinkle, J.L., & Cheever, K.H., Brunner & Suddarth's Textbook of Medical-Surgical Nursing, 14th ed., Philadelphia, Wolters Kluwer, 2018, Chapter 50: Assessment and Management of Patients With Biliary Disorders, Improving Breathing Pattern, p. 1443.

Which condition in a client with pancreatitis makes it necessary for the nurse to check fluid intake and output, check hourly urine output, and monitor electrolyte levels?

Frequent vomiting, leading to loss of fluid volume Hinkle, J.L., & Cheever, K.H., Brunner & Suddarth's Textbook of Medical-Surgical Nursing, 14th ed., Philadelphia, Wolters Kluwer, 2018, Chapter 50: Assessment and Management of Patients With Biliary Disorders, p. 1444.

The nurse is developing a plan of care for a patient with peptic ulcer disease. What nursing interventions should be included in the care plan? Select all that apply.

Frequently monitoring hemoglobin and hematocrit levels Observing stools and vomitus for color, consistency, and volume Checking the blood pressure and pulse rate every 15 to 20 minutes

A few hours after eating hot and spicy chicken wings, a client presents with lower chest pain. He wonders if he is having a heart attack. How should the nurse proceed first?

Further investigate the initial complaint.

A nurse cares for a client with obesity. Which medication that the client takes may be contributing to the client's obesity?

Gabapentin

A nurse is preparing a client for surgery. During preoperative teaching, the client asks where is bile stored. The nurse knows that bile is stored in the:

Gallbladder Hinkle, J.L., & Cheever, K.H., Brunner & Suddarth's Textbook of Medical-Surgical Nursing, 14th ed., Philadelphia, Wolters Kluwer, 2018, Chapter 50: Assessment and Management of Patients With Biliary Disorders, The Gallbladder, p. 1428.

A nurse teaches a client with obesity on the various types of bariatric surgical procedures. Which procedures will the nurse include when discussing restrictive procedures only? Select all that apply.

Gastric banding Sleeve gastrectomy Hinkle, J.L., & Cheever, K.H., Brunner & Suddarth's Textbook of Medical-Surgical Nursing, 14th ed., Philadelphia, Wolters Kluwer, 2018, Chapter 48: Assessment and Management of Patients with Obesity, p. 1369.

A client who is postoperative bariatric surgery is diagnosed with bile reflux. Which conditions are associated with bile reflux? Select all that apply.

Gastritis Esophagitis

A client has a gastrointestinal tube that enters the stomach through a surgically created opening in the abdominal wall. The nurse documents this as which of the following?

Gastrostomy tube

A nurse teaches a client homecare nutrition expectations who will undergo bariatric surgery. Which foods will the nurse suggest the client keep on hand at home after surgery? Select all that apply.

Gelatin Pudding Fat-free milk

During a psychotic episode, a client with schizophrenia swallows a small wooden spoon. Which medication would the nurse in the emergency department be most likely to administer to facilitate removal of the foreign body?

Glucagon

A nursing student is reviewing for an upcoming anatomy and physiology examination. Which of the following would the student correctly identify as a function of the liver? Select all that apply.

Glucose metabolism Ammonia conversion Protein metabolism Chapter 49: Assessment and Management of Patients With Hepatic Disorders - Page 1379 Functions of the liver include the metabolism of glucose, protein, fat, and drugs; conversion of ammonia; storage of vitamins and iron; formation of bile; and excretion of bilirubin. The liver is not responsible for the metabolism of carbohydrates or the storage of zinc.

The nurse identifies which type of jaundice in an adult experiencing a transfusion reaction?

Hemolytic Hinkle, J.L., & Cheever, K.H., Brunner & Suddarth's Textbook of Medical-Surgical Nursing, 14th ed., Philadelphia, Wolters Kluwer, 2018, Chapter 49: Assessment and Management of Patients With Hepatic Disorders, p. 1384.

Which type of jaundice is the result of increased destruction of red blood cells?

Hemolytic Hinkle, J.L., & Cheever, K.H., Brunner & Suddarth's Textbook of Medical-Surgical Nursing, 14th ed., Philadelphia, Wolters Kluwer, 2018, Chapter 49: Assessment and Management of Patients With Hepatic Disorders, p. 1384.

Which type of jaundice seen in adults is the result of increased destruction of red blood cells?

Hemolytic Hinkle, J.L., & Cheever, K.H., Brunner & Suddarth's Textbook of Medical-Surgical Nursing, 14th ed., Philadelphia, Wolters Kluwer, 2018, Chapter 49: Assessment and Management of Patients With Hepatic Disorders, p. 1384.

A client with severe peptic ulcer disease has undergone surgery and is several hours postoperative. During assessment, the nurse notes that the client has developed cool skin, tachycardia, labored breathing, and appears to be confused. Which complication has the client most likely developed?

Hemorrhage

A client and spouse are visiting the clinic. The client recently experienced a seizure and says she has been having difficulty writing. Before the seizure, the client says that for several weeks she was sleeping late into the day but having restlessness and insomnia at night. The client's husband says that he has noticed the client has been moody and slightly confused. Which of the following problems is most consistent with the client's clinical manifestations?

Hepatic encephalopathy Hinkle, J.L., & Cheever, K.H., Brunner & Suddarth's Textbook of Medical-Surgical Nursing, 14th ed., Philadelphia, Wolters Kluwer, 2018, Chapter 49: Assessment and Management of Patients With Hepatic Disorders, Clinical Manifestations, p. 1394.

A client has an elevated serum ammonia concentration and is exhibiting changes in mental status. The nurse should suspect which condition?

Hepatic encephalopathy Hinkle, J.L., & Cheever, K.H., Brunner & Suddarth's Textbook of Medical-Surgical Nursing, 14th ed., Philadelphia, Wolters Kluwer, 2018, Chapter 49: Assessment and Management of Patients With Hepatic Disorders, p. 1394.

The nurse auscultates the abdomen to assess bowel sounds. She documents five to six sounds heard in less than 30 seconds. How does the nurse document the bowel sounds?

Hyperactive

A nurse is educating a community group about coronary artery disease. One member asks about how to avoid coronary artery disease. Which of the following items are considered modifiable risk factors for coronary artery disease? Choose all that apply.

Hyperlipidemia Obesity Tobacco use Hinkle, J.L., & Cheever, K.H., Brunner & Suddarth's Textbook of Medical-Surgical Nursing, 14th ed., Philadelphia, Wolters Kluwer, 2018, Chapter 27: Management of Patients with Coronary Vascular Disorders, Chart 27-1, p. 752.

A client who is post op from bariatric surgery experiences dumping syndrome. What is the nurse's understanding of the cause of this condition?

Hypertonic food releases metabolic peptides. Chapter 48: Assessment and Management of Patients with Obesity - Page 1373 The previous theory behind dumping syndrome was that the hypertonic food bolus drew extracellular fluid from the blood into the small intestines, causing symptoms. This theory has since been challenged and the newest theory suggests the rapid release of metabolic peptides that release from the hypertonic food bolus causes dumping syndrome symptoms.

Clients diagnosed with esophageal varices are at risk for hemorrhagic shock. Which is a sign of potential hypovolemia?

Hypotension Chapter 49: Assessment and Management of Patients With Hepatic Disorders - Page 1389 Signs of potential hypovolemia include cool, clammy skin; tachycardia; decreased blood pressure; and decreased urine output.

During assessment of a patient with gastritis, the nurse practitioner attempts to distinguish acute from chronic pathology. One criteria, characteristic of gastritis would be the:

Immediacy of the occurrence.

A student nurse is preparing a plan of care for a client with chronic pancreatitis. What nursing diagnosis related to the care of a client with chronic pancreatitis is the priority?

Impaired nutrition: less than body requirements Hinkle, J.L., & Cheever, K.H., Brunner & Suddarth's Textbook of Medical-Surgical Nursing, 14th ed., Philadelphia, Wolters Kluwer, 2018, Chapter 50: Assessment and Management of Patients With Biliary Disorders, Clinical Manifestations, p. 1445.

The nurse is admitting a client with traumatic injuries who also has class III obesity. When planning this client's care, the nurse should address the client's heightened risk of what nursing diagnoses related to obesity? Select all that apply.

Impaired skin integrity Impaired gas exchange Hinkle, J.L., & Cheever, K.H., Brunner & Suddarth's Textbook of Medical-Surgical Nursing, 14th ed., Philadelphia, Wolters Kluwer, 2018, Chapter 48: Assessment and Management of Patients with Obesity, Nursing Management, p. 1367.

The nurse is working on a general medical unit. A client is scheduled for an upper gastrointestinal series. Upon returning to the nursing unit, what does the nurse identify as the client goal?

Increase in the amount of fluids Chapter 43: Assessment of Digestive and Gastrointestinal Function - Page 1235 The client, returning from an upper gastrointestinal series, needs to increase fluids in an effort to eliminate the barium from the body through a bowel movement. General anesthesia is not used. The client typically does not have nausea and vomiting following the procedure. If the client is able to ambulate independently prior to the procedure, the client will be able to ambulate independently following.

A nurse is providing preoperative teaching to a client undergoing a cholecystectomy. Which topic should the nurse include in her teaching plan?

Increase respiratory effectiveness. Hinkle, J.L., & Cheever, K.H., Brunner & Suddarth's Textbook of Medical-Surgical Nursing, 14th ed., Philadelphia, Wolters Kluwer, 2018, Chapter 50: Assessment and Management of Patients With Biliary Disorders, Improving Respiratory Status, p. 1438.

The nurse is working on a general medical unit. A client is scheduled for an upper gastrointestinal series. Upon return to the nursing unit, what does the nurse identify as the client goal?

Increase the amount of fluids The client, returning from an upper gastrointestinal series, needs to increase fluids in an effort to eliminate the barium from the body through a bowel movement. General anesthesia is not used. The client typically does not have nausea and vomiting following the procedure. If the client is able to ambulate independently prior to the procedure, the client will be able to ambulate independently following.

Place the pathophysiological steps in order of how a client with obesity is at greater risk for venous thromboembolism in comparison to the general population.

Increased adipose tissue Impairment of peripheral blood flow Blood stasis Formation of a thrombus Hinkle, J.L., & Cheever, K.H., Brunner & Suddarth's Textbook of Medical-Surgical Nursing, 14th ed., Philadelphia, Wolters Kluwer, 2018, Chapter 48: Assessment and Management of Patients with Obesity, p. 1367.

Which is a gerontological consideration associated with the pancreas?

Increased amount of fibrous material Hinkle, J.L., & Cheever, K.H., Brunner & Suddarth's Textbook of Medical-Surgical Nursing, 14th ed., Philadelphia, Wolters Kluwer, 2018, Chapter 50: Assessment and Management of Patients With Biliary Disorders, p. 1430.

Which of the following symptoms would indicate that a client with chronic pancreatitis has developed secondary diabetes?

Increased appetite and thirst Hinkle, J.L., & Cheever, K.H., Brunner & Suddarth's Textbook of Medical-Surgical Nursing, 14th ed., Philadelphia, Wolters Kluwer, 2018, Chapter 49: Assessment and Management of Patients With Hepatic Disorders, Clinical Manifestations, p. 1445.

Place the pathophysiology of obesity in the correct order.

Increased body fat Adiposopathy Release of biochemical mediators Chronic inflammation Disease Hinkle, J.L., & Cheever, K.H., Brunner & Suddarth's Textbook of Medical-Surgical Nursing, 14th ed., Philadelphia, Wolters Kluwer, 2018, Chapter 48: Assessment and Management of Patients with Obesity, p. 1360.

Place the pathophysiological steps in order for the normal role of leptin as it relates to hunger and satiety.

Increased fat stores Increased leptin Increased satiety Inhibition of food intake Hinkle, J.L., & Cheever, K.H., Brunner & Suddarth's Textbook of Medical-Surgical Nursing, 14th ed., Philadelphia, Wolters Kluwer, 2018, Chapter 48: Assessment and Management of Patients with Obesity, p. 1360.

A client is being evaluated for esophageal cancer. What initial manifestation of esophageal cancer should the nurse assess?

Increasing difficulty in swallowing

A client is recovering from an ileostomy that was performed to treat inflammatory bowel disease. During discharge teaching, what would the nurse stress the importance of?

Increasing fluid intake to prevent dehydration

A nurse cares for clients with obesity and recognizes that clients with larger waist-to-hip ratios are at greater risk for obesity-related morbidity in comparison to others with smaller waist-to-hip ratios. What findings related to this does the nurse recognize as true? Select all that apply.

Individuals with larger waist-to-hip ratios have android obesity. Individuals with larger waist-to-hip ratios are also referred to as apple-shaped. Individuals with larger waist-to-hip ratios have larger visceral fat stores. Hinkle, J.L., & Cheever, K.H., Brunner & Suddarth's Textbook of Medical-Surgical Nursing, 14th ed., Philadelphia, Wolters Kluwer, 2018, Chapter 48: Assessment and Management of Patients with Obesity, p. 1362.

The nurse working with clients who have obesity understand that gut microbes perform numerous digestive, metabolic, and immunologic functions. What is the nurse's understanding of how these microbes contribute to obesity? Select all that apply.

Individuals with obesity have less diverse gut microbes than those of normal weight. Specific gut microbes have been linked to a leaner body shape. Hinkle, J.L., & Cheever, K.H., Brunner & Suddarth's Textbook of Medical-Surgical Nursing, 14th ed., Philadelphia, Wolters Kluwer, 2018, Chapter 48: Assessment and Management of Patients with Obesity, p. 1361.

A client has a radical neck dissection to treat cancer of the neck. The nurse develops the care plan and includes all the following diagnoses. The nurse identifies the highest priority diagnosis as

Ineffective airway clearance related to obstruction by mucus

Crohn's disease is a condition of malabsorption caused by which pathophysiological process?

Inflammation of all layers of intestinal mucosa

A nursing student is caring for a client with gastritis. Which of the following would the student recognize as a common cause of gastritis? Choose all that apply.

Ingestion of strong acids Irritating foods Overuse of aspirin

The nurse is assessing the abdomen of the client with an undiagnosed disorder. In which sequence would the nurse conduct the abdominal assessment? Use all options.

Inspection Auscultation Percussion Palpation

A client has ascites. Which of the following interventions would the nurse prepare to assist with implementing to help the client control this condition? Select all that apply.

Instructing the client to remove salty and salted foods from the diet Administering prescribed spironolactone (Aldactone) Assisting with placement of a transjugular intrahepatic portosystemic shunt Chapter 49: Assessment and Management of Patients With Hepatic Disorders - Page 1386-1387 The goal of treatment for the client with ascites is a negative sodium balance to reduce fluid retention. Table salt, salty foods, salted butter and margarine, and all ordinary canned and frozen foods that are not specifically prepared for low-sodium diets should be avoided. Spironolactone (Aldactone), an aldosterone-blocking agent, is most often the first-line therapy in clients with ascites from cirrhosis. Transjugular intrahepatic portosystemic shunt (TIPS) is a method of treating ascites in which a cannula is threaded into the portal vein by the transjugular route. In clients with ascites, an upright posture is associated with activation of the renin-angiotensin-aldosterone system and sympathetic nervous system. This causes reduced renal glomerular filtration and sodium excretion and a decreased response to loop diuretics. Therefore, bed rest may be a useful therapy, especially for clients whose condition is refractory to diuretics. Other measures include assessment and documentation of intake and output, abdominal girth, and daily weight to assess fluid status.

A nurse cares for a client with a disorder of the endocrine function of the pancreas. Which hormones or enzymes may be impacted by this disorder? Select all that apply.

Insulin Glucagon Somatostatin Hinkle, J.L., & Cheever, K.H., Brunner & Suddarth's Textbook of Medical-Surgical Nursing, 14th ed., Philadelphia, Wolters Kluwer, 2018, Chapter 50: Assessment and Management of Patients With Biliary Disorders, p. 1429.

When caring for a client with hepatitis B, the nurse should monitor closely for the development of which finding associated with a decrease in hepatic function?

Irritability and drowsiness Chapter 49: Assessment and Management of Patients With Hepatic Disorders - Page 1401 Although all the options are associated with hepatitis B, the onset of irritability and drowsiness suggests a decrease in hepatic function. To detect signs and symptoms of disease progression, the nurse should observe for disorientation, behavioral changes, and a decreasing level of consciousness and should monitor the results of liver function tests, including the blood ammonia level. If hepatic function is decreased, the nurse should take safety precautions.

A nurse geneticist is researching the gut microbiome and its relationship to disease. What is true regarding the microbiome?

It has over 100 times more genes than the human genome. Hinkle, J.L., & Cheever, K.H., Brunner & Suddarth's Textbook of Medical-Surgical Nursing, 14th ed., Philadelphia, Wolters Kluwer, 2018, Chapter 48: Assessment and Management of Patients with Obesity, p. 1361.

A nurse works in a community clinic that treats clients with obesity. The nurse understands that obesity is considered a disease based on what characteristics? Select all that apply.

It impairs normal body function It has characteristic signs and symptoms It causes morbidity

The nurse is conducting a community education program on colorectal cancer. Which statement should the nurse include in the program?

It is the third most common cancer in the United States.

A nurse working with clients with obesity understands that the hypothalamus plays an important role in hunger and satiety. Which statement best describes the role of the hypothalamus in hunger and satiety?

It signals higher neural pathways that lead to eating behaviors. Chapter 48: Assessment and Management of Patients with Obesity - Page 1360 The hypothalamus signals higher neural pathways that lead to eating behaviors. The hypothalamus does not signal the release of ghrelin or neuropeptide y, nor does it signal the GI system to slow gastric motility.

A client is scheduled for a cholecystogram for later in the day. What is the nurse's understanding on the diagnostic use of this exam?

It visualizes the gallbladder and bile duct. Hinkle, J.L., & Cheever, K.H., Brunner & Suddarth's Textbook of Medical-Surgical Nursing, 14th ed., Philadelphia, Wolters Kluwer, 2018, Chapter 50: Assessment and Management of Patients With Biliary Disorders, p. 1432.

The nurse is comparing Crohn's disease (regional enteritis) with ulcerative colitis. Which of the following describes Crohn's disease?

Its course is prolonged and variable

While conducting a physical examination of a client, which of the following skin findings would alert the nurse to the liklihood of liver problems? Select all that apply.

Jaundice Petechiae Ecchymoses Hinkle, J.L., & Cheever, K.H., Brunner & Suddarth's Textbook of Medical-Surgical Nursing, 14th ed., Philadelphia, Wolters Kluwer, 2018, Chapter 49: Assessment and Management of Patients With Hepatic Disorders, Physical Assessment, p. 1380.

The nurse is managing a gastric (Salem) sump tube for a patient who has an intestinal obstruction and will be going to surgery. What interventions should the nurse perform to make sure the tube is functioning properly?

Keep the vent lumen above the patient's waist to prevent gastric content reflux. Chapter 44: Digestive and Gastrointestinal Treatment Modalities - Page 1244 The blue vent lumen should be kept above the patient's waist to prevent reflux of gastric contents through it; otherwise, it acts as a siphon.

When assisting with the plan of care for a client receiving tube feedings, which of the following would the nurse include to reduce the client's risk for aspiration?

Keeping the client in a semi-Fowler's position at all times.

A client who is recovering from anesthesia following oral surgery for lip cancer is experiencing difficulty breathing deeply and coughing up secretions. Which measure will help ease the client's discomfort?

Keeping the head of the bed elevated.

A client who is recovering from anesthesia following oral surgery for lip cancer is experiencing difficulty breathing deeply and coughing up secretions. Which of the following measures will help ease the client's discomfort?

Keeping the head of the bed elevated.

The nurse is administering medications to a client that has elevated ammonia due to cirrhosis of the liver. What medication will the nurse give to detoxify ammonium and to act as an osmotic agent?

Lactulose Hinkle, J.L., & Cheever, K.H., Brunner & Suddarth's Textbook of Medical-Surgical Nursing, 14th ed., Philadelphia, Wolters Kluwer, 2018, Chapter 49: Assessment and Management of Patients With Hepatic Disorders, Medical Management, p. 1395.

Which of the following is a proton pump inhibitor used in the treatment of gastroesophageal reflux disease (GERD)? Select all that apply.

Lansoprazole (Prevacid) Rabeprazole (AcipHex) Esomeprazole (Nexium)

A patient is not having daily bowel movements and has begun taking a laxative for this problem. What should the nurse educate the patient about regarding laxative use?

Laxatives should not be routinely taken due to destruction of nerve endings in the colon.

A nurse cares for older adult clients with obesity and understands that older adults have greater risk for developing obesity than the general population. What pathophysiological concepts contribute to this? Select all that apply.

Lean skeletal mass decreases as an individual ages. Adipose tissue increases as an individual ages. Metabolism decreases as an individual ages. Hinkle, J.L., & Cheever, K.H., Brunner & Suddarth's Textbook of Medical-Surgical Nursing, 14th ed., Philadelphia, Wolters Kluwer, 2018, Chapter 48: Assessment and Management of Patients with Obesity, p. 1366.

A positive Rovsing's sign is indicative of appendicitis. A nurse knows to assess for this indicator by palpating the:

Left lower quadrant.

A focused GI assessment begins with a complete history and physical examination. Identify the quadrant of the abdomen to be palpated or percussed for a patient with pancreatitis.

Left upper Chapter 43: Assessment of Digestive and Gastrointestinal Function - Page 1231 The pancreas, which is about 6 inches long, is located behind the stomach in the upper left side of the body.

The health care provider orders the insertion of a single lumen nasogastric tube. When gathering the equipment for the insertion, what will the nurse select?

Levin tube Chapter 44: Digestive and Gastrointestinal Treatment Modalities - Page 1244 A Levin tube is a single lumen nasogastric tube. A Salem sump tube is a double lumen nasogastric tube; a Sengstaken-Blakemore tube is a triple lumen nasogastric tube. A Miller-Abbott tube is a double lumen nasoenteric tube.

A client receiving tube feedings has prescriptions for several drugs. Which drugs would the nurse expect to administer to the client without any special preparation? Select all that apply.

Liquid stool softener Sublingual nitroglycerin

A nurse cares for a client with obesity who has type 2 diabetes. Which medication does the nurse recognize may assist in weight loss and is also approved to treat type 2 diabetes?

Liraglutide

After 20 seconds of auscultating for bowel sounds on a client recovering from abdominal surgery, the nurse hears nothing. What should the nurse do based on the assessment findings?

Listen longer for the sounds.

For a client with salivary calculi, which procedure uses shock waves to disintegrate the stone?

Lithotripsy

A client was admitted to a critical care unit with esophageal varices and a precarious physical condition. What predisposes the client to have bleeding varices? Select all that apply.

Little protective tissue to protect fragile veins Chemical irritation Straining at stool Rough food Chapter 49: Assessment and Management of Patients With Hepatic Disorders - Page 1389 The varices are at risk for bleeding if they lie superficially in the mucosa, contain little protective elastic tissue, and are easily traumatized by rough food or chemical irritation. Straining at stool may cause the varices to bleed. Esophageal varices overfill as a result of portal hypertension, so decreasing the portal tension will lessen bleeding.

While palpating a client's right upper quadrant (RUQ), the nurse would expect to find which structure?

Liver

A 66-year-old African-American client has recently visited a physician to confirm a diagnosis of gastric cancer. The client has a history of tobacco use and was diagnosed 10 years ago with pernicious anemia. He and his family are shocked about the possibility of cancer because he was asymptomatic prior to recent complaints of pain and multiple gastrointestinal symptoms. On the basis of knowledge of disease progression, the nurse assumes that organs adjacent to the stomach are also affected. Which of the following organs may be affected? Choose all that apply.

Liver Pancreas Duodenum

A client has just received a diagnosis of hypertension after the completion of diagnostics. What can the client do to decrease the consequences of hypertension? Select all that apply.

Lose weight. Manage stress effectively

A physician orders spironolactone (Aldactone), 50 mg by mouth four times daily, for a client with fluid retention caused by cirrhosis. Which finding indicates that the drug is producing a therapeutic effect?

Loss of 2.2 lb (1 kg) in 24 hours Hinkle, J.L., & Cheever, K.H., Brunner & Suddarth's Textbook of Medical-Surgical Nursing, 14th ed., Philadelphia, Wolters Kluwer, 2018, Chapter 49: Assessment and Management of Patients With Hepatic Disorders, Pharmacologic Therapy, p. 1386.

As part of the management of constipation, the client is instructed to take 30 mL of mineral oil orally. How does mineral oil facilitate bowel evacuation?

Lubricates and softens fecal matter

Which of the following are functions of saliva? Select all that apply.

Lubrication Protection against harmful bacteria Digestion

A client with a recent history of rectal bleeding is being prepared for a colonoscopy. Initially, how should the nurse position the client for this test?

Lying on the left side with knees bent

A critical care nurse is caring for a client with acute pancreatitis. One potentially severe complication involves the respiratory system. Which of the following would be an appropriate intervention to prevent complications associated with the respiratory system?

Maintain the client in a semi-Fowler's position. Hinkle, J.L., & Cheever, K.H., Brunner & Suddarth's Textbook of Medical-Surgical Nursing, 14th ed., Philadelphia, Wolters Kluwer, 2018, Chapter 50: Assessment and Management of Patients With Biliary Disorders, Chart 50-4, p. 1447.

A nurse is planning care for a client with acute pancreatitis. Which client outcome does the nurse assign as the highest priority?

Maintaining normal respiratory function Hinkle, J.L., & Cheever, K.H., Brunner & Suddarth's Textbook of Medical-Surgical Nursing, 14th ed., Philadelphia, Wolters Kluwer, 2018, Chapter 50: Assessment and Management of Patients With Biliary Disorders, p. 1443-1444.

What is a major concern for the nurse when caring for a patient with chronic pancreatitis?

Maintaining normal respiratory function Hinkle, J.L., & Cheever, K.H., Brunner & Suddarth's Textbook of Medical-Surgical Nursing, 14th ed., Philadelphia, Wolters Kluwer, 2018, Chapter 50: Assessment and Management of Patients With Biliary Disorders, p. 1443-1444.

From the following profiles of clients, which client would be most likely to undergo the diagnostic test of cholecystography?

Mark, suspected of having stones in the gallbladder Hinkle, J.L., & Cheever, K.H., Brunner & Suddarth's Textbook of Medical-Surgical Nursing, 14th ed., Philadelphia, Wolters Kluwer, 2018, Chapter 50: Assessment and Management of Patients With Biliary Disorders, Oral Cholecystography, p. 1433.

A nurse is preparing to perform a dressing change to the site of a client's central venous catheter used for parenteral nutrition. Which equipment and supplies would the nurse need to gather? Select all that apply.

Masks Skin antiseptic Alcohol wipes Sterile gauze pads

When performing a physical examination on a client with cirrhosis, a nurse notices that the client's abdomen is enlarged. Which of the following interventions should the nurse consider?

Measure abdominal girth according to a set routine. Hinkle, J.L., & Cheever, K.H., Brunner & Suddarth's Textbook of Medical-Surgical Nursing, 14th ed., Philadelphia, Wolters Kluwer, 2018, Chapter 49: Assessment and Management of Patients With Hepatic Disorders, Fluid Volume Excess, p. 1416.

The nurse is caring for a patient who has ascites as a result of hepatic dysfunction. What intervention can the nurse provide to determine if the ascites is increasing? (Select all that apply.)

Measure abdominal girth daily. Perform daily weights. Chapter 49: Assessment and Management of Patients With Hepatic Disorders - Page 1416 Increased abdominal girth and rapid weight gain are common presenting symptoms of ascites. If a patient with ascites from liver dysfunction is hospitalized, nursing measures include assessment and documentation of intake and output (I&O;), abdominal girth, and daily weight to assess fluid status. The nurse also closely monitors the respiratory status because large volumes of ascites can compress the thoracic cavity and inhibit adequate lung expansion. The nurse monitors serum ammonia, creatinine, and electrolyte levels to assess electrolyte balance, response to therapy, and indications of encephalopathy.

A client with acute pancreatitis has been started on total parenteral nutrition (TPN). Which action should the nurse perform after administration of the TPN?

Measure blood glucose concentration every 4 to 6 hours Hinkle, J.L., & Cheever, K.H., Brunner & Suddarth's Textbook of Medical-Surgical Nursing, 14th ed., Philadelphia, Wolters Kluwer, 2018, Chapter 50: Assessment and Management of Patients With Biliary Disorders, p. 1444.

Swallowing is regulated by which area of the central nervous system (CNS)?

Medulla oblongata Chapter 43: Assessment of Digestive and Gastrointestinal Function - Page 1226 Swallowing begins as a voluntary act that is regulated by the swallowing center in the medulla oblongata of the CNS. The act of swallowing requires the innervations of five cranial nerves (CNs), especially CN V, VII, IX, X, and XII. Swallowing is not regulated by the pons, cerebellum, or hypothalamus.

A client with obesity is diagnosed with type 2 diabetes. In order to promote weight loss in the client and aid in glucose management, which medication will the nurse anticipate the health care provider ordering?

Metformin Hinkle, J.L., & Cheever, K.H., Brunner & Suddarth's Textbook of Medical-Surgical Nursing, 14th ed., Philadelphia, Wolters Kluwer, 2018, Chapter 48: Assessment and Management of Patients with Obesity, p. 1363.

A young client with anorexia, fatigue, and jaundice is diagnosed with hepatitis B and has just been admitted to the hospital. The client asks the nurse how long the stay in the hospital will be. In planning care for the client, the nurse identifies impaired psychosocial issues and assigns the highest priority to which client outcome?

Minimizing social isolation Chapter 49: Assessment and Management of Patients With Hepatic Disorders - Page 1403 The nurse identifies psychosocial issues and concerns, particularly the effects of separation from family and friends if the client is hospitalized during the acute and infective stages. Convalescence may be prolonged, with complete symptomatic recovery sometimes requiring 3 to 4 months or longer. Even if not hospitalized, the client will be unable to attend school and/or work and must avoid sexual contact. Planning is required to minimize social isolation.

Which of the following is a true statement regarding gastric cancer?

Most patients are asymptomatic during the early stage of the disease.

A nurse admits a wealthy client who is 8 weeks postpartum after her third child and overweight. The client reports severe right upper quadrant pain that radiates to the back after eating Thanksgiving dinner with turkey and gravy earlier in the evening. What factors lead the nurse to suspect gallbladder disease? Select all that apply.

Multiparous Obese Eating a high fat diet Female gender Hinkle, J.L., & Cheever, K.H., Brunner & Suddarth's Textbook of Medical-Surgical Nursing, 14th ed., Philadelphia, Wolters Kluwer, 2018, Chapter 48: Assessment and Management of Patients with Obesity, Pathophysiology, p. 1431.

The nurse identifies a potential collaborative problem of electrolyte imbalance for a client with severe acute pancreatitis. Which assessment finding alerts the nurse to an electrolyte imbalance associated with acute pancreatitis?

Muscle twitching and finger numbness Hinkle, J.L., & Cheever, K.H., Brunner & Suddarth's Textbook of Medical-Surgical Nursing, 14th ed., Philadelphia, Wolters Kluwer, 2018, Chapter 50: Assessment and Management of Patients With Biliary Disorders, Chart 50-4, p. 1448.

The nurse in the ED admits a client with suspected gastric outlet obstruction. The client's symptoms include nausea and vomiting. The nurse anticipates that the physician will issue which order?

Nasogastric tube insertion

A client is prescribed a histamine (H2)-receptor antagonist. The nurse understands that this might include which medication(s)? Select all that apply.

Nizatidine Famotidine Cimetidine

A client is receiving parenteral nutrition (PN) through a peripherally inserted central catheter (PICC) and will be discharged home with PN. The home health nurse evaluates the home setting and would make a recommendation when noticing which circumstance?

No land line; cell phone available and taken by family member during working hours

A client is to receive parenteral nutrition for 4 weeks. What type of catheter would the nurse prepare to teach the client about the planned nutrition?

Nontunneled central catheter

A client is scheduled to have a laparoscopic cholecystectomy as an outpatient. The client asks the nurse when he will be able to resume normal activities. What information should the nurse provide?

Normal activities may be resumed in 1 week. Hinkle, J.L., & Cheever, K.H., Brunner & Suddarth's Textbook of Medical-Surgical Nursing, 14th ed., Philadelphia, Wolters Kluwer, 2018, Chapter 49: Assessment and Management of Patients With Hepatic Disorders, Laparoscopic Cholecystectomy, p. 1436.

The nurse working in the recovery room is caring for a client who had a radical neck dissection. The nurse notices that the client makes a coarse, high-pitched sound upon inspiration. Which intervention by the nurse is appropriate?

Notify the physician

A client recovering from gastric bypass surgery accidentally removes the nasogastric tube. What is the best action by the nurse?

Notify the surgeon about the tube's removal.

A nurse is preparing a presentation for a local community group addressing the influences on health care delivery. Which of the following would the nurse include in presentation when describing disease patterns?`

Obesity along with conditions associated with it has become a major health concern.

When preparing a client for surgery to treat appendicitis, the nurse formulates a nursing diagnosis of Risk for infection related to inflammation, perforation, and surgery. What is the rationale for choosing this nursing diagnosis?

Obstruction of the appendix reduces arterial flow, leading to ischemia, inflammation, and rupture of the appendix.

A client is admitted to the hospital with acute hemorrhage from esophageal varices. What medication should the nurse anticipate administering that will reduce pressure in the portal venous system and control esophageal bleeding?

Octreotide Chapter 49: Assessment and Management of Patients With Hepatic Disorders - Page 1390 Acute hemorrhage from esophageal varices is life threatening. Resuscitative measures include administration of IV fluids and blood products. IV octreotide is started as soon as possible. Octreotide is preferred because of fewer side effects. Octreotide reduces pressure in the portal venous system and is preferred to the previously used agents, vasopressin or terlipressin. Vitamin K promotes blood coagulation in bleeding conditions, resulting from liver disease.

A client is actively bleeding from esophageal varices. Which medication would the nurse most expect to be administered to this client?

Octreotide Hinkle, J.L., & Cheever, K.H., Brunner & Suddarth's Textbook of Medical-Surgical Nursing, 14th ed., Philadelphia, Wolters Kluwer, 2018, Chapter 49: Assessment and Management of Patients With Hepatic Disorders, Pharmacologic Therapy, p. 1390.

A client has undergone a liver biopsy. After the procedure, the nurse should place the client in which position?

On the right side Hinkle, J.L., & Cheever, K.H., Brunner & Suddarth's Textbook of Medical-Surgical Nursing, 14th ed., Philadelphia, Wolters Kluwer, 2018.

The nurse is creating a discharge teaching plan for a client after surgery for oral cancer. Which should be included in the teaching plan? Select all that apply.

Oral hygiene Follow-up dental appointment Follow-up medical appointment Use of humidification

A client with obesity reports pain in the joints. Which musculoskeletal condition related to obesity does the nurse suspect the client has?

Osteoarthritis

Patients with irritable bowel disease (IBD) are at significantly increased risk for which of the following?

Osteoporosis

A client with carcinoma of the head of the pancreas is scheduled for surgery. Which of the following should the nurse administer to the client before surgery?

Packed red blood cells Hinkle, J.L., & Cheever, K.H., Brunner & Suddarth's Textbook of Medical-Surgical Nursing, 14th ed., Philadelphia, Wolters Kluwer, 2018, Chapter 50: Assessment and Management of Patients With Biliary Disorders, Medical Management, p. 1452.

What initial measure can the nurse implement to reduce risk of injury for a client with liver disease?

Pad the side rails on the bed Hinkle, J.L., & Cheever, K.H., Brunner & Suddarth's Textbook of Medical-Surgical Nursing, 14th ed., Philadelphia, Wolters Kluwer, 2018, Chapter 49: Assessment and Management of Patients With Hepatic Disorders, p. 1408.

Which of the following factors may contribute to rapid and shallow respirations in a postoperative client? Select all that apply.

Pain Constricting dressings Abdominal distention Obesity Hinkle, J. L., Cheever, K. H. Brunner & Suddarth's Textbook of Medical-Surgical Nursing , 14th ed Philadelphia: Wolters Kluwer Health/Lippincott Williams & Wilkins, 2018, Chapter 19: Postoperative Nursing Management, p. 464.

A client diagnosed with acute pancreatitis is being transferred to another facility. The nurse caring for the client completes the transfer summary, which includes information about the client's drinking history and other assessment findings. Which assessment findings confirm the diagnosis of acute pancreatitis?

Pain with abdominal distention and hypotension Hinkle, J.L., & Cheever, K.H., Brunner & Suddarth's Textbook of Medical-Surgical Nursing, 14th ed., Philadelphia, Wolters Kluwer, 2018, Chapter 50: Assessment and Management of Patients With Biliary Disorders, Clinical Manifestations, p. 1445.

An elderly client comes into the emergency department reporting an earache. The client and has an oral temperature of 37.9° (100.2ºF) and otoscopic assessment of the ear reveals a pearly gray tympanic membrane with no evidence of discharge or inflammation. Which action should the triage nurse take next?

Palpate the client's parotid glands to detect swelling and tenderness.

A client is admitted to the health care center with hyperglycemia, a 15-pound weight loss, and reports of vague upper and midabdominal pain that increases in intensity at night. The client' health history indicates alcoholism, smoking of a pack of cigarettes daily, and diabetes for the past 20 years. Upon examination the nurse finds swelling in the feet and abdominal ascites. Based on the clinical manifestations, which condition is the most likely diagnosis?

Pancreatic carcinoma Hinkle, J.L., & Cheever, K.H., Brunner & Suddarth's Textbook of Medical-Surgical Nursing, 14th ed., Philadelphia, Wolters Kluwer, 2018, Chapter 50: Assessment and Management of Patients With Biliary Disorders, p. 1450.

Which condition is most likely to have a nursing diagnosis of fluid volume deficit?

Pancreatitis Hinkle, J.L., & Cheever, K.H., Brunner & Suddarth's Textbook of Medical-Surgical Nursing, 14th ed., Philadelphia, Wolters Kluwer, 2018, Chapter 50: Assessment and Management of Patients With Biliary Disorders, Intensive Care, p. 1442.

Which of the following conditions is most likely to involve a nursing diagnosis of fluid volume deficit?

Pancreatitis Hinkle, J.L., & Cheever, K.H., Brunner & Suddarth's Textbook of Medical-Surgical Nursing, 14th ed., Philadelphia, Wolters Kluwer, 2018, Chapter 50: Assessment and Management of Patients With Biliary Disorders, Intensive Care, p. 1442.

The nurse is caring for a client with acute pancreatitis who is admitted to the intensive care unit to monitor for pulmonary complications. What is the nurse's understanding of the pathophysiology of pulmonary complications related to pancreatitis?

Pancreatitis can elevate the diaphragm and alter the breathing pattern. Hinkle, J.L., & Cheever, K.H., Brunner & Suddarth's Textbook of Medical-Surgical Nursing, 14th ed., Philadelphia, Wolters Kluwer, 2018, Chapter 50: Assessment and Management of Patients With Biliary Disorders, p. 1443.

Which term describes the passage of a hollow instrument into a cavity to withdraw fluid?

Paracentesis Chapter 49: Assessment and Management of Patients With Hepatic Disorders - Page 1387 Paracentesis may be used to withdraw fluid (ascites) if the accumulated fluid is causing cardiorespiratory compromise. Asterixis refers to involuntary flapping movements of the hands associated with metabolic liver dysfunction. Ascites refers to accumulation of serous fluid within the peritoneal cavity. Dialysis refers to a form of filtration to separate crystalloid from colloid substances.

A nurse is gathering equipment and preparing to assist with a sterile bedside procedure to withdraw fluid from a client's abdomen. The procedure tray contains the following equipment: trocar, syringe, needles, and drainage tube. The client is placed in he high Fowler position and a blood pressure cuff is secured around the arm in preparation for which procedure?

Paracentesis Hinkle, J.L., & Cheever, K.H., Brunner & Suddarth's Textbook of Medical-Surgical Nursing, 14th ed., Philadelphia, Wolters Kluwer, 2018, Chapter 49: Assessment and Management of Patients With Hepatic Disorders, p. 1387.

Post appendectomy, a nurse should assess the patient for abdominal rigidity and tenderness, fever, loss of bowel sounds, and tachycardia, all clinical signs of:

Peritonitis

The nurse is monitoring a client's postoperative course after an appendectomy. The nurse's assessment reveals that the client has vomited, has abdominal tenderness and rigidity, and has tachycardia. The nurse reports to the physician that the client has signs/symptoms of which complication?

Peritonitis

After being in remission from Hodgkin's disease for 18 months, a client develops a fever of unknown origin. A healthcare provider orders a liver biopsy to rule out advancing Hodgkin's disease and infection. Twenty-four hours after the biopsy, the client has a fever, complains of severe abdominal pain, and seems increasingly confused. What should the nurse suspect?

Peritonitis from bleeding in the liver caused by the liver biopsy Chapter 49: Assessment and Management of Patients With Hepatic Disorders - Page 1382 After any invasive procedure, the nurse must stay alert for complications in the affected region—in this case, the abdomen. This client exhibits classic signs and symptoms of peritonitis caused by blood or bile after the liver biopsy. There is a reason to suspect bleeding resulting from the liver biopsy. It is rare to have a perforation of the colon after a biopsy. Liver biopsy doesn't involve the use of contrast media. The client's symptoms are not normal for a liver biopsy.

Tube feedings are advised for a client who is recovering from oral surgery. The nurse manages the tube feedings to minimize the risk of aspiration. Which measures should the nurse include in the care plan to reduce the risk of aspiration? Select all that apply.

Place client in semi-Fowler's position during and 30 to 60 minutes after an intermittent feeding. Check tube placement and gastric residual prior to feedings.

The nurse cares for a client with cholecystitis with severe biliary colic symptoms. Which nursing intervention best promotes adequate respirations in a client with these symptoms?

Place the client in semi-Fowler's position. Hinkle, J.L., & Cheever, K.H., Brunner & Suddarth's Textbook of Medical-Surgical Nursing, 14th ed., Philadelphia, Wolters Kluwer, 2018, Chapter 50: Assessment and Management of Patients With Biliary Disorders, p. 1432.

Which is the most common cause of esophageal varices?

Portal hypertension Hinkle, J.L., & Cheever, K.H., Brunner & Suddarth's Textbook of Medical-Surgical Nursing, 14th ed., Philadelphia, Wolters Kluwer, 2018, Chapter 49: Assessment and Management of Patients With Hepatic Disorders, p. 1389.

An examiner is performing the physical assessment of the rectum, perianal region, and anus. While this examination can be uncomfortable for many clients, health care providers must approach it in a prepared, confident manner. Which of the following considerations will help this examination flow smoothly and efficiently for both provider and client? Select all that apply.

Position the client on the right side with the knees up to the chest. Ask the client to bear down for visual inspection.

A nurse is preparing to discharge a client newly diagnosed with peptic ulcer disease. The client's diagnostic test results were positive for H. pylori bacteria. The health care provider has ordered the "triple therapy" regimen. Which medications will the nurse educate the client on?

Proton-pump inhibitor and two antibiotics

A patient has been diagnosed with a hiatal hernia. The nurse explains the diagnosis to the patient and his family by telling them that a hernia is a (an):

Protrusion of the upper stomach into the lower portion of the thorax.

The nurse observes dry mucous membranes in a client who is receiving tube feedings after an oral surgery. The client also reports unpleasant tastes and odors. Which measure should be included in the client's plan of care?

Provide frequent mouth care.

Because clients with pancreatitis cannot tolerate high-glucose concentrations, total parental nutrition (TPN) should be used cautiously with them. Which of the following interventions has shown great promise in the prognosis of clients with severe acute pancreatitis?

Providing intensive insulin therapy Hinkle, J.L., & Cheever, K.H., Brunner & Suddarth's Textbook of Medical-Surgical Nursing, 14th ed., Philadelphia, Wolters Kluwer, 2018, Chapter 50: Assessment and Management of Patients With Biliary Disorders, Intensive Care, p. 1443.

The nurse is scheduling gastrointestinal (GI) diagnostic testing for a client. Which GI test should be scheduled first?

Radiography of the gallbladder Chapter 43: Assessment of Digestive and Gastrointestinal Function - Page 1234 Radiography of the gallbladder should be performed before other GI exams in which barium is used because residual barium tends to obscure the images of the gallbladder and its duct.

Which liver function study is used to show the size of the liver and hepatic blood flow and obstruction?

Radioisotope liver scan Chapter 49: Assessment and Management of Patients With Hepatic Disorders - Page 1382 A radioisotope liver scan assesses liver size and hepatic blood flow and obstruction. Magnetic resonance imaging is used to identify normal structures and abnormalities of the liver and biliary tree. Angiography is used to visualize hepatic circulation and detect the presence and nature of hepatic masses. Electroencephalography is used to detect abnormalities that occur with hepatic coma.

Following ingestion of carrots or beets, the nurse would expect which alteration in stool color?

Red

A nurse is caring for a client who had an ileal conduit 3 days earlier. The nurse examines the stoma site and determines that she should consult with the ostomy nurse. Which assessment finding indicates the need for further consultation?

Red, sensitive skin around the stoma site

A client with cirrhosis has portal hypertension, which is causing esophageal varices. What is the goal of the interventions that the nurse will provide?

Reduce fluid accumulation and venous pressure. Hinkle, J.L., & Cheever, K.H., Brunner & Suddarth's Textbook of Medical-Surgical Nursing, 14th ed., Philadelphia, Wolters Kluwer, 2018, Chapter 49: Assessment and Management of Patients With Hepatic Disorders, Medical Management, p. 1390.

A client is treated for gastrointestinal problems related to chronic cholecystitis. What pathophysiological process related to cholecystitis does the nurse understand is the reason behind the client's GI problems?

Reduced or absent bile as a result of obstruction impacts digestion. Hinkle, J.L., & Cheever, K.H., Brunner & Suddarth's Textbook of Medical-Surgical Nursing, 14th ed., Philadelphia, Wolters Kluwer, 2018, Chapter 50: Assessment and Management of Patients With Biliary Disorders, pp. 1433-1434.

A client is scheduled for bowel resection with anastomosis involving the large intestine. Because of the surgical site, the nurse formulates the nursing diagnosis of Risk for infection. To complete the nursing diagnosis statement, the nurse should add which "related-to" phrase?

Related to the presence of bacteria at the surgical site

Which of the following is the most successful treatment for gastric cancer?

Removal of the tumor

A nursing instructor is preparing a class about gastrointestinal intubation. Which of the following would the instructor include as reason for this procedure? Select all that apply.

Remove gas and fluids from the stomach Diagnose gastrointestinal motility disorders Flush ingested toxins from the stomach Administer nutritional substances

A nurse caring for a client with obesity recognizes the client is at risk for renal complications related to obesity. Which disease or condition is associated with obesity?

Renal cancer Hinkle, J.L., & Cheever, K.H., Brunner & Suddarth's Textbook of Medical-Surgical Nursing, 14th ed., Philadelphia, Wolters Kluwer, 2018, Chapter 48: Assessment and Management of Patients with Obesity, p. 1359.

A client with pancreatitis is admitted to the medical intensive care unit. Which nursing intervention is most appropriate?

Reserving an antecubital site for a peripherally inserted central catheter (PICC) Hinkle, J.L., & Cheever, K.H., Brunner & Suddarth's Textbook of Medical-Surgical Nursing, 14th ed., Philadelphia, Wolters Kluwer, 2018, Chapter 50: Assessment and Management of Patients With Biliary Disorders, Medical Management, p. 1442.

The nurse is providing care to a patient with gross ascites who is maintaining a position of comfort in the high semi-Fowler's position. What is the nurse's priority assessment of this patient?

Respiratory assessment related to increased thoracic pressure Hinkle, J.L., & Cheever, K.H., Brunner & Suddarth's Textbook of Medical-Surgical Nursing, 14th ed., Philadelphia, Wolters Kluwer, 2018, Chapter 49: Assessment and Management of Patients With Hepatic Disorders, Promoting Rest, p. 1408.

The nurse cares for a client after an endoscopic examination and prepares the client for discharge. The nurse includes which instruction?

Resume regular diet.

After undergoing a liver biopsy, a client should be placed in which position?

Right lateral decubitus position Hinkle, J.L., & Cheever, K.H., Brunner & Suddarth's Textbook of Medical-Surgical Nursing, 14th ed., Philadelphia, Wolters Kluwer, 2018, Chapter 49: Assessment and Management of Patients With Hepatic Disorders, Liver Biopsy, p. 1382.

A nurse researcher examines the risk factors of obesity. Which statements does the nurse find true? Select all that apply.

Risk factors that identify the odds of being diagnosed with obesity are not clearly defined. Causes of obesity are complex and multifactorial. Obesity increases the risk of mortality. Obesity decreases the overall life expectancy. Hinkle, J.L., & Cheever, K.H., Brunner & Suddarth's Textbook of Medical-Surgical Nursing, 14th ed., Philadelphia, Wolters Kluwer, 2018, Chapter 48: Assessment and Management of Patients with Obesity, p. 1359.

A nurse is caring for a client who is post operative bariatric surgery. The client's pain has not been well controlled. Which nursing diagnosis is the nurse's priority?

Risk for impaired gas exchange Hinkle, J.L., & Cheever, K.H., Brunner & Suddarth's Textbook of Medical-Surgical Nursing, 14th ed., Philadelphia, Wolters Kluwer, 2018, Chapter 48: Assessment and Management of Patients with Obesity, p. 1371.

A client presents to the ED with acute abdominal pain, fever, nausea, and vomiting. During the client's examination, the lower left abdominal quadrant is palpated, causing the client to report pain in the RLQ. This positive sign is referred to as ________ and suggests the client may be experiencing ________.

Rovsing's sign; acute appendicitis

In actively bleeding patients with esophageal varices, the initial drug of therapy is usually:

Sandostatin Chapter 49: Assessment and Management of Patients With Hepatic Disorders - Page 1390 In an actively bleeding patient, medications are given initially because they can be obtained and given more quickly than other therapies. Sandostatin, a synthetic analog of the hormone somatostatin, is effective in decreasing bleeding from esophageal varices and lacks the vasoconstrictive effects of vasopressin. Because of this safety and efficacy profile, octreotide is considered the preferred treatment regimen for immediate control of variceal bleeding.

Clients with chronic liver dysfunction have problems with insufficient vitamin intake. Which may occur as a result of vitamin C deficiency?

Scurvy Hinkle, J.L., & Cheever, K.H., Brunner & Suddarth's Textbook of Medical-Surgical Nursing, 14th ed., Philadelphia, Wolters Kluwer, 2018, Chapter 49: Assessment and Management of Patients With Hepatic Disorders, p. 1397.

Which type of diarrhea is caused by increased production of water and electrolytes by the intestinal mucosa and their secretion into the intestinal lumen?

Secretory diarrhea

A patient is prescribed Sandostatin for the treatment of esophageal varices. The nurse knows that the purpose of this cyclic octapeptide is to reduce portal pressure by:

Selective vasodilation of the portal system. Hinkle, J.L., & Cheever, K.H., Brunner & Suddarth's Textbook of Medical-Surgical Nursing, 14th ed., Philadelphia, Wolters Kluwer, 2018, Chapter 49: Assessment and Management of Patients With Hepatic Disorders, Pharmacologic Therapy, p. 1390.

A nurse is teaching a client about the cause of acute pancreatitis. The nurse evaluates the teaching as effective when the client correctly identifies which condition as a cause of acute pancreatitis?

Self-digestion of the pancreas by its own proteolytic enzymes Hinkle, J.L., & Cheever, K.H., Brunner & Suddarth's Textbook of Medical-Surgical Nursing, 14th ed., Philadelphia, Wolters Kluwer, 2018, Chapter 50: Assessment and Management of Patients With Biliary Disorders, p. 1441.

A patient is receiving continuous tube feedings. The nurse would maintain the patient in which position at all times?

Semi-Fowler's with the head of the bed elevated 30 to 45 degrees

A health care provider counsels a client about bariatric surgery and recommends the Roux-en-Y gastric bypass. What is the best response by the nurse to further explain this procedure to the client?

Separation of the jejunum with an anastomosis

A client comes to the ED with severe abdominal pain, nausea, and vomiting. The physician plans to rule out acute pancreatitis. The nurse would expect the diagnosis to be confirmed by an elevated result on which laboratory test?

Serum amylase Hinkle, J.L., & Cheever, K.H., Brunner & Suddarth's Textbook of Medical-Surgical Nursing, 14th ed., Philadelphia, Wolters Kluwer, 2018, Chapter 50: Assessment and Management of Patients With Biliary Disorders, p. 1442.

The nurse should assess for an important early indicator of acute pancreatitis, which is a prolonged and elevated level of:

Serum lipase Hinkle, J.L., & Cheever, K.H., Brunner & Suddarth's Textbook of Medical-Surgical Nursing, 14th ed., Philadelphia, Wolters Kluwer, 2018, Chapter 50: Assessment and Management of Patients With Biliary Disorders, Assessment and Diagnostic Findings, p. 1442.

The nurse should assess for an important early indicator of acute pancreatitis. What prolonged and elevated level would the nurse determine is an early indicator?

Serum lipase Hinkle, J.L., & Cheever, K.H., Brunner & Suddarth's Textbook of Medical-Surgical Nursing, 14th ed., Philadelphia, Wolters Kluwer, 2018, Chapter 50: Assessment and Management of Patients With Biliary Disorders, Assessment and Diagnostic Findings, p. 1442.

The nurse is admitting a patient to the intensive care unit with a diagnosis of acute pancreatitis. What does the nurse expect was the reason the patient came to the hospital?

Severe abdominal pain Hinkle, J.L., & Cheever, K.H., Brunner & Suddarth's Textbook of Medical-Surgical Nursing, 14th ed., Philadelphia, Wolters Kluwer, 2018, Chapter 50: Assessment and Management of Patients With Biliary Disorders, Clinical Manifestations, p. 1441.

The nursing student has just reviewed material in the course textbook regarding pancreatitis. The student knows that a major symptom of pancreatitis that causes the client to seek medical care is:

Severe abdominal pain Hinkle, J.L., & Cheever, K.H., Brunner & Suddarth's Textbook of Medical-Surgical Nursing, 14th ed., Philadelphia, Wolters Kluwer, 2018, Chapter 50: Assessment and Management of Patients With Biliary Disorders, Clinical Manifestations, p. 1441.

A nursing instructor is explaining the pathophysiology and clinical manifestations of pancreatitis to a group of nursing students. The instructor evaluates the teaching as effective when a student correctly identifies which symptom as that most commonly reported by clients with pancreatitis?

Severe, radiating abdominal pain Hinkle, J.L., & Cheever, K.H., Brunner & Suddarth's Textbook of Medical-Surgical Nursing, 14th ed., Philadelphia, Wolters Kluwer, 2018, Chapter 50: Assessment and Management of Patients With Biliary Disorders, p. 1441-1442.

Which is the most common report by clients with pancreatitis?

Severe, radiating abdominal pain Hinkle, J.L., & Cheever, K.H., Brunner & Suddarth's Textbook of Medical-Surgical Nursing, 14th ed., Philadelphia, Wolters Kluwer, 2018, Chapter 50: Assessment and Management of Patients With Biliary Disorders, p. 1441-1442.

A nurse suspects that a client is developing rebound hypoglycemia secondary to parenteral nutrition being discontinued too rapidly. Which assessment support the nurse's suspicion? Select all that apply.

Shakiness Tachycardia Weakness Confusion

The nurse notes that a client has inflammation of the salivary glands. The nurse documents which finding?

Sialadenitis

Which term describes an inflammation of the salivary glands?

Sialadenitis

A client who has been having recurrent attacks of severe abdominal pain over the past few months informs the physician about a 25-pound weight loss in the past year. The nurse attributes which factor as the most likely cause of this weight loss?

Skipping meals out of fear of painful attacks Hinkle, J.L., & Cheever, K.H., Brunner & Suddarth's Textbook of Medical-Surgical Nursing, 14th ed., Philadelphia, Wolters Kluwer, 2018, Chapter 50: Assessment and Management of Patients With Biliary Disorders, p. 1445.

A client is preparing for discharge to home following a partial gastrectomy and vagotomy. Which is the best rationale for the client being taught to lie down for 30 minutes after each meal?

Slows gastric emptying

The nurse is reviewing the chart of a client with swallowing problems. Which factors would raise suspicion that the client has cancer of the esophagus? Select all that apply.

Smoking history of 20 years Male gender Previous treatment for gastroesophageal reflux disease

Which hormones released throughout the gastrointestinal tract promote satiety? Select all that apply.

Somatostatin Cholecystokinin Insulin Hinkle, J.L., & Cheever, K.H., Brunner & Suddarth's Textbook of Medical-Surgical Nursing, 14th ed., Philadelphia, Wolters Kluwer, 2018, Chapter 48: Assessment and Management of Patients with Obesity, p. 1360.

Gynecomastia is a common side effect of which of the following diuretics?

Spironolactone Hinkle, J.L., & Cheever, K.H., Brunner & Suddarth's Textbook of Medical-Surgical Nursing, 14th ed., Philadelphia, Wolters Kluwer, 2018, Chapter 49: Assessment and Management of Patients With Hepatic Disorders, Medical Management, p. 1408.

The nurse is concerned about potassium loss when a diuretic is prescribed for a patient with ascites and edema. What diuretic may be ordered that spares potassium and prevents hypokalemia?

Spironolactone (Aldactone) Hinkle, J.L., & Cheever, K.H., Brunner & Suddarth's Textbook of Medical-Surgical Nursing, 14th ed., Philadelphia, Wolters Kluwer, 2018, Chapter 49: Assessment and Management of Patients With Hepatic Disorders, Medical Management, p. 1408.

A nurse assesses a patient diagnosed with hepatic encephalopathy. She observes a number of clinical signs, including asterixis and fetor hepaticus; the patient's electroencephalogram (EEG) is abnormal. The nurse documents that the patient is exhibiting signs of which stage of hepatic encephalopathy?

Stage 2 Hinkle, J.L., & Cheever, K.H., Brunner & Suddarth's Textbook of Medical-Surgical Nursing, 14th ed., Philadelphia, Wolters Kluwer, 2018, Chapter 49: Assessment and Management of Patients With Hepatic Disorders, Table 49-3, p. 1394.

It is important for a nurse to have an understanding of the major digestive enzymes and their actions. Choose the gastric mucosa secretion that plays an important role in the digestion of triglycerides.

Steapsin

A nurse cares for a client who is post op from bariatric surgery. Once able, the nurse encourages oral intake for what primary purpose?

Stimulate GI peristalsis

When evaluating the function of the GI tract, the nurse needs to understand the role of hormones. Secretin, stimulated by the pH of chyme in the duodenum, is a major GI hormone that does which of the following?

Stimulates the production of bicarbonate in pancreatic juice

After teaching a group of students about the various organs of the upper gastrointestinal tract and possible disorders, the instructor determines that the teaching was successful when the students identify which of the following structures as possibly being affected?

Stomach

The nurse is reviewing the laboratory test results of a client with Crohn disease. Which of the following would the nurse most likely find?

Stool cultures negative for microorganisms or parasite

A client who has undergone colostomy surgery is experiencing constipation. Which intervention should a nurse consider for such a client?

Suggest fluid intake of at least 2 L/day

A client has been diagnosed with Zenker's diverticulum. What treatment does the nurse include in the client education?

Surgical removal of the diverticulum

A client comes to the clinic complaining of a sore throat. When assessing the client, the nurse observes a reddened ulcerated lesion on the lip. The client tells the nurse that it has been there for a couple of weeks but it does not hurt. What should the nurse consult with the health care provider about testing for?

Syphilis

A nurse is monitoring a client with peptic ulcer disease. Which assessment findings would most likely indicate perforation of the ulcer? Select all that apply.

Tachycardia Hypotension A rigid, board-like abdomen

A client with hepatitis who has not responded to medical treatment is scheduled for a liver transplant. Which of the following most likely would be ordered?

Tacrolimus Hinkle, J.L., & Cheever, K.H., Brunner & Suddarth's Textbook of Medical-Surgical Nursing, 14th ed., Philadelphia, Wolters Kluwer, 2018, Chapter 49: Assessment and Management of Patients With Hepatic Disorders, Liver Transplantation, p. 1420.

A client with an esophageal stricture is about to undergo esophageal dilatation. As the bougies are passed down the esophagus, the nurse should instruct the client to do which action to minimize the vomiting urge?

Take long, slow breaths

A client with peptic ulcer disease has been prescribed sucralfate. What health education should the nurse provide to this client?

Take the medication 2 hours before or after other medications

The nurse is collecting a stool specimen from a patient. What characteristic of the stool indicates to the nurse that the patient may have an upper GI bleed?

Tarry and black

While preparing a client for an upper GI endoscopy (esophagogastroduodenoscopy), the nurse should implement which interventions? Choose all that apply.

Tell the client he shouldn't eat or drink for 6 to 12 hours before the procedure. Inform the client that he will receive a sedative before the procedure.

A nurse enters the room of a client who has returned to the unit after having a radical neck dissection. Which assessment finding requires immediate intervention?

The client lying in a lateral position, with the head of bed flat

The nurse is providing instructions to a client scheduled for a gastroscopy. What should the nurse be sure to include in the instructions? Select all that apply.

The client must fast for 8 hours before the examination. The throat will be sprayed with a local anesthetic. After gastroscopy, the client cannot eat or drink until the gag reflex returns (1 to 2 hours).

A client receives a local anesthetic to suppress the gag reflex for a diagnostic procedure of the upper GI tract. Which nursing intervention is advised for this patient?

The client should not be given any food and fluids until the gag reflex returns.

A nurse is assessing a client with a family history of cancer. Which finding requires immediate follow-up?

The client states he feels like he always has a lump in his throat.

The nurse is teaching the client about the upcoming endoscopic retrograde cholangiopancreatography (ERCP). Although the nurse instructs on several pertinent points of care, which is emphasized?

The client will change positions frequently throughout the procedure.

The nurse is completing a morning assessment of a client with cirrhosis. Which information obtained by the nurse will be of most concern?

The client's hands flap back and forth when the arms are extended. Hinkle, J.L., & Cheever, K.H., Brunner & Suddarth's Textbook of Medical-Surgical Nursing, 14th ed., Philadelphia, Wolters Kluwer, 2018, Chapter 49: Assessment and Management of Patients With Hepatic Disorders, p. 1395.

A nurse is caring for a client with cirrhosis. The nurse assesses the client at noon and discovers that the client is difficult to arouse and has an elevated serum ammonia level. The nurse should suspect which situation?

The client's hepatic function is decreasing. Hinkle, J.L., & Cheever, K.H., Brunner & Suddarth's Textbook of Medical-Surgical Nursing, 14th ed., Philadelphia, Wolters Kluwer, 2018, Chapter 49: Assessment and Management of Patients With Hepatic Disorders, Table 49-3, p. 1394.

A client realizes that regular use of laxatives has greatly improved bowel patterns. However, the nurse cautions this client against the prolonged use of laxatives for which reason?

The client's natural bowel function may become sluggish.

A client realizes that regular use of laxatives has greatly improved his bowel pattern. However, the nurse cautions this client against the prolonged use of laxatives for which reason?

The client's natural bowel function may become sluggish.

Which is a true statement regarding regional enteritis (Crohn's disease)?

The clusters of ulcers take on a cobblestone appearance.

A client tells the nurse, "I am not having normal bowel movements." When differentiating between what are normal and abnormal bowel habits, what indicators are the most important?

The consistency of stool and comfort when passing stool

Ammonia, the major etiologic factor in the development of encephalopathy, inhibits neurotransmission. Increased levels of ammonia are damaging to the body. The largest source of ammonia is from:

The digestion of dietary and blood proteins. Hinkle, J.L., & Cheever, K.H., Brunner & Suddarth's Textbook of Medical-Surgical Nursing, 14th ed., Philadelphia, Wolters Kluwer, 2018, Chapter 49: Assessment and Management of Patients With Hepatic Disorders, Pathophysiology, p. 1394.

The nurse is inserting a Levin tube for a patient for gastric decompression. The tube should be inserted to 6 to 10 cm beyond what length?

The distance measured from the tip of the nose to the earlobe and from the earlobe to the xiphoid process

A nurse is admitting a client diagnosed with late-stage gastric cancer. The client's family is distraught and angry that she was not diagnosed earlier in the course of her disease. What factor most likely contributed to the client's late diagnosis?

The early symptoms of gastric cancer are usually not alarming or highly unusual.

Which is an accurate statement regarding gastric cancer?

The incidence of stomach cancer continues to decrease in the United States.

The nurse observes that a client's medical report indicates that the client has Cushing syndrome. During inspection, the nurse notes that the client's BMI is 31, waist circumference is 40 inches, and localized fat pads exist around the neck and upper part of the back. Which of the following must the nurse keep in mind while planning the client's care?

The nurse recognizes that the client's obesity may be specifically related to the endocrine disorder. The nurse performs a thorough nutritional assessment.

A client with diabetes begins to have digestive problems and is told by the physician that they are a complication of the diabetes. Which of the following explanations from the nurse is most accurate?

The pancreas secretes digestive enzymes. Chapter 43: Assessment of Digestive and Gastrointestinal Function - Page 1227 While the pancreas has the well-known function of secreting insulin, it also secretes digestive enzymes. These enzymes include trypsin, amylase, and lipase. If the secretion of these enzymes are affected by a diseased pancreas as found with diabetes, the digestive functioning may be impaired.

A patient with acute pancreatitis puts the call bell on to tell the nurse about an increase in pain. The nurse observes the patient guarding; the abdomen is board-like and no bowel sounds are detected. What is the major concern for this patient?

The patient has developed peritonitis. Hinkle, J.L., & Cheever, K.H., Brunner & Suddarth's Textbook of Medical-Surgical Nursing, 14th ed., Philadelphia, Wolters Kluwer, 2018, Chapter 50: Assessment and Management of Patients With Biliary Disorders, Clinical Manifestations, p. 1442.

A patient is scheduled for a diagnostic paracentesis, but when coagulation studies were reviewed, the nurse observed they were abnormal. How does the nurse anticipate the physician will proceed with the paracentesis?

The physician will use an ultrasound guided paracentesis Chapter 49: Assessment and Management of Patients With Hepatic Disorders - Page 1387 Paracentesis is the removal of fluid (ascites) from the peritoneal cavity through a puncture or a small surgical incision through the abdominal wall under sterile conditions (Gordon, 2012). Ultrasound guidance may be indicated in some patients who are at high risk for bleeding because of an abnormal coagulation profile and in those who have had previous abdominal surgery and may have adhesions.

A client is to have an upper GI procedure with barium ingestion and abdominal ultrasonography. While scheduling these diagnostic tests, the nurse must consider which factor?

The ultrasonography should be scheduled before the GI procedure.

A nurse epidemiologist examines the overall decrease in life expectancy related to obesity. What finding is true?

There is a 6-20 year decrease in overall life expectancy for those with obesity. Hinkle, J.L., & Cheever, K.H., Brunner & Suddarth's Textbook of Medical-Surgical Nursing, 14th ed., Philadelphia, Wolters Kluwer, 2018, Chapter 48: Assessment and Management of Patients with Obesity, p. 1359.

A nurse practitioner treating a patient who is diagnosed with hepatitis A should provide health care information. Which of the following statements are correct for this disorder? Select all that apply.

There is a 70% chance that jaundice will occur. Transmission of the virus is possible with oral-anal contact during sex. Typically there is a spontaneous recovery. Chapter 49: Assessment and Management of Patients With Hepatic Disorders - Page 1399 The incubation period for hepatitis A is 15 to 50 days, with an average of 28 days. The risk of cirrhosis occurs with hepatitis B.

The nurse determines which is a true statement regarding older clients, considering the age-related effects on their GI system?

They tend usually to have less control of the rectal sphincter.

The nurse determines which is a true statement regarding older clients, considering the age-related effects on the gastrointestinal (GI) system?

They usually have less control of the rectal sphincter.

When preparing to insert a nasogastric tube, the nurse determines the length of the tube to be inserted. The nurse places the distal tip of the tube at which location?

Tip of patient's nose Chapter 44: Digestive and Gastrointestinal Treatment Modalities - Page 1244 To measure the length of the nasogastric tube, the nurse first places the distal tip of the tubing at the tip of the patient's nose, extends the tube to the tragus of the ear, and then extends the tube straight down to the tip of the xiphoid process.

The physician has written the following orders for a new client admitted with pancreatitis: bed rest, nothing by mouth (NPO), and administration of total parenteral nutrition (TPN) . Which does the nurse attribute as the reason for NPO status?

To avoid inflammation of the pancreas Hinkle, J.L., & Cheever, K.H., Brunner & Suddarth's Textbook of Medical-Surgical Nursing, 14th ed., Philadelphia, Wolters Kluwer, 2018, Chapter 50: Assessment and Management of Patients With Biliary Disorders, Chart 50-4, p. 1446.

A client is admitted to the health care center with severe abdominal pain rated 10 on a 1-to-10 scale, tachycardia, hypertension, and muscle spasms. The nurse immediately administers morphine sulfate 4 mg slow intravenous pyelogram (IVP), as ordered. The nurse plans for which goal after administration of this mediation?

To increase the client's pain threshold Hinkle, J.L., & Cheever, K.H., Brunner & Suddarth's Textbook of Medical-Surgical Nursing, 14th ed., Philadelphia, Wolters Kluwer, 2018, Chapter 50: Assessment and Management of Patients With Biliary Disorders, Chart 50-4, p. 1446.

The nurse is caring for a patient with acute pancreatitis. The patient has an order for an anticholinergic medication. The nurse explains that the patient will be receiving that medication for what reason?

To reduce gastric and pancreatic secretions Hinkle, J.L., & Cheever, K.H., Brunner & Suddarth's Textbook of Medical-Surgical Nursing, 14th ed., Philadelphia, Wolters Kluwer, 2018, Chapter 50: Assessment and Management of Patients With Biliary Disorders, Nursing Implications, p. 1433.

A patient with bleeding esophageal varices has had pharmacologic therapy with Octreotide (Sandostatin) and endoscopic therapy with esophageal varices banding, but the patient has continued to have bleeding. What procedure that will lower portal pressure does the nurse prepare the patient for?

Transjugular intrahepatic portosystemic shunting (TIPS) Hinkle, J.L., & Cheever, K.H., Brunner & Suddarth's Textbook of Medical-Surgical Nursing, 14th ed., Philadelphia, Wolters Kluwer, 2018, Chapter 49: Assessment and Management of Patients With Hepatic Disorders, Pathophysiology, p. 1394.

The nurse is conducting a gastrointestinal assessment. When the client reports the presence of mucus and pus in the stool, the nurse assesses for additional signs/symptoms of which disease/condition?

Ulcerative colitis

Which of the following liver function studies is used to show the size of abdominal organs and the presence of masses?

Ultrasonography Hinkle, J.L., & Cheever, K.H., Brunner & Suddarth's Textbook of Medical-Surgical Nursing, 14th ed., Philadelphia, Wolters Kluwer, 2018, Chapter 49: Assessment and Management of Patients With Hepatic Disorders, Other Diagnostic Tests, p. 1382.

Which of the following diagnostic studies definitely confirms the presence of ascites?

Ultrasound of liver and abdomen Hinkle, J.L., & Cheever, K.H., Brunner & Suddarth's Textbook of Medical-Surgical Nursing, 14th ed., Philadelphia, Wolters Kluwer, 2018, Chapter 49: Assessment and Management of Patients With Hepatic Disorders, Other Diagnostic Tests, p. 1382.

A nurse cares for an obese client who wants more information about naltrexone/bupropion to help with weight loss. Which medical conditions does the nurse recognize as contraindications to this medication? Select all that apply.

Uncontrolled hypertension History of alcohol abuse History of bulimia Epilepsy Hinkle, J.L., & Cheever, K.H., Brunner & Suddarth's Textbook of Medical-Surgical Nursing, 14th ed., Philadelphia, Wolters Kluwer, 2018, Chapter 48: Assessment and Management of Patients with Obesity, p. 1365.

A client has a new order for metoclopramide. What extrapyramidal side effect should the nurse assess for in the client?

Uncontrolled rhythmic movements of the face or limbs

The nurse is inserting a nasoenteric tube for a patient with a paralytic ileus. How long does the nurse anticipate the tube will be required? (Select all that apply.)

Until bowel sound is present Until flatus is passed Until peristalsis is resumed

A patient with suspected esophageal varices is scheduled for an upper endoscopy with moderate sedation. After the procedure is performed, how long should the nurse withhold food and fluids?

Until the gag reflex returns Chapter 49: Assessment and Management of Patients With Hepatic Disorders - Page 1389 After the endoscopic examination, fluids are not given until the patient's gag reflex returns.

The nurse is teaching injection techniques for a client prescribed liraglutide for the treatment of obesity. Which areas of the body will the nurse suggest for injection sites? Select all that apply.

Upper thigh Abdomen Upper arm Lower thigh

A client with a nasogastric tube set to low intermittent suction is receiving D51/2NS at 100 mL/hr. The nurse has identified a nursing diagnosis of deficient fluid volume. Which of the following are data that support this diagnosis? Select all that apply.

Urine output that decreased from 60 to 40 mL/hr Heart rate that increased from 82 to 98 beats/min within 2 hours Fluid output of 2150 mL and total fluid intake of 2000 mL for the past 24 hours

A client being treated for pancreatitis faces the risk of atelectasis. Which of the following interventions would be important to implement to minimize this risk?

Use incentive spirometry every hour. Hinkle, J.L., & Cheever, K.H., Brunner & Suddarth's Textbook of Medical-Surgical Nursing, 14th ed., Philadelphia, Wolters Kluwer, 2018, Chapter 50: Assessment and Management of Patients With Biliary Disorders, Preoperative Measures, p. 1436.

A female client with chronic hepatitis B has been prescribed recombinant interferon alfa-2b in combination with ribavirin. Which of the following instructions should a nurse provide this client?

Use strict birth control methods. Hinkle, J.L., & Cheever, K.H., Brunner & Suddarth's Textbook of Medical-Surgical Nursing, 14th ed., Philadelphia, Wolters Kluwer, 2018.

The nurse is assessing a client for constipation. Which review should the nurse conduct first to identify the cause of constipation?

Usual pattern of elimination

A client who is postoperative from bariatric surgery is diagnosed with bile reflux after reports of severe epigastric pain and vomiting of bilious material. What statements are true regarding this condition? Select all that apply.

Usually occurs after disruption of the pylorus. Symptoms are usually relieved after pharmacological treatment. Hinkle, J.L., & Cheever, K.H., Brunner & Suddarth's Textbook of Medical-Surgical Nursing, 14th ed., Philadelphia, Wolters Kluwer, 2018, Chapter 48: Assessment and Management of Patients with Obesity, p. 1373.

Cardiac complications, which may occur following resection of an esophageal tumor, are associated with irritation of which nerve at the time of surgery?

Vagus

A nurse is providing care for a client recovering from gastric bypass surgery. During assessment, the client exhibits pallor, perspiration, palpitations, headache, and feelings of warmth, dizziness, and drowsiness. The client reports eating 90 minutes ago. What will the nurse suspect?

Vasomotor symptoms associated with dumping syndrome

Which medication is used to decrease portal pressure, halting bleeding of esophageal varices?

Vasopressin Hinkle, J.L., & Cheever, K.H., Brunner & Suddarth's Textbook of Medical-Surgical Nursing, 14th ed., Philadelphia, Wolters Kluwer, 2018, Chapter 49: Assessment and Management of Patients With Hepatic Disorders, p. 1390.

The nurse knows that the client with cholelithiasis can have a nutritional deficiency. The obstruction of bile flow due to cholelithiasis can interfere with the absorption of

Vitamin A Hinkle, J.L., & Cheever, K.H., Brunner & Suddarth's Textbook of Medical-Surgical Nursing, 14th ed., Philadelphia, Wolters Kluwer, 2018, Chapter 50: Assessment and Management of Patients With Biliary Disorders, Vitamin Deficiency, p. 1432.

During assessment, a patient with chronic liver dysfunction tells the nurse that he is experiencing spontaneous episodes of bleeding and has noticed increased areas of bruising on his chest and arms. The nurse suspects a deficiency in:

Vitamin K Hinkle, J.L., & Cheever, K.H., Brunner & Suddarth's Textbook of Medical-Surgical Nursing, 14th ed., Philadelphia, Wolters Kluwer, 2018, Chapter 49: Assessment and Management of Patients With Hepatic Disorders, Vitamin Deficiency, p. 1397.

A client with carcinoma of the head of the pancreas is scheduled for surgery. Which of the following should a nurse administer to the client before surgery?

Vitamin K Hinkle, J.L., & Cheever, K.H., Brunner & Suddarth's Textbook of Medical-Surgical Nursing, 14th ed., Philadelphia, Wolters Kluwer, 2018, Chapter 50: Assessment and Management of Patients With Biliary Disorders, Medical Management, p. 1452.

A patient with severe chronic liver dysfunction comes to the clinic with bleeding of the gums and blood in the stool. What vitamin deficiency does the nurse suspect the patient may be experiencing?

Vitamin K deficiency Hinkle, J.L., & Cheever, K.H., Brunner & Suddarth's Textbook of Medical-Surgical Nursing, 14th ed., Philadelphia, Wolters Kluwer, 2018, Chapter 49: Assessment and Management of Patients With Hepatic Disorders, Vitamin Deficiency, p. 1397.

A client is postoperative day 1 following gastrostomy. The nurse is planning interventions to address the nursing diagnosis of Risk for Infection Related to Presence of Wound and Tube. What intervention is most appropriate?

Wash the area around the tube with soap and water daily.

The nurse is administering Cephulac (lactulose) to decrease the ammonia level in a patient who has hepatic encephalopathy. What should the nurse carefully monitor for that may indicate a medication overdose?

Watery diarrhea Hinkle, J.L., & Cheever, K.H., Brunner & Suddarth's Textbook of Medical-Surgical Nursing, 14th ed., Philadelphia, Wolters Kluwer, 2018, Chapter 49: Assessment and Management of Patients With Hepatic Disorders, Medical Management, p. 1395.

A nurse is providing discharge instructions for a client who fell from a bicycle, resulting in a fractured jaw. The client underwent surgical intervention with rigid fixation. What teaching should the nurse include with client education?

Ways to obtain nutritional supplementation

Which of the following assessment findings would be most important for indicating dumping syndrome in a postgastrectomy client?

Weakness, diaphoresis, diarrhea 90 minutes after eating

The client is receiving 50% dextrose parenteral nutrition with fat emulsion therapy through a peripherally inserted central catheter (PICC). The nurse has developed a care plan for the nursing diagnosis "Risk for infection related to contamination of the central catheter site or infusion line." The nurse includes the intervention

Wear a face mask during dressing changes.

The client cannot tolerate oral feedings due to an intestinal obstruction and is NPO. A central line has been inserted, and the client is being started on parenteral nutrition (PN). What actions would the nurse perform while the client receives PN? Select all that apply.

Weigh the client every day. Check blood glucose level every 6 hours. Document intake and output.

The nurse checks residual content before each intermittent tube feeding. When should the patient be reassessed?

When the residual is greater than 200 mL Chapter 44: Digestive and Gastrointestinal Treatment Modalities - Page 1251 Although a residual volume of 200 mL or greater is generally considered a cause for concern in patients at high risk for aspiration, feedings do not necessarily need to be withheld in all patients.

After a client received a diagnosis of gastric cancer, the surgical team decides that a Billroth II would be the best approach to treatment. How would the nurse explain this procedure to the family?

Wide resection of the middle and distal portions of the stomach with removal of about 75% of the stomach

A nurse is caring for a client admitted with acute pancreatitis. Which nursing action is most appropriate for a client with this diagnosis?

Withholding all oral intake, as ordered, to decrease pancreatic secretions Hinkle, J.L., & Cheever, K.H., Brunner & Suddarth's Textbook of Medical-Surgical Nursing, 14th ed., Philadelphia, Wolters Kluwer, 2018, Chapter 50: Assessment and Management of Patients With Biliary Disorders, Medical Management, p. 1442.

The nurse confirms placement of a client's nasogastric (NG) tube using a combination of visual and pH assessment of the aspirate. The nurse determines that the NG tube remains properly placed when the pH of the aspirate is

acidic

A middle-aged obese female presents to the ED with severe radiating right-sided flank pain, nausea, vomiting, and fever. A likely cause of these symptoms is:

acute cholecystitis Hinkle, J.L., & Cheever, K.H., Brunner & Suddarth's Textbook of Medical-Surgical Nursing, 14th ed., Philadelphia, Wolters Kluwer, 2018, Chapter 48: Assessment and Management of Patients with Obesity, p. 1430.

A client has just been diagnosed with hepatitis A. On assessment, the nurse expects to note:

anorexia, nausea, and vomiting. Hinkle, J.L., & Cheever, K.H., Brunner & Suddarth's Textbook of Medical-Surgical Nursing, 14th ed., Philadelphia, Wolters Kluwer, 2018, Chapter 49: Assessment and Management of Patients With Hepatic Disorders, Clinical Manifestations, p. 1399.

A client with hepatitis C develops liver failure and GI hemorrhage. The blood products that most likely bring about hemostasis in the client are:

cryoprecipitate and fresh frozen plasma. Chapter 49: Assessment and Management of Patients With Hepatic Disorders - Page 1403-1404 The liver is vital in the synthesis of clotting factors, so when it's diseased or dysfunctional, as in hepatitis C, bleeding occurs. Treatment consists of administering blood products that aid clotting. These products include fresh frozen plasma containing fibrinogen and cryoprecipitate, which have most of the clotting factors. Although administering whole blood, albumin, and packed cells will contribute to hemostasis, these products aren't specifically used to treat hemostasis. Although platelets may be helpful, the best answer is cryoprecipitate and fresh frozen plasma.

The nurse on an evidence-based practice council makes recommendations to ensure patency of nontunneled central venous lines. The nurse recommends that daily saline and diluted heparin flushes be used in which situation?

daily when not in use

A preoperative client scheduled to have an open cholecystectomy says to the nurse, "The doctor said that after surgery, I will have a tube in my nose that goes into my stomach. Why do I need that?" What most common reason for a client having a nasogastric tube in place after abdominal surgery should the nurse include in a response?

decompression Hinkle, J.L., & Cheever, K.H., Brunner & Suddarth's Textbook of Medical-Surgical Nursing, 14th ed., Philadelphia, Wolters Kluwer, 2018, Chapter 49: Assessment and Management of Patients With Hepatic Disorders, p. 1452.

A client has a 10-year history of Crohn's disease and is seeing the physician due to increased diarrhea and fatigue. What is the recommended dietary approach to treat Crohn's disease?

dietary approach varies.

The client is experiencing swallowing difficulties and is now scheduled to receive a gastric feeding. The client has the following oral medications prescribed: furosemide, digoxin, enteric coated aspirin, and vitamin E. The nurse would withhold which medication?

enteric coated aspirin

The client is experiencing swallowing difficulties and is now scheduled to receive a gastric feeding. The client has the following oral medications prescribed: furosemide, digoxin, enteric coated aspirin, and vitamin E. The nurse would withhold which medication?`

enteric coated aspirin

The nurse recognizes that medium-length nasoenteric tubes are used for

feeding. Placement of the tube must be verified prior to any feeding. A gastric sump and nasoenteric tube are used for gastrointestinal decompression. Nasoenteric tubes are used for feeding. Gastric sump tubes are used to decompress the stomach and keep it empty.

A client has received a diagnosis of portal hypertension. What does portal hypertension treatment aim to reduce? Select all that apply.

fluid accumulation venous pressure Chapter 49: Assessment and Management of Patients With Hepatic Disorders - Page 1385

The nurse is conducting a community education program on peptic ulcer disease prevention. The nurse includes that the most common cause of peptic ulcers is:

gram-negative bacteria.

What does the nurse recognize as clinical manifestations consistent with ascites? Select all that apply.

increased abdominal girth rapid weight gain visible distended veins stretch marks Chapter 49: Assessment and Management of Patients With Hepatic Disorders - Page 1385-1387 The presence and extent of ascites are assessed by percussion of the abdomen. When fluid has accumulated in the peritoneal cavity, the flanks bulge when the patient assumes a supine position. Increased abdominal girth and rapid weight gain are common presenting symptoms of ascites. The patient may be short of breath and uncomfortable from the enlarged abdomen, and striae and distended veins may be visible over the abdominal wall. Foul-smelling breath is not a clinical manifestation of ascites.

A nurse is assessing a client and obtains the following findings: abdominal discomfort, mild diarrhea, blood pressure of 100/80 mm Hg, pulse rate of 88 beats/minute, respiratory rate of 20 breaths/minute, temperature 100° F (37.8° C). What diagnosis will the nurse suspect for this client?

inflammatory bowel disease (IBD)

A resident at a long-term care facility lost the ability to swallow following a stroke 4 years ago. The client receives nutrition via a PEG tube, has adapted well to the tube feedings, and remains physically and socially active. Occasionally, the client develops constipation that requires administration of a laxative to restore regular bowel function. What is the most likely cause of this client's constipation?

lack of free water intake

A physician orders lactulose (Cephulac), 30 ml three times daily, when a client with cirrhosis develops an increased serum ammonia level. To evaluate the effectiveness of lactulose, the nurse should monitor:

level of consciousness (LOC). Hinkle, J.L., & Cheever, K.H., Brunner & Suddarth's Textbook of Medical-Surgical Nursing, 14th ed., Philadelphia, Wolters Kluwer, 2018, Chapter 49: Assessment and Management of Patients With Hepatic Disorders, Medical Management, p. 1395.

A physician has ordered a liver biopsy for a client with cirrhosis whose condition has recently deteriorated. The nurse reviews the client's recent laboratory findings and recognizes that the client is at risk for complications due to:

low platelet count. Chapter 49: Assessment and Management of Patients With Hepatic Disorders - Page 1382 Prolonged prothrombin time (PT) and low platelet count place the client at high risk for hemorrhage. The client may receive intravenous (IV) administration of vitamin K or infusions of platelets before liver biopsy to reduce the risk of bleeding.

A nurse is providing education to a client with GERD. The client asks what measures can be taken independently to help reduce the symptoms. Which interventions would the nurse recommend? Select all that apply.

maintaining an upright position following meals avoiding foods that intensify symptoms

Lisa Bentley, a 32-year-old teacher, presents to the gastroenterology office where you work. She is known to have a history of Crohn's disease, and you have met with her several times to discuss the various health concerns that she has related to her diagnosis. When talking with the client, the nurse explains that having a GI disorder doesn't mean her problems are limited to the one area that is diseased but might also involve all of the following except ________.

metabolism

Lisa Bentley, a 32-year-old teacher, presents to the gastroenterology office where you work. She is known to have a history of Crohn's disease, and you have met with her several times to discuss the various health concerns that she has related to her diagnosis. When talking with the client, the nurse explains that having a GI disorder doesn't mean her problems are limited to the one area that is diseased but might also involve all of the following except ________.

metabolism The client with a GI disorder may experience a wide variety of health problems that involve disturbances of ingestion, digestion, absorption, and elimination. The client with a GI disorder may experience health problems that involve disturbances of ingestion, digestion, absorption, and elimination.

A nurse educator is providing an in-service to a group of nurses working on a medical floor that specializes in liver disorders. What is an important education topic regarding ingestion of medications?

metabolism of medications Hinkle, J.L., & Cheever, K.H., Brunner & Suddarth's Textbook of Medical-Surgical Nursing, 14th ed., Philadelphia, Wolters Kluwer, 2018, Chapter 49: Assessment and Management of Patients With Hepatic Disorders, p. 1397.

The nurse is providing discharge instructions for a slightly overweight client seen in the Emergency Department with gastroesophageal reflux disease (GERD). The nurse notes in the client's record that the client is taking carbidopa/levodopa. Which order for the client by the health care provider should the nurse question?

metoclopramide

Understanding the definition of eating disorders is important in communicating facts while managing these conditions. Which is not an eating disorder?

obesity

The Zollinger-Ellison syndrome (ZES) consists of severe peptic ulcers, extreme gastric hyperacidity, and gastrin-secreting benign or malignant tumors of the pancreas. The nurse recognizes that an agent that is used to decrease bleeding and decrease gastric acid secretions is

octreotide (Sandostatin)

Most of the liver's metabolic functions are performed by:

parenchymal cells. Chapter 49: Assessment and Management of Patients With Hepatic Disorders - Page 1382 The parenchymal cells perform most of the liver's metabolic functions.

A client with advanced cirrhosis has a prothrombin time (PT) of 15 seconds, compared with a control time of 11 seconds. The nurse expects to administer:

phytonadione (Mephyton). Hinkle, J.L., & Cheever, K.H., Brunner & Suddarth's Textbook of Medical-Surgical Nursing, 14th ed., Philadelphia, Wolters Kluwer, 2018, Chapter 49: Assessment and Management of Patients With Hepatic Disorders, Protein Metabolism, p. 1379.

A client who has worked for a company that produces paint and varnishing compounds for 24 years is visiting the clinic reporting chronic fatigue, dyspepsia, diarrhea, and a recently developing yellowing of the skin and sclera. The client reports clay-colored stools and frequent nosebleeds. Which type of cirrhosis is the likely cause of the client's symptoms?

postnecrotic

A client who has worked for a company that produces paint and varnishing compounds for 24 years is visiting the clinic reporting chronic fatigue, dyspepsia, diarrhea, and a recently developing yellowing of the skin and sclera. The client reports clay-colored stools and frequent nosebleeds. Which type of cirrhosis is the likely cause of the client's symptoms?

postnecrotic Hinkle, J.L., & Cheever, K.H., Brunner & Suddarth's Textbook of Medical-Surgical Nursing, 14th ed., Philadelphia, Wolters Kluwer, 2018, Chapter 49: Assessment and Management of Patients With Hepatic Disorders, p. 1406.

A nurse in the surgical ICU just received a client from recovery following a Whipple procedure. Which nursing diagnoses should the nurse consider when caring for this acutely ill client? Select all that apply.

potential for infection acute pain and discomfort alterations in respiratory function Chapter 49: Assessment and Management of Patients With Hepatic Disorders - Page 1452 Monitor for potential for infection related to invasive procedure and poor physical condition. Monitor for pain related to extensive surgical incision. Monitor for alterations in respiratory function related to extensive surgical incisions, immobility, and prolonged anesthesia. Client is at risk for fluid volume deficit related to hemorrhage and loss of fluids.

Ac client with a lengthy history of alcohol addiction is diagnosed with cirrhosis. The nurse emphasizes that the principal goal of cirrhosis therapy is:

preserving liver function. Hinkle, J.L., & Cheever, K.H., Brunner & Suddarth's Textbook of Medical-Surgical Nursing, 14th ed., Philadelphia, Wolters Kluwer, 2018, Chapter 49: Assessment and Management of Patients With Hepatic Disorders, p. 1408.

A client who was recently diagnosed with carcinoma of the pancreas and is having a procedure in which the head of the pancreas is removed. In addition, the surgeon will remove the duodenum and stomach, redirecting the flow of secretions from the stomach, gallbladder, and pancreas into the middle section of the small intestine. What procedure is this client having performed?

radical pancreatoduodenectomy Hinkle, J.L., & Cheever, K.H., Brunner & Suddarth's Textbook of Medical-Surgical Nursing, 14th ed., Philadelphia, Wolters Kluwer, 2018, Chapter 49: Assessment and Management of Patients With Hepatic Disorders, p. 1452.

The nurse inspects a client's tongue. Which finding would the nurse evaluate as an indication of potential oral cancer?

red plaque on undersurface of tongue

The nurse cares for a client who receives feedings through a nasogastric (NG) tube and assesses the client for signs and symptoms of pulmonary complications. The nurse determines the client may be experiencing pulmonary complications when which sign is noted?

respiratory rate of 30

A nursing assessment of a client with peritonitis reveals hypotension, tachycardia, and signs and symptoms of dehydration. What else would the nurse expect to find?

severe abdominal pain with direct palpation or rebound tenderness

Which symptoms will a nurse observe most commonly in clients with pancreatitis?

severe, radiating abdominal pain Hinkle, J.L., & Cheever, K.H., Brunner & Suddarth's Textbook of Medical-Surgical Nursing, 14th ed., Philadelphia, Wolters Kluwer, 2018, Chapter 49: Assessment and Management of Patients With Hepatic Disorders, p. 1441.

Which procedure is performed to examine and visualize the lumen of the small bowel?

small bowel enteroscopy

A nurse is teaching a client with malabsorption syndrome about the disorder and its treatment. The client asks which part of the GI tract absorbs food. What is the nurse's best response?

small intestine

An older adult client in a long term care facility is concerned about bowel regularity. During a client education session, the nurse reinforces the medically acceptable definition of "regularity." What is the actual measurement of "regular"?

stool consistency and client comfort

A client with acute liver failure exhibits confusion, a declining level of consciousness, and slowed respirations. The nurse finds him very difficult to arouse. The diagnostic information which best explains the client's behavior is:

subnormal serum glucose and elevated serum ammonia levels. Hinkle, J.L., & Cheever, K.H., Brunner & Suddarth's Textbook of Medical-Surgical Nursing, 14th ed., Philadelphia, Wolters Kluwer, 2018, Chapter 49: Assessment and Management of Patients With Hepatic Disorders, Chart 49-11, p. 1416.

What symptoms of perforation might the nurse observe in a client with an intestinal obstruction? Select all that apply.

sudden, sustained abdominal pain abdominal distention

A nurse is doing a physical assessment on a client with a GI disorder. Which position will the nurse most likely ask the client to assume when performing an abdominal examination?

supine with knees flexed slightly

A client who had oral cancer has had extensive surgery to excise the malignancy. Although surgery was deemed successful, it was quite disfiguring and incapacitating. What is essential to this client and family?

time to mourn, accept, and adjust to the loss

The nurse is admitting a client whose medication regimen includes regular injections of vitamin B12. The nurse should question the client about a history of:

total gastrectomy.

A nurse is helping a physician insert a subclavian central line. After the physician has gained access to the subclavian vein, he connects a 10-ml syringe to the catheter and withdraws a sample of blood. He then disconnects the syringe from the port. Suddenly, the client becomes confused, disoriented, and pale. The nurse suspects an air embolus. She should:

turn the client on his left side and place the bed in Trendelenburg's position.

The nurse monitors a client with nasoenteric intubation. When should the nurse contact the physician?

urinary output 20 mL/hr

A client with viral hepatitis A is being treated in an acute care facility. Because the client requires enteric precautions, the nurse should:

wash her hands after touching the client. Hinkle, J.L., & Cheever, K.H., Brunner & Suddarth's Textbook of Medical-Surgical Nursing, 14th ed., Philadelphia, Wolters Kluwer, 2018, Chapter 49: Assessment and Management of Patients With Hepatic Disorders, Prevention, p. 1399.

The nurse recognizes which change of the gastrointestinal system is an age-related change?

weakened gag reflex

A client with cholelithiasis has a gallstone lodged in the common bile duct. When assessing this client, the nurse expects to note:

yellow sclerae. Hinkle, J.L., & Cheever, K.H., Brunner & Suddarth's Textbook of Medical-Surgical Nursing, 14th ed., Philadelphia, Wolters Kluwer, 2018, Chapter 50: Assessment and Management of Patients With Biliary Disorders, Maintaining Skin Integrity and Promoting Biliary Drainage, p. 1439.


Conjuntos de estudio relacionados

SLUDGEM-BBB (CBR Nerve Agent - Muscarinic Symptoms)

View Set

everfi module 1-6 (business finance)

View Set

Chapter 15: Intraoperative Nursing Management

View Set

Chapter 5:Frictions in the Labor Market

View Set

Ch. 3 Exam Questions (Life) - Legal Concepts of the Insurance Contract

View Set

6.6 Gamete Formation and Genetic Variation

View Set

SCIENCE AND TECHNOLOGY DURING THE ANCIENT TIMES

View Set